{ "cells": [ { "cell_type": "markdown", "metadata": {}, "source": [ "# RANSAC sphere fitting\n", "\n", "The most difficult part of RANSAC is to create a model needed for fitting. Let us start with this step now.\n", "\n", "## Model creation\n", "\n", "### Model based on 4 points\n", "\n", "First let us fit a sphere to 4 points $P_i(x_i,y_i,z_i)$ with known positions with an algebraic method. This problem has a unique solution for 4 points $i=1,...,4$ since it leads to a linear system of equations with a unique solution (see Paláncz, Molnár (2012) [Wolfram Library Archive](http://library.wolfram.com/infocenter/MathSource/8491/)).\n", "\n", "Sphere with radius $R$ fits each point:\n", "$$ \\sqrt{(x_i-x_0)^2+(y_i-y_0)^2+(z_i-z_0)^2} -R = 0 $$\n", "\n", "hence for each point\n", "$$ \\sqrt{(x_i-x_0)^2+(y_i-y_0)^2+(z_i-z_0)^2} +R = 2R = c $$\n", "\n", "and the two equations are multiplied that yields\n", "$$ (x_i-x_0)^2+(y_i-y_0)^2+(z_i-z_0)^2 -R^2 = 0 $$\n", "\n", "If we introduce a new variable $\\alpha$ instead of $R$ the following *linear* system need to be solved for parameters $\\mathbf{p}=[x_0, y_0, z_0, \\alpha]$\n", "$$ x_i^2 + y_i^2 + z_i^2 -2x_0 x_i -2y_0 y_i - 2z_0 z_i + \\alpha = 0$$\n", "where\n", "$$ \\alpha = x_0^2 + y_0^2 + z_0^2 -R^2$$\n", "\n", "This solution was implemented in the following Python function:\n" ] }, { "cell_type": "code", "execution_count": 1, "metadata": {}, "outputs": [], "source": [ "def sph4model(X):\n", " \"\"\"\n", " sph4model(X) fits a sphere to four points given by rows of matrix X\n", " Reference: Paláncz, Molnár (2012): Fitting sphere to quantized depth information\n", " Input: X: 4x3 matrix, coordinates [x,y,z] of points stored as its rows\n", " Output: a vector [x0, y0, z0, r] with four elements, where:\n", " r: radius of sphere\n", " x0, y0, z0: coordinates of the centre of sphere\n", " Remark: If there is no solution (given 4 points lie approximately in a plane) then\n", " function returns None\n", " \"\"\"\n", "\n", " # subtract mean\n", " Xm = np.mean(X,axis=0)\n", " Xs = X-Xm\n", "\n", " # matrix M\n", " M=np.hstack((-2.0*Xs,np.ones((4,1))))\n", " # vector h\n", " h=-np.sum(Xs**2,axis=1) # row sums\n", " # check condition number of M:\n", " if np.linalg.cond(M) > 1000:\n", " return None\n", " # check determinant:\n", " if np.abs(np.linalg.det(M)) < 1.0e-15:\n", " return None\n", " p=np.linalg.solve(M,h)\n", " # radius of sphere\n", " r=np.sqrt(np.sum(p[0:3]**2)-p[3])\n", " # centre coordinates\n", " c=p[0:3]+Xm\n", " return np.hstack((c,r))\n" ] }, { "cell_type": "markdown", "metadata": {}, "source": [ "Check this function!" ] }, { "cell_type": "code", "execution_count": 2, "metadata": {}, "outputs": [ { "name": "stdout", "output_type": "stream", "text": [ "[0.54748534 0.49907036 0.62914037 0.41285427]\n" ] } ], "source": [ "import numpy as np\n", "\n", "X = np.array([[0.655416348349, 0.200995185452, 0.893622387647], \n", " [0.281886543129, 0.525000382971, 0.314126774995], \n", " [0.444615678299, 0.299474455628, 0.282689857776], \n", " [0.883227485267, 0.270905975731, 0.704419015849]])\n", "\n", "p = sph4model(X)\n", "print(p)" ] }, { "cell_type": "markdown", "metadata": {}, "source": [ "### Least squares fit to more than 4 points\n", "\n", "Least squares method can be used to fit a sphere to more than 4 points. This procedure is implemented in the following Python functions." ] }, { "cell_type": "code", "execution_count": 3, "metadata": {}, "outputs": [], "source": [ "def svdsolve(a,b):\n", " u,s,v = np.linalg.svd(a)\n", " c = np.dot(u.T,b)\n", " w = np.divide(c[:len(s)],s)\n", " x = np.dot(v.T,w)\n", " return x\n", "\n", "def sph4fit(X):\n", " \"\"\"\n", " sph4model(X) fits a sphere to four or more points given by rows of matrix X\n", " Reference: Paláncz, Molnár (2012): Fitting sphere to quantized depth information\n", " Input: X: nx3 matrix, coordinates [x,y,z] of n points stored as its rows\n", " Output: a vector [x0, y0, z0, r] with four elements, where:\n", " r: radius of sphere\n", " x0, y0, z0: coordinates of the centre of sphere\n", " Remark: If there is no good solution then this function returns None\n", " \"\"\"\n", "\n", " # subtract mean\n", " Xm = np.mean(X,axis=0)\n", " Xs = X-Xm\n", " n = Xs.shape[0]\n", "\n", " # matrix M\n", " M=np.hstack((-2.0*Xs,np.ones((n,1))))\n", " # vector h\n", " h=-np.sum(Xs**2,axis=1) # sum rows\n", " # check condition number of M:\n", " if np.linalg.cond(M) > 1000:\n", " return None\n", " p=svdsolve(M,h)\n", " # radius of sphere\n", " r=np.sqrt(np.sum(p[0:3]**2)-p[3])\n", " # centre coordinates\n", " c=p[0:3]+Xm\n", " return np.hstack((c,r))" ] }, { "cell_type": "markdown", "metadata": {}, "source": [ "Check this function also:" ] }, { "cell_type": "code", "execution_count": 4, "metadata": {}, "outputs": [ { "name": "stdout", "output_type": "stream", "text": [ "[0.50949211 0.39356424 0.54173242 0.5305028 ]\n" ] } ], "source": [ "# 10 points\n", "Xp = np.array([[0.236419323876, 0.94539908057, 0.448603985892], \n", " [0.132256383655, 0.0585420373716,0.189678073452], \n", " [0.777527902317, 0.237423931798, 0.978964303253], \n", " [0.0367820896797,0.191109494886, 0.64247854129 ],\n", " [0.233535838244, 0.0910807008584,0.899334281424], \n", " [0.983635710836, 0.211019679874, 0.552776867983], \n", " [0.494141778444, 0.579783629292, 0.811875091404], \n", " [0.983877207446, 0.262955546077, 0.993804390297], \n", " [0.901832244664, 0.240795911508, 0.635966928702], \n", " [0.151086132434, 0.251504104295, 0.33397148847 ]])\n", "\n", "p10 = sph4fit(Xp)\n", "print(p10)" ] }, { "cell_type": "markdown", "metadata": {}, "source": [ "To see the fit, let us calculate (signed or unsigned) deviations from the surface of the sphere using the following function:" ] }, { "cell_type": "code", "execution_count": 5, "metadata": {}, "outputs": [], "source": [ "def sphdist(sph,X,pos=1):\n", " \"\"\"\n", " sphdist(sph,X) calculate distances of points X from sphere sph\n", " sph: 4-element vector: [x0, y0, z0, r]\n", " x0, y0, z0: centre of sphere\n", " r: radius of sphere\n", " X: (n,3) matrix, coordinates of n points\n", " pos: if pos=0 calculate signed, otherwise unsigned deviations\n", " \"\"\"\n", " r = sph[3]\n", " c = sph[0:3]\n", " # distance from centre squared\n", " dc = np.sum((X-c)**2,axis=1)\n", " if pos==1:\n", " dist = np.abs(np.sqrt(dc)-r)\n", " else:\n", " dist = np.sqrt(dc)-r\n", " return dist" ] }, { "cell_type": "markdown", "metadata": {}, "source": [ "Residuals of the 4-point fit at the points:" ] }, { "cell_type": "code", "execution_count": 6, "metadata": {}, "outputs": [ { "name": "stdout", "output_type": "stream", "text": [ "[0.00000000e+00 5.55111512e-17 5.55111512e-17 0.00000000e+00]\n" ] } ], "source": [ "print(sphdist(p,X))" ] }, { "cell_type": "markdown", "metadata": {}, "source": [ "The same for 10-point fit:" ] }, { "cell_type": "code", "execution_count": 7, "metadata": {}, "outputs": [ { "name": "stdout", "output_type": "stream", "text": [ "[0.09220366 0.08471174 0.00558926 0.00648709 0.01312137 0.02231322\n", " 0.20203613 0.13768049 0.09905282 0.09255243]\n" ] } ], "source": [ "print(sphdist(p10,Xp))" ] }, { "cell_type": "markdown", "metadata": {}, "source": [ "When there are more outliers, least squares fit is not satisfactory. We use RANSAC instead (Fischler M A, Bolles R C, Random Sample Consensus: A paradigm for model fitting with applications to image analysis and automated cartography, Communications of the ACM, 24(6): 381-395, 1981, [PDF](http://www.cs.ait.ac.th/~mdailey/cvreadings/Fischler-RANSAC.pdf)." ] }, { "cell_type": "markdown", "metadata": {}, "source": [ "Until now we considered modeling and solution of the sphere fitting problem by least squares. In practice, however, our data often contain a large percent of outliers. Fischler and Bolles introduced in their 1981 paper an excellent method of parameter estimation from such data that can advantageously be applied for the present problem.\n", "\n", "As we have seen in the [ellipse fitting problem](https://nbviewer.jupyter.org/github/gyulat/Korszeru_matek/blob/master/RANSAC_en.ipynb), RANSAC consists of the following steps:\n", "\n", "1. Generate a prescibed number of models (hypotheses) $M$, each from a minimum number $n$ of data necessary for the unique creation of a model (determination of model parameters) \n", "2. Evaluate each model (hypothesis):\n", " * calculate deviations of data with respect to the model\n", " * data with deviations under a specified threshold are considered conform to the model (elements of the consensus set) \n", "3. Select the model with maximum number of elements of its consensus set (original RANSAC)\n", "\n", "The following algorithms (functions) are required for computing with RANSAC:\n", "* model creation using a minimum number $n$ of data\n", "* calculation of fit residuals with respect to a particular model\n", "\n", "These functions have already been implemented for the sphere fitting problem, therefore we only need to integrate them into the [ransac](http://scikit-image.org/docs/stable/api/skimage.measure.html?highlight=ransac#skimage.measure.ransac) function of the [scikit-image](http://scikit-image.org/) library.\n", "\n", "Description of the function \n", "```python\n", "sm.ransac(data, model_class, min_samples, residual_threshold) \n", "```\n", "tells us that it has got four required parameters. \n", "1. data matrix `data(N, D)` where `N` denotes the number and `D` is dimension of data,\n", "2. Python implementation of the class with required member functions `success = estimate(*data)` and `residuals(*data)`,\n", "3. integer `min_samples`, which is the same as $n$.\n", "4. `residual_threshold`, maximum distance (residual) of conform data." ] }, { "cell_type": "markdown", "metadata": {}, "source": [ "### SphereModel Python class\n", "\n", "In our case the requred class `SphereModel` for RANSAC contains a simple constructor and two member functions. Member functions require also code of the `svdsolve` function. Member variable `params` stores various parameters (centre and radius) of the best fitting sphere." ] }, { "cell_type": "code", "execution_count": 8, "metadata": {}, "outputs": [], "source": [ "class SphereModel:\n", " \"\"\"sphere fitting model \n", " \"\"\"\n", "\n", " def __init__(self):\n", " self.params = np.zeros(4)\n", "\n", " def estimate(self, X):\n", " \"\"\"\n", " estimate(X) fits a sphere to four or more points given by rows of matrix X\n", " Reference: Paláncz, Molnár (2012): Fitting sphere to quantized depth information\n", " Input: X: nx3 matrix, coordinates [x,y,z] of n points stored as its rows\n", " Output: a vector [x0, y0, z0, r] with four elements, where:\n", " r: radius of sphere\n", " x0, y0, z0: coordinates of the centre of sphere\n", " Remark: If there is no good solution then this function returns None\n", " \"\"\"\n", "\n", " # subtract mean\n", " Xm = np.mean(X,axis=0)\n", " Xs = X-Xm\n", " n = Xs.shape[0]\n", "\n", " # matrix M\n", " M=np.hstack((-2.0*Xs,np.ones((n,1))))\n", " # vector h\n", " h=-np.sum(Xs**2,axis=1) # sum of rows\n", " # check condition number of M:\n", " if np.linalg.cond(M) > 1000:\n", " return False\n", " self.params=svdsolve(M,h)\n", " # radius of sphere\n", " r=np.sqrt(np.sum(self.params[0:3]**2)-self.params[3])\n", " self.params[3]=r\n", " # centre\n", " self.params[0:3] = self.params[0:3]+Xm\n", " return True\n", " \n", " def residuals(self, X, pos=1):\n", " \"\"\"\n", " residuals(X) calculate distances of points X from sphere sph\n", " sph: 4-element vector: [x0, y0, z0, r]\n", " x0, y0, z0: centre of sphere\n", " r: radius of sphere\n", " X: (n,3) matrix, coordinates of n points\n", " pos: if pos=0 calculate signed, otherwise unsigned deviations\n", " \"\"\"\n", " r = self.params[3]\n", " c = self.params[0:3]\n", " # distance from centre squared\n", " dc = np.sum((X-c)**2,axis=1)\n", " if pos==1:\n", " dist = np.abs(np.sqrt(dc)-r)\n", " else:\n", " dist = np.sqrt(dc)-r\n", " return dist\n", " \n", "def svdsolve(a,b):\n", " u,s,v = np.linalg.svd(a)\n", " c = np.dot(u.T,b)\n", " w = np.divide(c[:len(s)],s)\n", " x = np.dot(v.T,w)\n", " return x" ] }, { "cell_type": "markdown", "metadata": {}, "source": [ "### Test data\n", "\n", "We use laser scanner [data](./dat/sphere.dat) of a reference sphere (data from [Nikolett Rehány](http://www.epito.bme.hu/rehany-nikolett)). First plot these data:" ] }, { "cell_type": "code", "execution_count": 9, "metadata": {}, "outputs": [ { "data": { "image/png": "iVBORw0KGgoAAAANSUhEUgAAAcwAAAHBCAYAAADkRYtYAAAABHNCSVQICAgIfAhkiAAAAAlwSFlzAAALEgAACxIB0t1+/AAAADh0RVh0U29mdHdhcmUAbWF0cGxvdGxpYiB2ZXJzaW9uMy4xLjMsIGh0dHA6Ly9tYXRwbG90bGliLm9yZy+AADFEAAAgAElEQVR4nOy9d5hkZZn3/z2Vc+qc08x0nNRxyAOSZIARJIjg4iKua1jxfXd1WVH3lV1kcHXf1UHF3RdGVnYBRf0RJDoMwhKGAWboNB2rc67YldM5vz96zumq6qruU6m7uvv5XJcXTnfVqdOnqp7vue/nvr83xTAMCAQCgUAgrI5go0+AQCAQCITNABFMAoFAIBB4QASTQCAQCAQeEMEkEAgEAoEHRDAJBAKBQOABEUwCgUAgEHggWuP3pOeEQCAQCNsNKtYPSYRJIBAIBAIPiGASCAQCgcADIpgEAoFAIPCACCaBQCAQCDwggkkgEAgEAg+IYBIIBAKBwAMimAQCgUAg8IAIJoFAIBAIPCCCSSAQCAQCD4hgEggEAoHAAyKYBAKBQCDwgAgmgUAgEAg8IIJJIBAIBAIPiGASCAQCgcADIpgEAoFAIPCACCaBQCAQCDwggkkgEAgEAg+IYBIIBAKBwAMimAQCgUAg8IAIJoFAIBAIPCCCSSAQCAQCD4hgEggEAoHAAyKYBAKBQCDwgAgmgUAgEAg8IIJJIBAIBAIPiGASCAQCgcADIpiEbQvDMGAYZqNPg0AgbBJEG30CBMJGQNM0/H4/PB4PRCIRxGIxhEIhRCIRKIra6NMjEAhZCLXGHTa5/SZsKRiGQSgUQiAQAMMw8Pv9oCiKizYpiuKEUyQSQSgUQiAgiRgCYZsR866ZCCZh28AKJE3TnEj6/f4IQWSFM/x7IRAIIBaLiYASCNsHIpiE7UswGEQwGOSiSIqiuLTsagIYT0DD07hEQAmELQcRTML2g2EYTixZoWThI5ixjhcuoOPj46isrIwQ0OjXIRAIm46YX2BS9EPYsrCCGB5Vpkr0cRYWFlBRUQG/3w+/3889JjqFSwSUQNj8EMEkbDmio8pMpkxZAWVfg408/X4/fD4fgNh7oERACYTNBxFMwpYiurBnPYSJjWABcP8VCoUrzik8AmWrcEUiERFQAmGTQASTsCUIbxcBVqZOMwWf12BbVVgYhkEgEIg4VyKgBEL2QwSTsOlhGAYWiwVisRgSiSTrxSaWgAaDwQgBFQqFXBqXCCiBkB0QwSRsatjCnrGxMRQXF0MqlSb0/FSFiO3nTPUY4efBRsvhbTDhEShbiUsgENYXIpiETclq7SKbnXgCGggEuJ8TASUQ1h8imIRNB03TCAQCEYU96Yj0kmE9XjdWJW60gIancImAEgiZgViUEDYNbFTp8/lWVMFulGBuBKx4hrep0DSNt99+G06nE3a7HQ6HA16vl0vrEgiE1CERJmFTwFaWhkKhmCnYZATT7XbDZDJBp9NBLpcnHZVttCCFR9lCoRAMw4CmaXi9Xu4xbARKJrIQCMlDBJOQ9fBx7ElUMKenpzEyMoK8vDwMDQ3B6/VCqVRCr9cnJKDZKDyx9kBZAWWvUfQeKPHDJRDWhggmIWtJxLGHr2AGg0H09vYCANrb2yMmlzidTthsNgwNDcHj8UClUkGn00Gv16cUgW408QTU5/Ot6kZEIBAiIYJJyEpiFfasBh/BtNls6O3tRWVlJYqLizkHHvb5arUaarUaZWVlYBgGLpcLVqsVw8PDcLvdnIDqdDooFIoNLTZKhVgCyjDMCgElhvIEQiREMAlZRbKOPasJF8MwMBqNMJlM2LdvHxQKBa/jqVQqqFSqFQJqNBrhdruhVCrh9XrhdrshFos3raDEE9BwOz8ioAQCEUxCFpGKD2w8wfR4POjq6oJer0dbW1vSqcZ4AtrT04OxsTFuD5RN4bIR6GYkloACIBNZCNseIpiErICNKpMdxRVLMGdnZzE8PIz6+noYDIaYzws3Tk/09VQqFWQyGerq6iCRSOB2u2G1WjEyMgKXywWFQsEVESmVyk0rKNGG8mQiC2G7QgSTsKGkaxRXuGAGg0H09fUhEAigvb0dYrF41eelQngfqFKphFKpRGlpKRiGgdvths1mw+joKCegbAS6lQQUWDmRxefzQSgUQq1WEz9cwpaBCCZhw6BpGmazGTabDWVlZSktqKxgLi4uoru7G+Xl5SgpKVm38V6xzocV0JKSkggBHRsbg9Pp3DICCqw0lLdYLGAYBiKRiPs9mchC2OwQwSSsO+GFPYFAAC6XKy2L58LCAiYmJrBnzx6oVKo0nCk/+FTJxhJQj8cDq9XKCahcLudSuCqVatMLilAojEjjxpvIwj5us/+9hK0PEUzCuhJd2CMQCFJuy/D5fJiYmIBUKkVHR8e69hAmu8hTFAWFQgGFQhEhoDabDePj43C5XJDJZFwEutkENPo9XW0iCwsxlCdkO0QwCesGu1cZXtjD+qAmy8LCAgYGBpCbmwu5XL5pG+7DBZTtEWUFdGJiAk6nkxNQnU4HtVqd9YKy2vnxmchCDOUJ2QYRTELGWa2wJ9nG/1AohIGBAbjdbrS1tWFhYYFL960nmTIuiCWgXq8XVqsVk5OTcDqdkEqlESncbCLR6uNYE1lomobH4yECSsgaiGASMspaPrDJCI7D4UB3dzeKi4tRV1e3aR13EoGiKMjlcsjlchQXFwMAF4GyAurxeDA2NsalcDcy2k7XUG0ioIRsgggmISPwbRdJZA+TYRhMTExgamoKTU1NUKvV3O+28jzMeLACWlRUBAA4efIkJBIJpqam4HA4IJVKI1K46y2g6RSweAIabyIL8cMlZAIimIS0k4hjD0VRvPYw/X4/uru7IZPJ0N7eHtHCwB5nK0eYfBAIBCgqKuIElE3hTk9Pw+FwQCKRcEVEmRbQZA0h+LLaRBYWYihPSDdEMAlpIxkfWD5CZzab0dfXh507dyI/Pz/p42w3ZDLZCgG12WzrIqCZFsxo1jKUdzgcEAqF0Ol0XBRKBJSQKEQwCWkhOgXLd7FcTehomsbg4CAWFxfR0tICmUyW1HEyyWYSaplMhsLCQhQWFgJYasexWq2YmZlBf38/xGIxV0Sk0Wg2taBEfwZZwVQoFMRQnpA0RDAJKcNnwHM84u1hulwudHV1oaCgAK2trWmJVAmRSKXSFQJqs9kwOzuLwcFBiEQiLgJNVEDXO8JcC4ZhIBAIYvrhEgEl8IUIJiFp0uEDG72HyTAMpqamMD4+jsbGRmi1Wt7HIRFmakilUhQUFKCgoADAsoDOzc1FCKhOp4NWq131/c5GwQw/n7UM5VmxJIbyhHCIYBKSItEBz/EIF5xAIICenh6IRCK0t7dzPqSJHoeQHqIF1O/3w2azYX5+HkNDQxAKhREp3OhCrGyCpulVBZ6vgIZHoERAtx9EMAkJwRb2zMzMwOFwoLq6OqVFg03JWq1W9Pb2oqamhksRJsJGCuZ2EWqJRIL8/Hyu8IoV0IWFBU5A2RRuKBTKKjFJxkgBWDmRhfU/Zh9DDOW3F0QwCbxhF4xQKAQAaVkU2Skeg4ODaG5uhlwuT+o4G5mS3a6sJqDz8/OwWq3Iy8vjUrgbGYGmI0UcPZGFCOj2gwgmgRfRhT1CoTAlD1gAcLvd6OrqAkVRaGtrS8t4r0RgGIabU5nKHE7CEuECSlEUdDodAMBkMmF4eBgCgYCLQNdbQDOxpxpLQMMnsgCISOESN6LNDxFMwqrEK+xJdcrIzMwMjEYjGhoacPbs2bTc/SdyPoFAAN3d3fD7/QgEApBIJNx+XKI9idslJZsIDMNALBZDp9MhLy8PwNI1t9lsMJvNMBqNnKiuh4CuRxESmciy9SGCSYjLaoU9fB16ogkGgzh79ixomkZ7ezvEYnFazjURwbTZbOjp6UF1dTVycnIArHTFCTc23wyTQbKNWAIlFouRl5e3poCyKdxEir7WYq2in0yw1kSWmZkZlJaWEj/cTQQRTMIK+Dj2JBNh2u129PT0oKKiAsXFxWn3Gl3rfBiGwcjICBYWFrB//37I5XLub4x2xWGHO09OTsLhcMQd7kyqc5MnloDa7XZYLBaMjIyAoihotVouAk1FQLOhzSXaD3d6ehrFxcXEUH4TQQSTEEF4Yc9q7SKJRJisUM3Pz2Pfvn1QKBTpPGXufFYTLp/Ph66uLqjVarS1ta0p+OGTQaKHOzudTigUCuh0OgQCASKYMUhGoMRiMXJzc5GbmwtgKRths9lgtVoxMjICAFwEqtPpEhLQbBDMaFjx5GMoTwQ0OyCCSeBIxLGH7+Bnr9eLrq4uaLVatLe3ZywttppgskOma2trucU40WNHz6Z0u92w2WxYXFyE3W6HRqPh9uMUCsW2X9jSIVAikSimgNpsNoyOjgLgL6DZKJjR8DGUJxNZNhYimISkHHv4pCLn5uYwNDSEuro6bq8wU8Q6H9aL1uFwoLW1FVKpdMXzkllIKYqCUqmEUqmE2+1GTk4OpFIprFYrjEYj3G43lEol9Ho99Ho95HJ51i/Wm4FYAmq322G1WjE2NgaGYSJSuOH74xuxh7kafLISZCJL9kEEc5uTrGPPahFmKBRCX18f/H4/2traIJFI0nnKMYk+b7ZlJS8vDy0tLRkTLPa4rICWlpZy7SpWqxVDQ0PweDxQqVScgMpksi0voOsR0YlEIuTk5HA3Y6yA2my2CAHV6XSb3kgBWHsiC7DSD5cIaHohgrlNiS7sSfSLFU8wFxcX0d3djbKyMpSWlq7bIhW+p8q2rDQ2NnK9gOsJRVFQqVRQqVQoKysDwzBwOp2wWq0YGBiAz+dbIaBbjY3Y140W0FAoxEWgdrsdXV1dXOEWO+Zro0hHxBtPQImhfOYggrkNSWTAczyiU6AMw2BsbAwzMzPYs2cPVCpVOk+Z9/l0d3cjGAymtWUlVSiKglqthlqtRnl5OWia5gSUjcTVajUnoLFSx5uRjV6YhUIhDAYDDAYDnE4ndu7cyc0EnZiYQCgU4lK46y2gmUgRxxJQIPZEFvZ/REATgwjmNoONKpMZxRVOeITp8/nQ3d0NhUKBjo6ODUkDsSnQuro63pEt+5hU0ofJtJUIBAJoNBpoNBpUVFSApmk4HA7OTzcQCECj0XAL+WYU0GwrsqFpGmKxGAqFAgaDAcByBBotoGwEmsmthPXYU13LUB6IvQeaTe9btkEEc5vAFvZ8/PHHqKurS/lumhUKtgJ1165dXD9dsueXzBeVYRhMTExgcnKSS4EmQjb0UQoEAmi1Wmi1WlRWVoKmaSwuLnJGCsFgMEJA12NPOFU2+ppGE+vzFR6BAksCyl73ycnJjAoom91ZT+IZyodPZDGZTCgqKiITWeJABHMbEF7Y4/P5UvaABcD1Jo6Pj8etQOULK1qJfjFZezuJRIKWlhacOXMm6XNIlkwILuu5yu6/xlvINyKVmAjZtNDy+Xyx48r0ej2A5etus9m4656uGxd2oPVGE+2HOzY2htzcXGIoHwcimFuYaCsutkk6VcF0Op2caXpzc3PKX6BkXIOix4GxE1S2IrEWcraYZXx8nKsGZVuD0mkplyzZlpJN5nzCr3tVVVWEgE5NTXGRP3tzk8hNY7a1uYRDJrLEZ+O/WYSMEK+wJxXBZBgGk5OTmJiYQFNTE7q7u9PyZWErXPmYb0fb27GuQcl626aD9U4/RqcS2XaKubk5fPzxx2AYJmlHnHSSTQtpOgQ8+sYlVuqcLd5aS0CzWTDDIRNZIiGCuQVhI41YhT3JCqbf70dPTw8kEgk6OjrSOlmCb1rT5/Ohs7MTGo2Gs7cLP8ZGkA2LA9tOIZVK0dLSssIRJ9zUXKfTrctYrWzbwwTS/15Fp85ZAbXZbJienuaKt1gHqHABzVbBXOsabfeJLEQwtxB8HHuSEUyz2Yy+vj7s2LEDBQUF6TrdiHNaa4HlY2+XjYv0RhDtiMNOBQk3Nc/0WK1sS8muB+ECyhZvsdXPZ8+ejWgfysbrk8z3Z62JLADw5JNP4vDhwyguLk7buW4URDC3CHx9YBMRTJqmMTQ0BLvdjpaWlow12K+WTuVjb8ceYyPIhirbtYg3Vosd7CwUCiMENB2RTzYKwnoTXv0MIEJAFxYW4PP5EAgEuBTuRhtYpNNMIdxQ/uWXX8YVV1yRjlPccIhgbnIS9YHlu9cXbi3X2tqa0cUvnuiw55Cfn59Re7vtRrSA+v1+2Gw2zM/PY2hoCCKRiBNQjUaTlanDzUi4gCoUCrhcLhgMBthsNvT398Pn80GtVnPXfr0FNFNmCm63OyMTijYCIpibmGQce/hEmFNTUxgbG0NjYyN3d5xJYgnmRtvb8WUzRJhrIZFIkJ+fj/z8fABLe8U2mw2zs7MYGBiAWCzmil3UajWvRZVEmKvDFrmxAsoaWLAOUKyAshaKOp0Ocrk84+eUiZsjt9u97s5fmYII5iaEz4DneKy2XxgIBNDb2wuBQID29vZ1q64MP6dQKISzZ89mnb3ddkIqlaKgoIDbr/b5fFwlqMPhgFQq5aIglUoVc5Hd7DcRmSaWOMVygHI6nbDZbBEexOERaDpvSkKhUEb2sz0eT8bFfr0ggrnJ4DvgOR7xIky2MKGqqgpFRUUJnU+qX1o2TexwONDV1bXuxu2psBUizLWQSqUoLCxEYWEhgKUZp6yJgtPphFQq5SJQlUrFvW+b4f3bKPg4/YQLaHl5eYSJ/+DgILxeb1oFNFMRJk3TWdEbnA62xl+xTUhkwHM8ogWTYRgYjUaYTCbs27cvob0G9ljpuCudnZ2F2WzG7t27oVarUz4eIXPIZDIUFRWhqKgIDMNwAjo+Pg6n0wm5XA6v1wuXywW1Wk2EMwYMwyT8vYk28Y8loOwcVjaFm8i1z1SEuZUggrkJSGbAczzCBdPj8XAjj6L7GvmQjugqEAjAYrFArVajvb193b+wbrcbIpEoZZuz7QpFUZDL5ZDL5SguLuYsE0+fPo3x8XG4XC4oFAouAlUoFERAkb6K1FgCarPZMDQ0xAkoG4GuJaCZiDC32neDCGaWQ9M0ZmdnoVAo0rJnwQrm7OwshoeH0dDQwDmXJHusZGHt7ZRKJaqqqtZVLNmWGbPZDACcOw7bWsE3hUQW/0goioJCoYBEIkFjYyOApZsSq9UKo9EIt9sdEQVtVwHNVEUqK6DsHFZ2is/w8DA8Hg8noLGufSbNFLbKe0wEM0sJL+yZm5tDYWFh2jbOp6amIJfLUy6qSVYwo+3txsfH19XWzuv1orOzEwaDAS0tLdy1Zv1Z2eZ+NirSaDQkVZUE7CKpVCqhVCpRWloasYizAhpdCZruxTUbo5z1Gu8VPcjc5XLBZrPBaDTC4/Fw0b9Op0MwGEz75zxbHY2ShQhmFhJd2CMUCtMiKHa7HUajERqNBnv37k1LsU6ii5HP50NXV1eEvd16Fs6wjkH19fUwGAzcbFDWXi4nJwfAcnP/wsIC15sYq7ViOxT9pJNYizi7D8emEdPdSpGNLS4bNd6LvfbhNy82mw0jIyOw2+3cbMx0pc+3UoUsQAQz64hV2JNq6pNhGIyOjmJubg6VlZURtlWpkOh5xbO3S2ZaSaLwdQxiiW7uZ3sTZ2Zm0N/fz1WG+v3+LbUgrDex9uEcDkdEK0WqzfzZKJjZMN4rWkAnJye5wdIjIyNwuVwRKVylUpnwdXS73Vvq+0EEM0tYrbAnlQjT6/Wiu7ubK6oxm82wWq1pOWe+QreWWGV60gibgs3JyUnaMSi6N9Hj8cBqtWJ+fh6zs7OwWCyksCUNUBQV0UoRbifX19cX4ccabWgej2wUzGxMVTIMA4VCgaKiIi4CdbvdnIl/eAEXXwFlRXerQAQzCwgf8ByrXUQgECQ173F+fh6Dg4MREV065mGy8BE6t9uNzs5OFBQUxBWrTKY1o1Ow6YKtDGUYBgzDQK/Xc/tybHEFu6hvpTvs9WY1P9be3l4EAgFotVouAo1V7ZyN4pSN5xQKhSKuH0VR3P5zSUlJhICOjY3B5XJBLpdz1z6WgLL7pFsFIpgbCF/HnkRFLhQKob+/H16vF21tbSu+BOkSp7UiTL72dplIybJRrdPpXHEN0k34wsLembP7cuFpxUSiIkJswgWUnQhit9u5oc6hUChCQMViMYkwebJWT3UsAWUzLayAymSyiApo1kfWZrPh7rvv5mboPvbYYzjvvPNw9OhRPPzwwxCJRDh06BB++MMfRrxmf38/br31Vu7fRqMR999/P77xjW/gm9/8Jp5//nlIJBLU1NTg2LFj0Ol0GB0dRX19PWprawEABw4cwCOPPJKWa0QEc4NIxAc2kQjT4XCgu7sbJSUlqK+vjxmtZjrCTNTeLp0izjbSd3Z2Ii8vD83NzRk3jo++BtH7ctFRUTAYhFar5RYWYv+XPAKBgLsRqaqqQigU4oY6T05OIhQKQaVSwe/3IxAIZM213oiin7VIVMTZFiKFQhEhoDabDb29vfjSl76E3NxcyOVy3HHHHbj++uvxzDPPwO/3w+1248SJE3j22WfR2dkJqVSK+fn5Fa9RW1uLM2fOAFhaV0pKSnDDDTcAAK644go8+OCDEIlE+Pu//3s8+OCDeOihhwAANTU13PPSCRHMDYCNKvk69giFwogJ57FgGAbj4+OYnp5GU1NTXLecdApmrGOx9nbl5eUoKSnhtSikaw+ToijMzc2l3F+abqKjovBFfXx8POkeUMJKhEIhJ6DA0ndtfn4eNpsNnZ2dYBgm4mZlo651NhT9RJNq1BsuoMXFxfjoo4/wn//5n3juuefwxhtvYHx8HM8//zweeeQRlJSU4Be/+AXuvfdeLuPCmv/H4/jx46ipqUFFRQUA4Morr+R+d+DAATzzzDNJnztfyDdzHUnWsWetCNPv96Orq4vrrVwtrZLuCJONDBmGwcTEBKamprBnz56EphOkI8KkaRperxeTk5MZT8GGk8y5Ry/qwWAwbg9ouuZTbleEQiFXRNTY2BhxrUdHRwFgQ25WsjElm25rPIFAALVajZ07d8Jut6OhoQGnTp3Cd77zHTz88MMYGBjAW2+9hfvuuw8ymQw/+tGP0NbWFvd4Tz31FG677baYv3vsscciUrcjIyPYv38/NBoN/vmf/xkXXXRRWv4mIpjrxFqFPauxmsiZTCb09/dj586da96hrXWsRGGPFQgE0N3dDYlEkpS9Xarn5PF40NnZCYFAgL1792ZN2o0vyfaAEvgRvocZfa2DwSBsNhssFgt3sxIuoJkyrMhGwczEOXk8HkgkEnz00Uc4evQofvWrX+Gee+7BkSNHEAwGYbVa8d577+HUqVO45ZZbYDQaY66Nfr8fzz33HB588MEVv3vggQcgEolw++23AwCKioowPj6OnJwcfPjhh/jUpz6Fnp4eaDSalP8eIpgZJrqwJ5kPZCxBoWkaAwMDcDqdaGlp4d2flm7BdDgcGBwcRE1NDTfNIlFSScnOzc1haGgIDQ0NGBwcTPj56bge6S5YitUDGj1eK9Z0EEJsViv6EYlEyM3N5arI2ZsVk8mE4eFhCIXCiGHa6RLQbBTMUCiU9nNyuVxcm0pHRwcA4KabbsKRI0dQWlqKG2+8ERRFob29HQKBACaTifvch/PSSy+hubmZa+tiefzxx/HCCy/g+PHj3HsslUq5NG9LSwtqamowMDCA1tbWlP8eIpgZJJkBz7GI7sN0uVzo6upCQUEBamtr0xatJgLDMLBarfB4PGhpaUmpdDyZtCZN0+jv74fH4+FSsMkcJ1WxWw+xih6vxVYmstNB2N44mqYTrgi1ewJQy0QQxHgOwzBw+0NQSIQxj2l1ByATCyAXZ7dtYCLXJPpmxe/3w2azYX5+HkNDQ1w6XafTpZwuz7YbnXRNHgrH4/GgsrISZWVl6O/vR21tLY4fP46GhgbU1NTg9ddfx8GDBzEwMAC/3x9haBLOk08+uSId+/LLL+Ohhx7Cn//854j1Z2FhAQaDAUKhEEajEYODg6iurk7L30MEM0Owe5WpjOJiYfcwGYbB1NQUxsfH0dTUlFSKIR2C6fP50NnZCQCoqqpKuc8q0bYStrezsLAQdXV1EfMXt4NNXfR0ENbc3Ofz4dSpU5y13FrOOG8NWfDIW2O4siEPd3aUrvj96wNm/PepKXx6fxGuaYxM949bPPjX40bkqiS47+odWbf4h5NKgY1EIkF+fj633eH3+znDisHBQYjF4i2TLs9EhMm2lRw9ehS33347/H4/qqurcezYMSiVStx1111oamqCRCLB448/DoqiMD09jbvvvhsvvvgid4zXXnsNv/zlLyOO/bWvfQ0+nw9XXHEFgOX2kTfffBPf+973IBKJIBQK8cgjj6StB5sIZpphC3s+/vhjNDU1peUDKBAIuGOKRCK0t7cnXZyQqmCG29u5XK6kjxNOIkLHpmBj9XZuhGBm6jUnbR68NWTF5bU5yFPH79sM742bmZlBa2sr1wPa398Pn88HjUazVGAkVeHPw3YcqNKj3CCHyx9EiGHg8sUuKFv0BBEIMbC5V1Zos/qYxTrJkc4WDolEEuH4FJ0ul0gkEenyzSSgmYgw2ek0+/btwwcffLDi90888cSKnxUXF3NiCQAKhYKbKhTO0NBQzNf89Kc/jU9/+tMpnHV8iGCmkfDCnsXFxbQd1+FwwGw2o6mpKel9QpZkF/hY9nYejydt7SBrHYemafT19cHn88Xt7Uz2b8uGxnaGYeDwhaCRLX0ln3h/Cn/qM8HpC+KLF5THfPxj707A7gniKxdXQHYuLbpaD+hv3hnFyyN+nBqU4VufqMSlO3SoLVChRBs7Cr1uTwH2lKhRbljpVFSml+P71+6CVCTY8Gu3Fpl8f6PT5eww7ampqYj9Zp1Ol/XDtDNxnVjB3CoQwUwDsdpF2H3HVO7YaJrG8PAwzGYzNBpNymIJJLdvEs/eTiAQrNkfyvecVhO68BRsLDMGvsfJBIm85rTdi5+eGMUFNXpctzuyeOE3H83g6Q+n8YXzy3GoKR9X1efB4Q3h4h2xU0mL3iD+2D2PEA1cu7sAu/JjL0oTNh9++5EJVzfk4eaL8+GVTuCicgUWFxe5HtDQuaIWnU4X8XkVCSjU5MVf7LTyzVGNvJ43RDKZDEVFRYb26xAAACAASURBVCgqKgKwvN88OTkJh8MBuVwOvV7PbbFks4CmA2KNR4ggXmGPUChMqa/J7Xajq6sLubm5aG5uzohrBR9Ws7dLl0Cttoc5OzvLvT7rJxqPjdrDDNGxX/ODcRs8fhoX1uhBURS6px34aMIOuzewQjCt7gBCNAObZ+kGZH+ZFvvL4v+9WrkYX7ukEoveIGpyIxckf5CGWLj0WfzVuxP4/zrn8MpZE178Shu+d01txGOj2yoEAkFEWwWbUpxd9IIChQLN5rP120hhit5vZp1w/H4/3n//fSgUilW9WDc7JMIkAFjbBzaVvcLp6WmMjIxwIhUKhdZ1wDLAz94uXRW3sVKyrB+uz+dDW1sbr97KTAomwzCYXfShQCONqCj9XbcFL5214juHdGivXHYWsrkDeODlITAMUKJrQHWuAhfWGODyh9BQuNLU4a7zynBwZw52xIkUY3FZ7cqKwg/GbPjha8O4ujEfd51Xhstqc/H6gBlFWmnMxThWWwVb1ML2gEKmwb++Z4NYJML/valh00SWLNnSwhHuhDM1NYXW1tYV00DYcVobMfUmE6/lcrkSMjHJdohgJkF0CjbWB42NMBMhGAyit7cXANDR0cEV9qSzd5IPfO3t0mWaHi10LpcLnZ2dKC4uXjUFu9Zx0skzZ2bx65OT+NSeQtx1fhn38wmbH94Qg0mbF+1hj1fJROio1MPtD6LwXFSmkAhxw97YaXWJSIC6GELKB4ZhsOD0g2YYmJ1++EIMpmxeAMBFOwx44cttkIgEEAnWvo5isZirCp1b9AGhAByLdlChAPwBD/rP9qIozxA3IjrRb8JTH87gopwQ4nu2rOTtYQveGrbg9rYSlOnTO90l21Kf7Gc0lpk5O9DZaDRy0Vn41JtM/h2Z+O6QlOw2J9aA51gkKnKsYXFlZSWKi4sjfrdeX/ZE7e3S5QEbLrwzMzNcdL1WCjbW+WRKMOlzaddQ1PH/qqMA5xWLcXVUilUkoHDvlTUZORenLwiJUACJaClqeq3PhJ+/OYYGTRD335qHMp0MlTnLi5RalvjXfHbRi2/+oQ8SIYWjtzThPz5fBApAKODjplM4nc4VC/rZOSfmnT5MSON/LmYXffjpGyPYW6LBrS1Ln/XuGQeMJjdGzZ4tL5jxIt7ogc6sgFqtVgwNDcHj8US0DKVzbFymrhFJyW5TEvWB5RthMgwDo9EIk8mEffv2bdjdWDL2dumMMGmaRk9PDwKBAO8UbKzjZEowb24uwvnVehRFVZSqpEI05kshFq5Pym/S5sHf/f4scpQS/PTmRggFFAIhGjTDIEQvCfXukuQswEbNbkzbfThQpYNIsBSRSkUCCM/9FwAgit0DOjw8jAW7G/uUClQ066D2m+K+zoTVA6PJDX+Q5gTz5v1FaKvQobEo9tCAVMhGweQ7lIAV0LKysphj4/j23K5FJnowgaWs2Xr5Oq8HRDB5kIwPLJ+RXF6vF11dXdDpdGhra1vXfZbwRYQdO5WovV26Ikyv14v5+Xns3LkTZWVlSS9uyQjm3NwcfD4fDAZD3JTXojeIR98Zx75Sbdqjn7Uwu/z49ftTuKBKh7ZKPXwBGkGagcsXBM0wEILCNY352F2swdRgV0qv9b0/DmDRE8R9V+9AW4UOD9/SCEG4WEYRnlL0SPR49MMBFKuBr7aIMD6+VNTC9oCGD3feX6bFPZdWoVS3vMAblBIYlCsX1kCIxslRGwrVEuzITy5lnS17mCzJGinEahliBbSvrw9+vz/puauZ6MHcihDBXAW+A55jEW1nF83s7CyGh4dRX1+fNhcKvoQLCxvd7t+/P+HoNh17q9PT0xgeHoZWq0V5+cp+w0RIRDDDi5pUKhWGhobg9Xq5BcdgMHAL/KlRG57rnMM7Rhsu3ZWT9GuuRiBE48irwwjRDP7hqh2cSL09bMUfzsyiZ9qBtko9avKU+L+fboBSIuSiWoqiUG6QY47HHmU0Ll8QndMO7C3RoKVMi7OzTpTpl4RMKeW/PARpBgwD0BCivLwcJpMJzc3NcDgcsFgsEcOd9Xo9Wkv5zQEdWnDjyQ+moVeI8U/X7krqZirbRmmlS8AFAgE3iaWiomLF3NVAIMDdsOh0ulUFNBM3FVvRdYsIZhwYhkEgEEAoFErK2i5eSjYYDKKvr49LPW5EukIgEMDj8aC3txdarTbp6DYVsQgXrL1798JoNCZ1nGTOhy0qKikpQXFxMYLB4Iom/56eHm7Qc5VKi6sbctFSrlvz2HwI0Qx+9d4kRALgcx2lEFAUzK4A3h+1gQGw4PShVLcUyZ5XpcOhxnxcULNcgZtslOvwBnFmchHNZRpODI+9O4EXuhdweE8B7rm0KqnjMgwDnVyEBw/XIk8l5bIX4XNA2eHO7Git8fFxfDwfwIhbgjs7SlBVlBszwtHJRdhfqkFVbvIVo9mYks2EgEfPXWUNVFgnomAwGCGg4WtPukd7hZNN1z5ViGDGgG9hz2rEir4WFxfR3d2NsrIylJaWbtgHKRgM4vTp06irq4trdsyHZCNMp9OJrq4ulJaWorS0FF6vNyPVtrFgI/umpiZotdqIm5pYg57tdjssFgs+YbCDci3CaHRE9ChSFIUQTaN3xoEyvTyiwIZmGHz/j4OYd/pw5HAd144xYfXgmdMzAIAr6/NQpJWhUCPFt66oBs0gwnknTy3Ft9JUPPTouxN4sXseh/cU4KuXVAIAavKUUEgsqMlLfu/8HaMVPzpuRGu5FvddvZPb549GKBTCYDBwGZVjT3dhcN6J2qF52GYnIBAIuHSiRqPBhxOLOPrGKC7dlYNPxGih4ct2Ecxo2J5atn86fHD55ORkRMQvEolIhMkDIphhJDvgORbhESbDMBgbG8PMzEzCw5VZ2P3CVM6Jtbfz+XxobW1NeT5cMnuY09PTGB0djTCPT9de6GqCGT7dJF5fKbAUhRnNbjQVqVcs8IFAAGMzJnSNzEAWHIREIoFMJsO7E2483W9BU7EGP7qxnjuWN0Dj9KQdwRCDabuPE8wyvRw37y+CWBhpBHDRjsh0byIwDINgDAOFNwbNsHuWjBLqC5T4nyERaguWqxY/2ZiPqxvyEhKUM5N2/KnPhNvbSlCklUEgoEBh7UjCF6Txzy8NggHwnat34O4LytE55cD1ewuhkYm4ySBzc3MYHBxElwVweUJYsLtSEr1s28NMp7dtIkQPLg+P+M1mM3w+HwYHB7kINNVh2j6fL6VCpGyECOY5UhnwHAvWMN3n86GrqwsqlQodHR1Jf3HZaC7Z54fb26Xjy8CeUyJ7hr29vaBpeoV5fCrVthNWD8xOP/aWaiIEc97hw3/8zygO7sxFc4kCnZ2dyM/P56abDC+48ELXLD61pwAFquVzeeDlQZyeXMTXLqlc4cYDgRBf/P0oPIEQHvnMbtTlSTE5OQlB0IuAPwjKu4ipqSmu5F8hEeIH19fB5gmgLkykhAIqopczHfzLn4z4U48PPy5d5Kpk7Z4A/uU1I2iGQW2+Clc15OOqhpVDxlf7rJucfvzPsAUXVOs5E/gn3p/GB+M2FGtl+GxbCc6r0uPnn9mNHOXSDUE8cbN7AhhcWDLsX/QGsa9Ui32ly61D7GSQnNw8/HnQjJZ8CrsrXJAEnTh16hRkMhm34CfiipNtEWa27KmG3xDq9XqYTCYYDAaubYhhGC5CTWbNcLlcW6oHEyCCyRX2GI1GruAjHV8uoVDIFTvU1tamlPoEUiuwiba3s9vtaeuf5HMcNgVbVlYW0wgh2b3QYIjG15/uhCcQwg9vaIQ87DhvDpjwfOcsuies+FpjCA0NDdydNQA89vYoXj27AF8giK9fUsH9vFArg2jagTzVyr1llz8EuzcImmYws+jDvjItcnJycD7D4MZLK4GgD3abLaKAKFevx87S9Hymwumfc6Jr2oFrGvOhkAgxYfUgQDOYd/q5x6hlIlxVnwubJxDTQJ0P/3VqCn/4eBaD8y5884ql1PDn2otRrJXistrliLgwLFKOFqgZuxdnJhdx0Q4D7rtqBxgA+atMYOmaXsTP3hyDSirEsc/theDc++r1emGxWDA6OooJsxNGlwgHa/NRVZS7alN/tglmtkW8wNI5iUQi5OTkICdn6X1lbRNZJyKKoiIEdK09T3a011ZiWwtmeGGP3+9HIBBIyxcrFAphenoaLpcLHR0dCZV3x2Otqtt45xHL3i5TDj2xmJqawtjYGHbv3g21OnaPXbIpWaGAQmWOAuMWN3JUEni8y+/dRTsNODUgQ502xE1XCefTzSXwhxgcaopso7nnYCW+dGF5zKHIOrkYD1xXi3mHD1c1RE6FV8vEAMRQn2s6Z0v+LRYLent7uQIig8GQ8N06wzDonXEiRyXhhOnBV4cxYnJDJhbg2qYCfP/QLrzyzkcRVbwCisLXkyzkcftDMDn9OL9aj7OzTlwUZgK/t1SLvaX8TSV+9uYY3huxwuYJ4LbWkjUfX2FQoLFIhZpcBWdDSFEU5HI5SkpKUFJSguMnRvDKyDxCQjsOumzwer1xexKJYK5NrD7MWLaJrO+w0Whc4TscLaBbzbQA2MaCGV3YIxKJErayiwUbTWm1WiiVyrSIJcCvrzOc1ezt0mW1t9pxgsEgzp49C4Zh1pzfmWyESVEU/vWmJtDMkngOm5eE1+fzYby/G19sy0FNTU3MxbK5XIfmcl1E2xB7zFhiyfLJqEHK8c49vOQ/vIDIarVyd+vs4h5ucj5p82Da5kNrhZYTizOTi7jv+X6opSI8edd+CCgK1zTm4c+DFuwpXkq/GpQS7NAJI3xu+dI1tQihgEJDmGnAfc/1YWDehe9+ciceuW13Qsdjv1PjFg9ylGJcvMMAi8uP/TxFNkQz+GRjPlrL4z/+kp0G2DwBXL2/BLvylUvj0c5VOLM9iWxBS7wipI0iGwWTTx+mWCxGXl4e8vKWbhZZATWZTBgeHoZQKOQEVC6XcxGmzWbD3Xffje7ublAUhcceewznnXcejh49iocffhgikQiHDh3CD3/4w4jX6+/vx6233sr922g04v7778c3vvENWCwW3HrrrRgdHUVlZSV+85vfQK/Xg2EY3HPPPXjxxRehUCjwq1/9Cs3NzWm7TttOMOMV9iTj/Rp9XNZWrqmpCcFgEDMzM+k6bd4ix8feLp2CGUss+HrRrnUcPlAUBSG1/P+dTieGh4fTkgZPJ9EFRGMmJ97sm0GjaxbBwUGIxWIYDAb875fn4QjQ+P6hXTivaimFrFOIIREKlgzUzx3vluZi3NJcHOfV+DNl8+Lbz/cDAI7dsRe551LRUpEQFEUl5WDEMAz6LSH813s9qDDI8dObG3Flfd7aTzzHv50YwekJO/76ooqV+8hYin4/Grfjkh0GbqwZRVFLlbWzflgYCa7dnwePa6mp32QywWq1IicnJ20FLamwUUU/q5GMiEcLKFu0NTY2hi984QuQy+VQqVS47bbbcPjwYTzzzDPw+/1wu904ceIEnn32WXR2dkIqlWJ+fn7F8Wtra7kpTaFQCCUlJbjhhhsAAEeOHMEnPvEJ3HvvvThy5AiOHDmChx56CC+99BIGBwcxODiIkydP4stf/jJOnjyZ4tVZZlsJ5mqFPUKhMOnZjn6/H93d3ZDJZJytXLr2CVn4iBxfe7tMTRlhGAZTU1MYHx9fNQUbi1RTxAzDwGKxwOPxoK2tbd2q89hrMGXzokAjjTA4HzG78be/68X+Mi2++8mdEc87+uY43h2x4c6OUvzVhfXc4OFiBY0Bsw+O2TFMil0wGAyoNMjxmy80QyRMvRgNWJpo8srZBXz+QBl0chGKzp13eEvMP123C3ZPkBPQtTA5/fj5m2Noq9Di0hoNxEJAQAEK8dqL8JkJO/77g2n8RUcpmorVaChSYdTsQVVO7P2vnhkHnuuah1omwiU7c7hr4vaH8Kv3JhEIMdhdosGu/KW9Nr/fj/z8fDAME1HQEh7hr6fLTTZGmKFQKOWecLZoKz8/Hx9++CGeeuop/O53v8Pbb7+N2dlZ/OEPf8Bjjz2GkpIS/OIXv8C9997LZeDy81cWo4Vz/Phx1NTUoKJiqd7g2WefxRtvvAEAuPPOO3Hw4EE89NBDePbZZ/EXf/EXoCgKBw4cgM1mw8zMDDefNFW2hWDycewRCoXwer0JH9tsNqOvrw87d+6MeNNTjVijWWsPMxF7u3QKJit07KQViqLWTMHGOk4sJq0edE7ZcemuPMglKxc0hmHgC9IQMCF0dXWBpmlUV1evKpY0zeCtITPy1VLUJ+hbajS58daQGYeaCiKE5MSYF7979QyuqMvD34f1TE5YvVj0BtE17VhxrEt25GDa7kNbxVLakR08fPRzRaBpmhs8zBYQqVQqrpqRb5qfYRj85MQIxq1e/OM1O7m2lsfencCZyUWU6uS480Ap/v2ze1Y8VywUrCqWI2Y3TE4/Wsu1oCgKH03YcWLAhP45Jw5Wq7FDL8axO+qglK4tRK/1mXBq3IZSnQxNxWp8trUEn43a6wzSDJ7+cBoqiRCX1+XiUGM+avIizQzkYgFu2l8Es9OPyrAiJ4ZhIBQKodFouAifLWgxm81cOjG8BzSTgpYtVbLhZELEZTIZKisrYbFY0NDQgA8++ADf/e53cfToUQwMDOCtt97CfffdB5lMhh/96Edoa4s/3+app57Cbbfdxv17bm6OE8GioiIuQp2amkJZ2XIFemlpKaampohg8iXegOdoEhU4tqdxcXERLS0tKxbpdI/kireHGW7e3tzczGuCQToFE1hOwVZUVKCkZO2ijmhomsHr4wFMiCdxS+tyCvcHLw3g9KQNdk8At7WtbMN48OUB/OnsPD63i8E1bbXweDycgL9rNOPIK4O4s6MMNzYvn9NHE3b84/NnIRUL8cJXD0AYtQfJMAyOvDqMSZsXD1xXC51iuV/zpydGcHLUBk+Axl9fVMFdA/rc80NR1/TCGj2+f2hXzArV6/YU4Lo9K9ONwNL7w3q0slMrWIu58AIidoFnb04mHDSOvzqEW1uKUZWjQCDE4JWzJvhDNAYXXGg951R013lleK3PhCvqk0tZB0I0/vZ3vfAFGRw5XIvdJRqcX63Hba0l2FOi5vYww69dNBNWD548NY1rmvJxR3sJinWyVc0Jxsxu/O70DAQUhYt35uCLF660UaQoCoea8mFy+iEWLn/Pl26sGLxjtKC2QIUcpWRFQUt4D+jAwAAkEgl3fdVqdVpTqNkaYab7nFwuFyQSCT766CMcPXoUHR0duOeee3DkyBEEg0FYrVa89957OHXqFG655RYYjcaY19nv9+O5557Dgw8+uOZrxspUpfO92/KCyUZBa/VWJiKYLpcLXV1dKCgoQGtra1xXk3RGmLFEzufzobOzM2F7u3QJJnsz0t3dnbQhAwCMWdz440gAsulRXLAjB6XnrN8uqDHA5PJhd8nK4g+GYdA/ZYHH54e+aCcKCgowPj7O/f70uB2zdh/eG7FGCGaZXo6ic+OvRILlzwaLL0jjjUEzAiEGI2Y39iuWX/vaPQXwBEK4eEek9+9lFTIcaqtFiS7qpomicEFN6j7B7P5cdAHR8LQJb3WPoEKzVGzx8ogfnZZZAMC9V+6ARCTA9w/twrTdG9Hv2FqhQ2tFYjZ/p8Zs+M+Tk/irC8rRWKzGrnwVxq0erj1EJRXhr86JmNvtXvN4r/Qu4IXuOVg8ARw5XIfb21a/0SrTy3D9ngKoJCLo5PGXrd9+OI3ffzyHz7YW4/C52aM0TeONYTueOrOA86v1+NtPVK94Xng6EQCXIp+cnITT6Uy6BzQWbAtHNpEJ83WPx4OioiKUlpaio6MDAHDTTTfhyJEjKC0txY033shlpAQCAUwmE7cfGs5LL72E5uZmFBQs32AWFBRwqdaZmRnufSstLcXExAT3uMnJyRXjElMhu961DMGnqISPwDEMg+npaYyNja05rzHTgrmwsICBgYGkilvSIZjBYBA9PT0IhUK8x4HFo1Qvx4EiEcrLCiPGZ93eUYbbO1ZGlsFgEN3d3fhyswpC/S60VCyLEvs+/8WBclTmKtBaoY94bp5Kgl9/vgWic8Us0Z8LmViIfz7XOrK3NNIJ6fLaXFweJwqqjLPflg5eHzDB5g7iU3sLIKCopWpEvR4P/m4ELh+DB66tgkYZwkVFU6AoAeqlS4UXer0ebRVaUFRi4kgzDH770Qw0chE+ec7o4I/d8/hw3I5XDAvYXaLBkU/VxX1+vDaOuUUfXuiew6W7cnF1Yx6s7gCuaVx97woA3h+14qHXjLhpfyFual49teanGTAAAqHIrMGOXDnK9DLsDRt9xjAMzK4AcpTiFefLpsiLiorAMAyXIh8dHYXL5UppsHM2RpiZOCeXy4WysjKUlZWhv78ftbW1OH78OBoaGlBTU4PXX38dBw8exMDAAPx+f9x17Mknn4xIxwLA9ddfj8cffxz33nsvHn/8cRw+fJj7+cMPP4zPfOYzOHnyJLRabdrSscA2EUw+rLVHGAgE0NPTA6FQyGuPLt0pWfb8aJrGwMAAnE5nzP5CPqR6bqwnbmVlJVwuV8p3pmKhADfvkuD883es+Vg2/VsZY9B2+N+lkolwTVSPZTBE445jH2Ju0Ydf/2ULF8lG05ZA9JXOdM9bwxa82D2Pr1xcwRms2z0BPPDyEBgGqM1XorF4ad+VApCrlMAbCMGgkiEvV4G6vFF87po2+Hw+WCwWTE5OwuFwQKFQcIu7QhHbxJxhloRGQFHom3XiP94eB0VROK9SD51CjC+cX4YyvYyXwLHHOjW2tC/J3gT9/sws/uuDKQwvuPGDw3WcEUIsAiEaVncA+WopxixeeAIhziUoFnOLPvzLn4bRVKzBj2+sj4j2/UEa04t+/M0llajJW+4LfKF7Hr8/M4sb9xXGrMZloSgKCoUCCoUCJSUlKwY7h/eAGgyGNb+T2Vglmwnzda/XC4VCgaNHj+L222+H3+9HdXU1jh07BqVSibvuugtNTU2QSCR4/PHHQVEUpqencffdd+PFF18EsJSteO211/DLX/4y4tj33nsvbrnlFjz66KMoLy/Hb3/7WwDANddcgxdffBE7duyAQqHAsWPH0vo3bQvB5NPnt1qfI1tQU11dzftuJV3mAOHH83g8eP/991FQUIDa2tqkv3SJ9nSyxGpZGR0dTeockmFychLj4+Orpn9Xu+ZBmsG0zYsgzcDi8scVzHSw6A3i5KgV7RU6rtgmnNf7TWAY4LLa5SrPY+9OoGfagV35SvzleUuRtVomwhV1ubC4AqjKXY5iKYrCzz/TBH+QhkIijPi7pVJpRHTEDnk2Go3weDwrFneaYfD13/RgZtGHn93ahKpcBS6qMUCnEENzLv1ZppfjC+fzH7/WvRDA428MQK8Q47//cj+Apb91yOTC9XH2bsN58JUhnBq349tX7cDhPQWoMMgjPHCBpfT5lM2Lqhw5hk0u9M06YXYF8PkDpSvO5ZmhKVTmWPHTW5q4nwdpBvQ5D95Rsxv/750JXkbvsQY7sz2gZ8+ejegBjZ4KAmyfoh82Et+3bx8++OCDFb9/4oknVvysuLiYE0sAUCgUMJvNKx6Xk5OD48ePr/g5RVH42c9+luKZx2dbCCYfYhkX0DQNo9EIi8XCu6AmUzgcDszPz2P//v3c9IFkEQgECbfQsGlQkUiUcgoWAN4zWvCnvgV88cIKFGhWVrWGaAYmpw/5ailomkZvb2+ECUKstB/r3xsPmViIxz/fDJsngD0xmuinbF7o5KKE5kCOWX342alF3CW3oq1yOf37izdH8VzXPD7ZmIfvXB3ZUjJt9+KfXh4EAOzMV3JFQV+7pBKvnl2IiOIEFIV7r4wdeYsEFEQxqofDGbV4oJNLuMkwDMNgZNaCH7w+gp3qcZxXJIBCpcHwggM0I4DZ6UehRop/PLSL9zVgCYRonBqzoVxFIVchhEIi5PokAaC2QIUf39gQ87m+II3OqUU0FqmhkAgRODdfMxhiIBEJ0F658jP/kxMjeNtoxV9fVI7La3PxjcuqURFVYOXwBpEnp9BUrEJbZeR+8uE9BWiv0KFIK8UbA2YMm1zQykUJT0YJ32Nm51JGTwVhG/p1Ol1WpmQzUfTj8XiINd5WJTrq8ng86OrqgsFgQGtr64Z9wFl7O4fDgbKyspTFEkg8JRuegk1mA93mDuC9EQvOrzZAcy7a+vmbI+idcaDCIMfnDqyMXB56dQCv9MzjqxeVoSw0wy34FEWhf86Jrz/diQOVenz/+vqI5zEMA5cviP96fwJ7SrQ4UB25SOappXi+cxZ2dwAHa5cLDM5MLuJbfziLfLUET3x+/4rzGZx34cFXh3BtUwFu3Lec6n2lz4L3p30QnJyKEMzdJRqcGFh24gknVylBS5kONMNETCtpLtOiuYy/5dxadE878L9/3wutTISnv9AMwbnCt15zEB/O+DDhlOKrh/bDbrfj3gsYzJgX4Znux7CHX3+iyxdEkGa4CPq3H83gP96ZwL4iBb60R4Lf3t0EvjmQJ96fxNMfzuDyulx864oafOfqnZh3+FCqi98ipJAIIaAAuWhpmPbldZFCZ3UH8He/74Xb5cEjdzRAq1xZlCUWUhicd+H8aj3kEiF25Ka+wIeP1QqfA8r64Hq9XgQCATAMs+49oPHIRNEPscbbpPBJXYYX6bBm5dGG3etNuGNObm4uXK74eziJwNeKLjwFu3fv3qQ//D95fRgv9czhxn1F+NZVS5HLly6sxJ/6FnB5few9MZ+fRjAUwvDoGK6+en/EKLJJqwdufxA9M5H9jWwa/MSACf/xP2PIU0nw4t+cH/GYt4bM+PXJCegU4gjBZLsQhILYn5WTo1b0TDsQopkIwbyuKRdTJis+H9XmcE1jftz9PolIEDEGLBlcviAUEmHEZztAM/i3143IVUlwR3sppCIBBFgSlvC/6uIdORhp9aC5XMM5EF0SNsIs3O5MJBIt85p5DgAAIABJREFU9X6qtFAoVZw4egIh/OUTH8MboPHvn92NQo0MlTlyyEQC1OTKANBr2vT5gktzROsLVSjVySAWUly0LRUJ4g7KNprcePSdCVy/Ox+3tRYjRxm7XzREL6VbQzSDWLeHNMPg//xxAIveIO67eifnrJRuol2eenp6oFKpIizl1qsHNB6ZSsmSCHOLwopIV1cXV/kZb2Zipom1V7iwsJC2qlu+rkE9PT0Qi8VrpmDXMrduKdfh5KgV+8Kipwt25OCCOPMfaZrGjVU0WrVKXHneXkij9oAu3ZUL2Q2NqDCs/DIyDIO2Ch06qvQ4ULWypaO9Qo9LduagJap6dneJBr++c1+E20041+0uQJBmViyqJVopvrBXhcaS1GaLxmPc4oFSKowQhdfOLuCBV4ZweE8B/tdlyy0SY4s0ft89B4paOt+d+Uo8/YVmyMWRwqqRifA3Bytjvl603ZnP58PknAl3P9kLfzCE712oRVVRDmQqDQIhBiEG3BzO86sN+ONX2uBYXMTs7GzEcRmGwaTNi0KNlLPbe/Sdcfz+zCwONeXjf11Wjctqc3lZ8Z0YMOGtYQtohsGDh+NX684t+vCViyvgnRmGRrbyu0wBKNbJELR4uFYVm3tpBNm+Uk1StoB8MRgM3D683++H1WrF7OxsRA8o+5j1KBDKhGCy++VbCSKY57Db7XC5XFzaMV0f0kQnJcSzt0tn1e1ax7Lb7ejp6UFVVdWaRU5sVGd2+aGVi2MuMtfuKcS1e1Z3HwLAle93dnaisLAQ1zY1xLx2AgGFC2pWii17LgUaGR7+zN6Yr5GjkuBHN8U2Ew9Pj0ajlYvx+QPpnWG5FgPzLnz16W4oxAI888UW7trOOfygGWDG7ot4fIVagOt25yBPJYHmnPDHKjhai9MTdnz7uX4casrH1y6pRF5+AUTiaUDIoLS8Agg4MTMxhq/WByGRKyH02OCTCyCVSrlRXNHv2x975vHTE6O4sEaP712zlGUo0sogElBcFW08gQrRDL77Qj9MTj9++Kl6XLe7ACGawWWr7DVO2bz4xz8OgKKArzSsPJ8Rsxv//j/juKo+FxdeuYOzNPzl2+P4YMyGvzyvjFdFcDJEF/1IJBIUFBRwfYZsD+jExAScTifkcvmaVc7pOKd0H5ekZLcgDMNgZGQE8/PzXNl4umCFie/ewGr2dushmAzDYHx8HNPT07xTsBRF4a1BE+5/cQAdVXr84FONqz7eFwhBLBRAEJX6pCgK8/PzGBoa4uZ2JkOilckMw6RUzRwrvR2ilyz7FFEFOW8MmPFC9zy+frByhfuP2eXH3/ymB0UaKf7lxnoIKAoSIQUBBUijIsTPtBajvlAVMZQaAMRCCt+8PH6rRjz+7cQIBuZcuP/aXchVSTBscsPlC3GWfhqZCI/esRdBmjk3XkyH0tJSNIVVh7IORBqNZqny9py/7oLTh70lGi49G56mvWFvIa5uyFt1OgywlLrtPpcOn3P4UFugwl9dWBHxGLc/hG/+4SwoCviXG+qhk4tQqJVCJhJAKnSuEIPTE3Z8OGEHRQEHdy0Lb32BCsMLblTol0T8j91zODvrRL5agk825HNDtFNhrWguXg/oyMgIXC5XxBizdBUiZkKEiWBuUuJ9GLxeL7q6uqDRaNDe3o733nsvra/L7ouuJZh87O2SmYcZj1iCyUa2Uqk0oSpYgUAAjz8ImllatFajZ3oR9/ymC3tLNfhxWJRH0zS8Xi8mJiaW/CQFInj8oZj+saux2t4swzAInKu4ZAkEAujs7ITb7YZUKo2bBnN4gzFTtUaTG0OzNhRGveY3nulBz4wTP725EU3Fy361j707gd5ZJ+oKlbg7qkVjwurBlM2LmUUfvIElsa3MUeDpu5ohFQsiTN1FAgotUaOv+Ir+0IILz5yewS3NxajOVSBEM3i+aw7+II2+OScuVBlw/e4C5CglaChcTqfF8pUNrw4VaPJhdvqQr1oy9zBbLLjn5QUEGAG+e0U5rqwrwt4SDfLVkceJJ5asD+6C049vX7UDRw7XweELRlTdhuP0BTFj94Gilt6vfLUUD59rITl16hQoioIvSOMnJ4yQioS4o70EFJbGvLEEQjRqC5T4ZGMjxEIBAiEavz8zi1GLB0qJEGqZGDfsXTtTshaJpD9X6wEdHBzkBpWzApqucYLpwO/3p2zonm1sC8GMxfz8PAYHB1FXV8dNGAfSm5rgExWyor2WvV2yvZN8zstut6O7u5uXcXusYx3cacDOQm2ES08szC4/gqGlyIPF6/Wis7MTAoEAe/fuBQ0BPvvoKbj8ITx+Z0vMNOm0zYu3h824vD4PesXyF5KiKLwx4sSb86O487zyiBTfP73Yj1d75/Hjm5rQUWXgKn+rq6shV2nwg5cHoZhcxHWVDq5/zGAw4JURP469P4O7ziuNSMkyDIOvPNUFtz+EL+6RYU9Ylndu0Q+aYWBxR7bufP3SSrzSu4Brm1b2Ie4t0eB7n9yJHJU4IjJdzY81mujPrcMbxCtnF9BRqeMKaB59ZwKv9Zng9tO4/9pdEAoo/OD6WoyaPeg417ohEQkiBlHHwnsuUyAUUAiEaHzlqW74gjR+dGM9SouKIJFIUFvsw9C8E5TPgdOnZyESiRA4d1MS7c86anbjzSELrmnMR65KArc/hJd7FxBilm5Mdq+yR+wNhDAw78I/XFUDlVTE2fVFM7vow7tGGyiKwu1tJbhhX+R2w39/MI3nOmdxw95C3NFeCrFQgLsvKEfX1OKSiUNV6hXqQGr7hdE9oLEGlWs0Gk5A+dZhpLNnPJxsa59JlW0nmKFQCP39/fB6vWhra4u4A2IjwnT5PK5lj8fa20WLdiwykZJlGAZjY2OYnZ3F/v37k6poY6O6cAeVeFy0Iwf/dssezoWFnfRSV1cHo9G4FAUyDBzepXYFTyD2tfvhq4P486AJYxY3/u6K5R5Ht5/Gox/ZQAkc2FcW6Zc6YnIjSDOYsnkxOTmJiYkJ7N27F3K5HB+MmvHGkBUUgC9f1gatXASXywWLxYK+8Vn4/QH0jc1hoUYGnU4HsXjJSm13iQY904soUEYuCj//TBOm7V7sLo6chtJaruMM0GNdx0/UJWeGbnL68cDLg8inAmhtXf75f52awmPvTqC5TMsNgb61pRgefwg371++Meqo1KOjkn+FaN+sE1//bQ/qC1X4yc2NEAoolOhkmLZ7YVCKAXpp1uxPbm5EiAEXHft8PlitVkxNTcFmt0MkkaMof2lG5dE3xvDOiBVOXxBfubgSSqkI3756B0xOP+duFI+nP5zBf74/ec7svjbmY2zuAGbsXnzpogoopcKYEbNOLoJIIIi4CWst16KxSA2NTAR/kIY/SEdkKZIhnU4/0YPKaZrmBpVPTk6CpukIE4VYa1smxDJTArzRbAvBjJ6qUVpaivr6+pgjvtIpmPFELhl7u3QLZjAYxOnTpyGXyznz42SPxffLQVEU9pVpwTAMhoaGYLFYuEkvIyMjYBgGKqkIx+5shjdAx/VnPbgrB0aTC+dH9VgqpUJcXq1AUKJGfWFkdd6Pb2pC38wilJ4ZWCwMl3amaRqNRWrc3laCHKWYi+jYu/j7i0vRNWVHqYLG4qKNM3jX6/X49sFCSCTlGB4ejnitXJWE9xzJRGAYBj8+bsSUzYfvX7uLK+w5NWbDuyM2SAU0/iHs8R2VOrx6dgGXhUWL+0o1+LebV99njuZnfx7F6wNm/OD6WtQWqGBx+xFiGMwsLhUdCc65DoVoBmKhAGazmxt2IAr7ikmlUhQWFqKwsBBfe7obQwsufPdgAFarEbskDsxpBWjUM/B6vZDJZLg4ThX1/3tnHH/qM+H/XLMLdYUq7MxXQCUVoaEwvrD+/+ydd3xddf3/n+fu5N4kNzd77zRJ23SkE1pmkdIyC2WjAhVBUEAcKIIIioCgQkHxiwwBBRSQgkCBMstoKR1Jkzar2XvclbvHOb8/bu5pbu5Nm9JW+Ymvf3jQe8+5Z+W8Pu/1et33Thufd1m56th8VlbFNrNeMSOVebmJcjQuSRK/eL2FHqubH60o4aXaAQQEbjypKKo+fSg4mko/CoVCji4hFCBYrVZZB1cQBFlEITwDejSP58smAXi4+EoQZjiS6uvrO6Cs2tHwsJy8P5fLRV1d3SHL2x3JGma4UaO6ujrCAeBQ0NBn5/82d7A0xU9p6fSPy+fzUVdXR2JiYoQgxETizTEeuJHh7LnZnD03WkBBoVBwyexEqqqi1WR0QgDFSCvJ2dnk5eVFmocLyHZdk6FVKfYLuKeFXuB+vx+LxcLQ0BBWqxWfz0dXVxcmk+mQnCyCosSn7RYqMgxRBNs85ORPmzu5aGG2HJV6AiIb6gYRpVCUF1a/Oa7UxEULsoh3R7rWz8tL4qWrFnAo+P277WzcM8Q951TKQuWb95nps3nYO+BgRoaBpUXJ/O7cKrImpMsVgoBifJg1VlmjoW+MhoExzpydgValYMjhQwI0CcnMLihi1iyJi8dTi42Njfj9/ilTi9s6rAzYvbQMO6nINHBMsYmXr0qO+s1ui5ttnVZMfonydD17BhzkTzHbKUoSP3q5kVGnj1+fVSGbV3v8QUQR3H4Rj1/E7QuyvcvKMcWmKWd2p4N/F5EolUpSUlLkDFasOdvExERZp/q/LYV6pPGVIEybzYbL5TpoM8vRJsz+/n7a29upqqo65C7QI1HDlCSJjo4OBgYGMBgMX5gsATbU9rO5dRS3U83qxQeOMPcNO3H7g+TEBdmzZ0+U2TaM+0oe5oJgqqafcOp7qu7bQ315qdVq2QrK6/WyZ88elEolG7e3kCh4yUlJkAfVtVotb+0d5tXdQ/xgRaR028u1A/z23XbK0uJ58utzI37jxZ39vNc8issflAkzTq3k1tPK6LN5mD+h6UevVfHd4wvZscN8SOcx5gnwm01tlKfHc+mikP7qrh4bY94grUNOmTDvOrOChv4xvlaZJl+v6ilqilaXn9vf7iZHL/CTCS5at73WTL/dQ7xGyemzMnj4/JkMOXxyY5EgCCQkJJCQkCDLy4VTi93d3Qw5g7zUDieWpXDLqSU0D7sjbNZi3cP173ewpcPK13IlfrY2l8sW5UR8T5IkPmw1Y4xXMzs7IeRrKoEkhT6r7bXz7WX56LUqspN0FKXEcdu/mvnDh50oBIEBu5eKTAMzD9GI/D+JyXO2Pp+PoaEh/H4/n3/+udz8lpycfFgzoP+NDT/wFSFMo9E4rfbmIxnFwf40aljeLhAIsHDhwi8kiHC4Kdmwb2VcXBw1NTXs3LnzC+8L4NLFeejUSirjHQee6XT7ueqZnfj8AW6cr+G0Y6O7gF2+IE/WOake6+PK4w7uWBILoihFEWY49Wuz2aLq1QB1PTZGnT6WFkQ2oAREibf2DlNgiot6GQZEiae39pCdpOPU8dSeQqGgcUzLA9vdZCZqefyC/IgmjD9u89NpCzI7S89VywvlfeWb4lAqBEpjdH5euCAbtz/IefMiG1PCpHWoGHX6+N27bSwuTJadOXZ029jUOMz7LQouWpATagA6q4I9/Q6OL9tPRsWp8RQfQDKu2+ImQavCGK9md98Yu/pc1AsSPxIlOQo7bWYaH++zyCSclqCdckRjeMzLs9v7OLk8lZnFIdZ9cUcv9UPtmF1DlKnNpKhU9PU4ZIPncGRkc/t5u3GExYVGVlSkYvcEqDCF/Dknv/ybBp3c904baoXAM9+cx33nVOLyBUlL0LKrx8avNraSqFPx50uqATDpNSzIT6K2bwyzy8/zO/opNMVx7zmHp9r0n4RGoyElJQWLxcLs2bNxu91YrdbDngF1uVz/Ue3to4WvBGFO9yYfTLz7UKFUKnE4HLS0tJCfn09OTs4XXrEdTgonPCcXjuzC6ZfDQb4pnu+vKGXPnj0H3JcKEb0igFIFyxfPJy4uupN2V7eVj3u8fDbQw6VLi9DGGDWo67Vx0wv1rKxK56ZTIsXMP24d5YcvNXD6zBTOyA8RZjj1m5SUxDsjidz1+E5+t3Y2BeOpNqc3wDV/q0WUJB5cW8WcCWLsH7Wa+dXGVnRqBZu+uzji2m/vssrWV8tLTYSPNMWgQSEIZCfpIiKlYDDI9+L6eHPPEMWKYbZvH8VkMpGcnExNXiLvXr9EluWbiKKUeO44PXYDy8Hw1t5hNreauf7EIjnV+9E+M2/tHeHzLptMmAsLjJw7N5PSdL1MbNlJOrIP0u08EY0DDq77RwMGjZIXvlXD4kIjF80xkaaTIlKW647JjxqlCcMXEOmyuClOjUchCLyye5C/beujod/Bn8ablb5WlYHVE2RhgZHqnES5gaivr4+xsTHZ4Pn1dh9PbxtkYaGR351bxdcq09i2bVvE74mSxJZ2Cwk6NZUZBtIMGuLUCgRBkIX30xO0JMWpmZGxP8XeZ/OwtDiZby7Nw+ry02uNbuz6/xEThdfj4uKIi4s77BnQcKf5fxu+EoQ5XRzJCFOSJGw2G2NjY9TU1PxHJKLCKdihoaGI+c5wvXBLm5lfb2zm0sV5rK2ZWrDB6w/i9AUxxdDsPNDso91uZ/fu3Txy/gwyMjKnrPnMyzNyYkEcswrTY5IlQNOAA4vTz2cd1qjP9o048QdFmoZcnJ6nxWq10tDQQHl5OWlpabz12kdYXX4a+u0yYerUSmblJDBg90YRRGl6PJmJWmZlJUQtVCozE1hYYCQ3WUecWoHfHzr/ubmJvHHtInTqyBqQUqnkxFl5nDgrNJLi8/mwWq2yDFr4RW8ymQ5ZxaVxwMGv3mzlvHmZnDFrf4p7/fsd9Nu9zM5O4PyaUK33+NIU6maPRXh9xmuU3HSIQgf+oMh9m9qI1yj57gmFqJUCCkCnViAQGklZW52Cy+Wa9j7verOVD1pH+c7yAtbOz+akGak09Ds4a4INWIJOFWEvNrGB6J2mETY0DnG+TiRLsJMVF6TK4KG/vz+mFvTnnTZ+OR49Pnv5vJjXPCtRy91nzsA0vuDwBURufbUJT0DkztNnUJwaP2Xd+/83TCWuEmsG1OFwYLFYaG5uxuv1TjkD+t/oVAL/I8wIHKkaZlgEwO/3U1hY+B8hS5/Px+7du9Hr9VPOd9b22hga87KtwzIlYUqSxLf+uot+q4f1F1ZTMakT8bn6MT59c4TfXzCHqqxEeZuenh56enqYO3fuQVeacRol35ibSF7e1OnGs+ZkodeqYs7jXbggl9zkOIqTlPS1NdLY2BgxJvP7tdU0DY6xomI/qSgVAn+6ZB6SJOHz+SL2l2uM48Vv1cQ8jkSdigem6DKdTuekRqOR659AhFely+XCr4qnKCuV1JT9RsRuf5AfvrQXrVrB3WdVyPOl77eMsqd/jH9IEqfP3H/trjuhkM2t5ohZSmO8mltPi4zMJyMgShECCSMOH9e/0EBOko67z65AIQjsG3bxekOoueiCmmxK0kJ6tTq1Ql4QHWyW2eEN8K/dgywqTKY4NR6tSoGAgG58sVSUEs9vz41tAwYh0vb4RVlM4olPu2kbcbGgwMQ5y+dxxjJJnk1sbGzE6XTSsLeRz4YVVOSYyE8JRZVVmVPX6N5uHOHhDzo4bWY6Vy8vkEdnhsZ8cnfyv+oH6bF4+PriXOLUCnxBCe1hjpz8JzBda6+Jdeb8/HxEUYxSekpISGDnzp1kZWWh1+uxWq2sW7eO+vp6BEHg8ccfZ+nSpaxfv56HHnoIlUrF6tWruffee6N+b6ptL7jgApqamuTvGI1Gdu3aRUdHB5WVlcyYEcrMLFmyhEceeeSIXqv/EeYEHAnCnChvFwwGD9l38khgcgp2KlyyKI98Uzw1U8wGhuHxBxEJrbInY3u/h2FHgIa+MUrTDNzzZhMOq4WLZhsOSTHoYA4qGpWCVTEG/sOfHV9qora2Fr/fz7JlyyJ+d3ZO4gEH3w8HBzruEYcPk14d5dohSRKNg07yk3XoJ6zgX909yF0bW1iQ7eWKqgECgQBGoxGrFM+ObhuCIDDq9JE57h96/vwsJOCEshT5WABOrUzj1EOsdb7dOMwdr7dw/vwsvntCEQAdZhddZjc9Fg/egEicWklZup5LF+YQr1WSMa7akzxJXCFMmOExmD6bl1+sLpcJ7qVdAzz8QSeVWSM8edkcfnRKCZcvzZP3dzDc9NJemgcd3L+mipnZCVx7fCGf7DNz3HjddfKLfdu2bfT59Tz1eSeq7YP8apmOW48JRfVhJa6P9pn5v4+6uHxpHieWp+DyhZ758CywKElcsTSPHKMOtVJBQJT4+/Z+3P4gS4uTeb1+kLq+Me5YPYOyKdSIwtfmy4Yvau2lUChISkoiKSmJwsJCgsEgIyMjbN++nY8++giz2cyxxx7LihUrePLJJ9FoNLhcLt577z02bNhAXV0dWq2WoaGhmPu//vrrWblyJS+88AI+n0/OWjz//PPyd2666SaSkvaXU0pKSti1a9chn8t08ZUgzEMZ3fiihBlL3q6/v/+INhFN5xja29sZHh6eluG1QavitJkH7pQVBIFHL52H1eWX05kT8YNj0+hyCpxRncnenhFe2dWLSqnk+tXzp/wjdHgC3L+plbJ0PRcvCqUqD2WeczKcTqc8qiMIAiICXl/wsGblJkKSJHb12MlM1EapGQVEiW0DftKtHlmQAeAfO/r47bvtnDU7g5tPjWxk+lf9EL9+s5WZWQk8Ot5QAqHRBUEQEDRxzJs3U56hk0ZHOb9UgVatwD3aj00KKeWY9BquGU8LHupz+/RnPQyP+bju+EI0KgWdo26CokTr8P5U6vy8JG5ZWUqaQStL2CkVAt8+QCqy2+Jm37Cb4iQlbr/Iq/VDiKJE05BD7vZdWGCkNG1YnodUKoRxjdrY2NZhYc+AkwtqstCplVhdfkQJxryhfoOlRckxrbnMTh9/+LCD1KDIBSdlsmCfg/J0PfPnZWO1WhkdHZXttd7rVtBldvF5p5UTy1M4qzqD6pxE2Y/z8U+7eaNhmK8vzmHN3CxUCoGrl+fTZ/VSlqbnvWYzY94A9f1jByXML9ts4pEaJ1EqlWRkZHDfffexadMmNm7cyIYNG9BoNJxyyik8++yzFBYW8sc//pGbb75ZzqDEWtTb7XY+/PBDnnzySSCUmZnctCdJEn//+9959913D/vYp4uvBGHC9DwglUolXq/3gN+Jhank7Y70mMqBEG5ySUhIiErB7uiy8tfPull3bCGVX6AFPilOPaXrRX6ylhnZcZiHBxnrbeeKpXkk6uPJSpr6Bbi9y8pruwdQKQUuWJCLUiFEjZVIksQdrzXRb/Nwz5qZUb//Sl0/f/6ok6sXpWL0DTFr1izUajWjZgsXPLqNEYePpy6viRI/CIoSv3+nlYAocdOK0oi6atuIi99s2sfZczIjIrRP2izcvKERg1bFG9cuitjfv+qHebTOw9u9Dfxj3f40rt0TQACs7ugMg36cyA3aSEJfOy+T6uwEClJCC52JM3Tl5aF7bDab6evrw263Ex8fL9c/p2rhv+P1ZrZ22HhgbRWl42pMNrefP37YCYREIObnJfH1xbnMyDBQnbP/+VAIAiurpu/Y4Q2IrPtrHR5fgNtOzCA/X8nPTyuj3+5l7oSmqspMA3+9PNqkG0L3p3HQQXFqvEzSP3+9Bbs7QLZRyykVaTy4NjSSMpWubBiftFvYuGcEo0bk6ng1v5nQzZqamkpqakhZqXnAxpzgMDnxEgVxI9TVOUlOTiY9ORn1eEeWRqlAEEL/hVBnt06t5PTZ6ejUSr65JIeWIRcrZhxYsetIqvwcKUw3JXsocLlciKJIfn4+w8PDeDwefvnLX/LAAw/Q3NzM5s2bueWWW9DpdNx3330hDekJaGtrIy0tjcsvv5za2lpqamp44IEHIso7mzdvJiMjg7Ky/aWG9vZ25s0L+ef+8pe/ZPny5Uf0vL4yhDkdfBGCO5C83ZFU54H9s4qTH26z2czevXvlJpfJeGFHLx80j5CeoJ0WYbp9wZD58DQGs8PenRqNhiWLF7NsGipJCwqMnD03i/J0g0xYkxc0Ll+Q1+sHkSSJpkEHiyZJt22sH6RtxMFbDX5+c/ES1Go1Ho+HoChhdfsJShIOb3THc4/FzbOf9wJwztzsiGjgrb3DbO2wYnH5IwgzdbwDdmIEGUZxajwaBVFzid9YksfCAmPMl/pJM1J5JScxSidWEAQqMqeud2s0GrnRRZIkuf7Z2trKgNXFg7t8lDTv5N41VfLqfUuHFbPTT/OQUybMRJ2Kby7JZXBs/xykRqVgeWm0f+iBYHX5ufyZWgwaJY9dOgeVQiAjQUu/NUiSTimf66Hg7zv6eGRzF8cUJ8tel2vnZbGj20Z1dugaG+PVU2rs3rdpH1s7rNx7TiXLik2cOctOGpaI7wRFCY8/iF4bkru7+ZUWfEGJe86qoCrLIHeGdnR00NA/xkttEqsqknlwTTm5KaG/nye2dPPWnhHWzM3kG0tyuXhh7rTO72iq6nxRfNGU7IEQNjTYsWMH69evZ/HixVx//fXcfffdBAIBLBYLW7ZsYdu2bZx//vm0tbVFjnYFAjG3vfPOO+XvPPvss1x00UXy/2dlZdHV1UVKSgrbt2/n7LPPpqGhIcJ8/nDxP8KcgEMhzOnI2x3pCDNMwOE/uHAaeHR0VJaYi4Urjikg1aBl7fxodZzJaBwY47rn6piRYeDhi2J7Sobhcrno6enBaDRSXV097ZWzXqvip6dFjkxMJky9VsVdZ1XSb/dG1Vi9Xi+rc7yUJpr4+vEz5blWQRBQKeCvVyzA5vZHNSgB5CXH8c2l+fiDYpT+7TlzMkNkOUk6bUaGgTevWxTTr3FOTgLrTzYwf35kQ41KMfVwP8R2/5gMUZJweoNRTil/29bLpqYRfrG6nLzcXHJzc3HsG2Xo0wZGexzsrKtHJUgYjUZ+cUouA25FxPymIAhR9ljTwc//1czWTivr186kLF3PqNOHxeXH4vLj9AVJjlfzxGXSxdr9AAAgAElEQVRz6OntQylML73+4q5+Xto1wC2nllKVlUCSTo0AJMXtP+fLl+Zx+dLYXqSjTh/9Ni8zs0INPFs7rAw7fOwbdrGiIpUbTixgzx5nxDY/3dBIw4CDe86qoDLLQHFqPH02L6kGjdwZ6kNNVnY2u919DLk72d7rYk5SG/1tPpKSkjCpFMSpBYxxSvxBkX6bl417QmnmydZtE/FlVNM5GsfkcrnIysoiNzeXxYsXA3Deeedx9913k5uby5o1axAEQZblHBkZiVjs544/15O3DSMQCPDSSy+xfft2+d+0Wq38Hq6pqaGkpITm5mYWLDg0tasD4X+EOQHTJbjpytsdLcKEEGmErckmSszFQmm6ge+viBYEiFVPsbj8BESJAbsn6vsTMTg4yPaGFj4c0rE8Tsucw0wzxaphnlwZnQ4MNzQtnjmD1amR0UuYdLOSdFM6pygUAtedsF+CZuJvZiRq+cmpsYUTdFOMu0wn1f9FcfPLjWzeZ+aesysjVG2e2dbLoN3LR/ssXLQg9HJeWJDEpZUallRXsKjQSCAQCNU/LRbwWtld2y+nbyc7hcSCyxfkgffaKU/Xc+64eML2bht2t599Iy7K0vWUpOm5b00lcSql3PijVIQWLRC9f7c/yJNbeihL07NiXGj+td1DNA85+bTdQlVWAqtmpbOkyDhtl5YbXthDn83Dz1eVc1ypiXvOrqRtxMUJ4x3CsZ5xs8tPUJSwewMoBEFO1Ya/t3mfmXve2sfxpSa+c1wByXoNc3ISyErSMTzm4d43m8k3wNkFfl7b3kZ3/yAetLzdascXFFk7L4v732lDr1Vyw4lFEeWELyNhHkn97DDChJmXl0dTUxMzZszgnXfeoaqqipKSEt59911OOOEEmpub8fl8cno8jMzMzJjbhrFp0yYqKirIzd0f2Q8PD2MymVAqlbS1tdHS0kJxcTFHEl8ZwpxuDfNgBHco8nZHOiUb3l/Y5WOqFOx0EL4ek18mS4qSWX9B9ZRNGOHI2ul00q/O5o2WdhpHfKxeVDHlb9X22Fj/XhtXHpvP0uLYNR5BEHD7phaNCOsBDw4OTtnQdDTJazIsLn9EFAShbuK/bO2hMsPAcWWR5/lu8wj3bWrjO8sLOH12ZKOV1eXnyr/WYYxT8chFs+VIdsDuRQJGHJF19V+sLueTNgurJ8xeKgSBY3PU1Izry6pUqog6ndfrxWw209PTw9jYGHq9PjQ7Z0ji+pda0KgUPHLRbHksYmuHlQ11gygEOGtOJiqFwINrZ9I67IxIs8ZyXwmnHZsGHbQMO1lZlY5KIfBJm4Wnt/agUSk4eUYKgiDw05WlfNpm4cwJM5ex5n0nXuMNdYPMzEpgVnYC6QYNg3YvyeP3YrIqUZigui1ubn21iYUFRn5zTiWDY145XT75b8DlDSBKIULVa1URYu3NQy7qBz30O9TcsnImtY4eEg0qZsYFGB4NUsAIL26x836LHYNWydlzMpmT8+UmzKOVktXr9axfv55LLrkEn89HcXExTzzxBHq9niuuuIJZs2ah0Wj4y1/+giAI9PX1sW7dOl5//XWAmNuG8dxzz0WkYwE+/PBDbrvtNlQqFUqlkkceeQST6dDKDAfDV4Ywp4MDEWYgEKCxsZFAIMCiRYumtSI7GhFme3u7LIYwVQp2uvuK9ccbtq2KBY/HQ21tLWlpacyYMYM0i5utbcMszz/wgPKLO/rY2mHBoFVNSZhvtozx+I5Orlru5arlRRGfBQIB6uvr2TUcBH0GNZpoMu+3efjbZ10UCgEWTfrs49ZRXqkb4LsnFpM7SXy7x+LmuW1dnDE7M0r+7f5NbewddHD3WRURKdTX6ge5841WvlaZyu2rymSSfr/FzGOfdKNRKvjopmMi9vV+s5kBm5d3mkaiCLPX6qHf5qHPForskuJC92T9+TNpHXYxLy/yfiwsMEYIEEwHWq0WhzKRn3/Sx6mVWVxUlILZbGZbfXPIs1IQaO3qpzwvHbVazYL8JFbMSGVGpl6ezTyYRF7rsBPjhGjqhhf2YHP70ShDKeH5eUksKUqmKmv//GNpml6urcZCt8WNRqmQfVE3NY3w+/fayUjQ8vK3F/CbNZWy4fZEBMWQmk9uQkjBp2nQSfuoG4cvyLXHF8aMYN/cM8SjH3dzzXEF/P68mXKHLED7qIt3Gkc4pTKVby/LpyQ1nsrMBG48uZjntvehEuO5++IK/H4/jV1DtAy7SVb5CQ630y2GlJ30ev1XpuknLFwwd+5cPv/886jPn3nmmah/y87OlskSmHJbQO6enYhzzz2Xc88994sf9DTwP8KcgKkILmwLdqjydkcywvR6vVitVtLS0g6agg2jrsfG9i4ra+fnYJhUBxvzw/ee383MnCSuPeHgaYuRkRGampqorKyUV235pnjuOLXgoKouVx4b8iBcM2/qGmr/WABx3Cx4IhwOB3V1dWTm5PPwG61I0hjlGQkcUxJJvI9+1MHzn/dSZVJw7orIfd//TitNAw5yk3V898RIZZtHNrfz8q4BWoZcPHTBLPnfg6LEC7v6EUWJul57RFQVNoYedUZ2wNbkJVKdk8ic3OgFx/dOKKQ8Xc+KiugFQ1WWgTtPn0FinCoifZcUp6Zmgsh6LIiSRFCUECZF1g+8186Obhv3nF0pZws+67TSNuLi1d2DrDs2H71eT15eHvFpI/i9buLwUldXhyRJJCcn8/1laVNmUUadPlQKQT7e2l473/t7A3qNkj+cnolWqWRZSTKfd9nkSC45Xn1AQYLw+YTnVrvMbq54pg61UuAf6+Zj0Kqozk6kPF3PsePWbgpBiDk+9MaeIe7b1EZhso4fLtBwfJkJb6CE0rT4iN96t2kUk17N/LwkdvbYGXb62N03xsmTmpWe3trDpqZRvAGRa48vlP/d4vLzbtMoKoXAmdUZGLQa0tNT+dXaLOI1StxuN2azmY6ODpxOJ1qtFlEUcbvdXxqt1aMZYf634StDmNMhucmEGe4A7e3tPaAt2HT390URTsEmJCSQl5c37dXgHa830Tnqwhiv5pxJdliddomdPXYah5xcc1zRlB2xYQFzq9XK7LnzSdRH/pFPx2WkICWeH59afsDvXDY/hZq8BFbM209o4fT37NmzMRgMnFHtoHPUFbPT94zqTBr67CxPi669Xnd8MS/X9nNmdVbUZ2dWZ9E4MMa5EwyVIVSL+/WZM2gddrGsJDKtc/GCHGZnJ1KWFhltpSVoZaHuyUg1aLh0UWw1pemYR3db3AyOeanJS5KfZVGSuPTJXfTaPDx+8ayIZ/yFnf14AiI7u22cNjOUuj2rOgN/UGLhJBI+vjzyt8NdjGELKLVaLevfGgwGBuxeLnlyFyqlwEvfqsGgVRGvVqIQwqMyIfK+ZeWBlYUm46EPOnhhZz93nl7O8tIUNCoFKqWAVqWQo9x8Uxx/meTsEr4WT3/Wi0KASxfmkJcch1aloCxVhyCEfDonprAB6vvGuOftVlQKBS9dVcPVywqoyU+KMNMWJYnaHjvHl5rw+EW59gohsgxKcPHCbEzxagxaFfV9Y9z1ZiuFKXHcdWYFaq0OqyKRwrIM9Bol/f399Pf3y9JyiYmJ8rX9IqYMRwJHq+nnf9J4/+WYSHBheTutVntIijUTcbiWXBPJqqamhra2tkOKWM+bl817zSMx60yz0zVcdWwaFdnGKcnS6w1FHEajkQZ/Ojc8vI3vryjh3Pn7X/yHIzgwETq1kpqckBGwKIo0NTXh8Xgi0t+3rZ66Tjovz8hz6xby6aefRn12UkUaJ1XErvUuKkzmb9+MrSd6QnkqJ8TgeaVCYG7ufhnAI1k37TK75fpkuDs2KEp8/S+78AUl7l9TyZLxIf2gKNFr8xAISow4fUx83d5zdiV7B8YiImODNjROciA4vQGCEhEWUB6Ph79t6aB9eJCVuUF8yjgCwSAKQUn41MvS9bz87QXo1EoG+3oOukD1B0Vuf60ZUYI7Ti9HrVSwd8CBLyDSMuxieWkKmYla/nHlfFRKIWbT1aDdS5xGSaJORafZzROfdgMh5aM5OSFtX6fDQU9Pj7yN1eXnyS3dLC40UpmZQHm6gcxELVqVgji1klMmPSfvNY9y79v7KE3T8/CEDATAT19ppN/m5WenlbIg30hQlNi8bxSry49yfDH12CedPPt5PyeVp3DbqnJ0Oh1JSUmUlpYiiiJ2u12uLYuiKJs7G43GIx71TYWjNYf5vwjzvxzhppGJ8naZmZkH33AKHM5DGBZDMBqNLFiwAEEQDlkc/sKFuVw4xXyYVq1kbUXGlFHz5NnOvzY1EpQkOkYjU6axIsy/fdbNzm4bPz2tnOT42A0coijhHp+FC+9HkiS5Tpqenk5FRcUh1XsOtzYUFCUCYmw9UEmSkCBK5m533xjvdAWonidGbPdZh5WfvdrEhQuyuWLSSIQvIHLji3vwB0UeWDtTHtAH+NmrTdT3jTHk8PK9cYk6hQA5Rh2dZjdpE2qpaqWCp74+lxGHjznZeuomjBseU5zMMcXR6jdhuHxB2kdcVGYZ5HNyegOseXQ7vqDEs5fPk1O5okLNn7aNArCqppKF6Rp+nziEw2Zlb92O0JjFeJSkUiliNpNtqBtgW6eNm04uJjlezYDdywctZhCg2+KhODWeX6wup75vLOK4J4/UhLFv2MnVz9Wj1yj5x7r55CXHsWpmOkqFIIvpK2LMLb/fMsrfd/TzabuV56+cH0WC4X3/fUc/a+Zmkm7QoFUpyE+O7hdIM2gYcfhI1IWWKh+2jvJZh42S1DiuGR/baR9x4/QGcfnGJfYmHI9CocBoNMpp73Bns9lspq2tDaVSGdHZfLSahf6Xkp0+/keYk+D1emlubp6WtNzRQrheOFkM4VAjVn9Q5NpnazG7/Pzp4rmkTHjZTlVfnUpe74enlIYaNyZFq7H284cP2nH5gpw4I21K/debX27gg+ZR7j9vFstKU1AoFLIPX2VlJQG1nu+/UM/y0pSYtc9AUOS1+kGKUuOpzold5wtr32omEJk/KPKH99vINsaxtiaHoaEh9u3bR0JiIj9738qQM8iTX59D0QSFIEmSuPKZOtpGXTx52ZwI9aDvv7gXs9PPjIYhzpmzf3H1WaeVEaeP95tHowhzyOFlW6cVQRDotXoiml5OmpHCgN3LkglpQUEQeOobcwmKUtQsaIEpjgJT3AFt6QKihMMTiGh0+fHLe9nWaeO7JxRyycJQxiAoha5ZUApdpzDi1Aouqsmix+pldk4iBq2K+RWhhVbY6NlsNtPZ2RmS9hMEun1xmOwqlo93Cz/4fgd2d4DFhUbOmJ1BrlHHjScXIYoSRePKRqkGjTwOMhk2t58/fdRFTX4SJ89IjSJklULgR6dEO69IksSfdtjp+nA7vz23iqVFyZxQnsLykqm7J/+xs59Xdg/iDYjcvrqcF79VI6eERUniVxtb6bd5uPOMGcRrlMSplZidPh7+oBOnL8B3ji+Ula6+e0IRCwuMshD+gZp+Jnc2+3w+LBYL/f39NDU1odVq5YWJXq8/Ys1DRyMl63a7/0eY/z/jYA9XOKKTJGlKd4+jDVEU2bdvH1arNaYYwqE2EXn9Iu2jLoJiKGV3MML0+/3s3r2b+Pj4qGug16qiGm0g9ijHT1aWU9dr47jSqWXC+mweREliaMyLJEkMDw9js9lYvHgxOp2Of2zv5Z3GYWp7bDEJ8/3mEX7xWiNapYJPfnRcjHlSH+c88hkKAV6+ejGJ480pn3daeXJLKHVXrnPgc9mZOXMm9jEng2ND+AIiW3c1oChNx2QyYTAYCEqhDtCAJNFv90YQ5sqqVN6u75ONkcP4xuJc0gyamFFerjGO21eVERAlSiZ1nX5zSR7fXBI9pK8QBBSxjDPH4fQGeaXVi5Rqi2oU+s5zu6nrHeN351XJmqsJOhUSYJjQMJOoU/HsFfPwByXyJnQTC4LADSfFbgwTBIFPev0kaI0ct6AEv9/PltpGfv3hMEjD3HasgdkFaVyzNJvdQx6ZqARB4Ny50TXlibC5/STqVAiCwActZl7Y2c+7TSOcPCOV4tR4nvnGXOI1yqhFhD8oMuLwkZWkQ5Ik9gz78IgKOkZdHFeWwl1nRqf2n9raw+sNQ9x2Whnnzs3CFxBloY+J+/cFRHZ02/AFRAbsXtlgPDFOzYKCpFCdc0YqH7SM8sSWHq5Ymse587IYcfhCGriHQE4ajYaMjAwyMkKLzokKRIfiTXkwhAXojyTCx/ffhq8MYR4IE+Xtmpqa/iNk6fF4qKurw2QyySnYyThUwjToVDx04RzGPH5mZEQ+vJP3ZbPZqK+vx67L4MVtY1yrtzMv7+CjC7GOafXsTFbPPnAqe/0F1bQOOanO1rNr1y5EUSQ3N1celTm5Io3dvXaOnYJ0Z2QYyEzUMTs7Mea1cvmCuH1BBCEkap44/j6ZnZPI8hITap8dnQpm19Tg9/uJi4vjr1fUMOr0UZmmw2w209XVhcPhwGAw8JvTcnAJOpYURl6TH6wo4fgkc9S4RYJOxQU1U3cFTxV5Twcv7RrA4w9y0YJs+dxf3zPMhlY/nww28tZ3F0d83zLe1Wv37I9C7zx9Bjec6CM9IXJRFnZCmQqjTh97BxwsKUpGpRDYM+DglxtbAHj16oWkGjSkG/UUmTxICiULZ5Xjd9mpiDOTl+qkt92P22TCZNpvXyZJEs1DTvJNcXJ6+vWGIX61sZVVM9O4ZWUZxxQnc2J5CosnXP+MKWaFb321mU/bLfxsZSk1GUp+fKwJyZDGsgniD/6gyJZ2K5WZBlINGj5us9Br9bBnwMF587K4bVV08bptxMXbjcPccGIRSoUgywp6/EH+/HEXZel6Vs1M56ktPezqsWN2+mkZcjIzK4FfvN6CNxDksuoECpO+2Gs3bO6cnZ2NJEk4nU7MZnNEA1GYQKfSFo6FoxFher3ewxp7+7LiK02YE4fwp5K3+3fgQHq0EzG5hjlo9/Ljl+opzzBESc2FMZkowwgT3cRO4Hnz5vHzN9rY1mHh5V26gxLme03DWMecFKsPvbEpOV5DRYqX7Z9/LqtxOJ37JcxMeg13nFk51ebkmeJ5/bqlU36eY4zj6ctrUCqEiBer5HNzYb6T4uIyWZNV3mdynBxZZWVlya7zYW9Fs7mPzz/vwmg0YjKZDqkxIyTWrYiqgdrcfj5ps7CsxBRVr3thZz//qh/i56vK5BRxn83D3W+1AiE3kbD27KKCJPISFayeG71QefTiarotbmZl7+8uViqEKLKcjG6LG7PLHxE93/CPBpqHndx4YhEXLsihwBTHjHQ9iTqVLOSgVsD6s4v2q7cYDfJL3uFwsK93mO7eBgQpSFJSEp8MCvzf1iEWFhhlr9ERR8ijdGR8dCfVoOGes6Ofh6Aoce/b+/AHRX5yailqpQLveDrZFxQRRYF8o4aSkshO4H/WhizGqnMSWH/+LH62spT6vrGIlLDLF2Rrh5V5uSHd38c/7ea95lHWzsvihpP2zwo3Djp5vWEYlVJgQb6R1hEXeq2Sm04uZmFBEgFRIiiJNA06ePhTD7ef/MX7IsIQBAGDwYDBYJC9Ke12OxaLhd7eXkRRlGvLB3tOj8Zs6JdRM/dI4CtLmFPJ200lcH44mMrSRxRFWltbsdvt0yLsyTXMthEn7aMuBsa83CxK0xJLn7gvv99PbW0tarVa7gS+5vgiso06zj3AzCTAgM3DT17egyRJ3HZsAlOpzvZZPfz+3VZOm5nBiTP2dyD29vbS2dlJdXU1HtR8sreb8sQja4VWPmmx0NfXR0dHx7RHhML3LeytWFBQIFtujYxbQ6nVarxeL2NjYxgMBmp7x5AkiXl5+9Oi2zqtfPfvDczPS+QPF86O+I1fbWzlnaYRVs9K547TIxc9T27pocfiZlPjCN86Nh8INZocV2rCExApTNmfhiswxXH7MfHMnx+tEXsgsXKAf+0e5NN2Cz9YUSJL3PkCIpc+uYuAKPHwBbPkruCi1Hhahl2yAIRBq+LJGGMeYXgDIkFRIl6jDAkIWILc8NoAJWnxPH7JHGw2G2J3D8FgEPeYjfb2dkwmExfWZDE7OyFKvN7m9vNpu4WlRckkxanpt3tkQ+tLF+VSnBrPr8+cQZ/NS1FKHCMjI/Lf8gcto3SMurloQTZFKfHo1Aoqxp+RiYulMJ7a2sNz2/s4oSyF21eXc1Z1Bm5/UNYa9gZEPmo1U5oWz5p5mWQl6jBoFJw/L5MNu4foNLs5vsyEIAj8YnU5d21sRQp4iDtCtnMTMbGBqKioiEAgINeW29vbUSgUcvSZmJgY8X4L152PFL6Mnp9HCl8Zwpz4QITn+2bOnBlhPgr7R0uOFGGG9zdZGSicgk1JSaGmpmZaD2yY5MJYVJjMT08rJ8cYd0hkCaF6ZXNzM2VlZWRn7yfH4lQ9N54cW091Ikx6DQsLjDi9fky6qX/7tfoB3qgfpHnQwYkz0hBFkb179+L3++WRkW8/9jl7B+xcVp1ERexAmUBQZFPjMGXphijR9KAo8cQnnRi0KvJjbBsMBnnugzr0igCnL9s/puL1B/nxPxuIUwnctqo8wuprR7eN656vZ0VFagSRKZVKbFIc6/41Qnm6gYfPK2fHjh10dXXROTLG7Vv8CILA374xm5KM0LM17PAhAb22aOu4ObmJvN8yyuzs6NnS204r452mkCNGGGqlgvvWHHj4fyps77Lx+KfdXLO8ICLavO+dNuyeAPPykjhvXDdWqRBIM2gYcvgiDKJ/sbqcn60si2ikmozwQsPlC7L2z9tx+0X+Nt516/GHFkUuX6huZjKZuOJkE6sWeknUwJjNSn9/P3a7HZ1Oh5lkMJmIj49HEAR+9247b+wZZtXMNH6+qpycJB1XLy8gKEryAkKnVsop8vCxBEWJO15vwRcUKUmLZ1mJiTeuXRQV8W/eZ+azDitXLs2jMtNAglbF7HHLs8kKSxv3DPHAex3MykrgwfNn0jbi4v532tHrlNT22mnoH2Pt/ND1tLsD3L+mkq6uLrSqoz8uolKpZGs42N9ANDAwQHNzM1qtVu7APVoE92VTNDoS+MoQJkxP3i5McEdqiDhWjS+cgp2omvNF9qVUCJxadei1sNe3NXPXpj5OrzJxfPbBHUxiQaNSsP7CObjdbvbu3Tvl91bPyqR50MGqWRm43W5qa2vJysoiPz9/vzxaup6mwTGyEqZ+HN/cM8Str+zFGK/m3RuXRXy2p3+MP3zQDsCdSyPvm9fr5fl3t/P77R6USgWrlu1/0bcOO3m/eQSAby8vkMcRwp/5gxJ1vWNRx9Jj9RAYN0XWaLWo1WpmzpxJgcdPav0O/IEg/Z37MHcFMRqNLEw38uB5lZSkRUe1lyzM4eIJtciJWFRoZFHh9CTwJr70zE4fd25sYX5eEpct2j9W9Ngn3XzcZkavUXLvBG/I608s4tM2CydM0L9VKgSeu3I+oihFkKMgCGhUsV+E/9jRxx83d/HNaj2nJycTECVcviBBKSS8DrCsxMSTX58To3Ya+v94XajJRZIkucll4+fNbO1xsaYqiaIENfFqhRzxCoIgd/lOxu5eO/e+2ckZ5QYKCwXOm5dJy7CLWeONOmGynJguf+C9dvptHsrT9ZwxO4Pjy6JLJHZPgD39YxSa4shK1LKwMLzoljC7fKTo9XxzcS4mvYZ4jZI732hhV4+dy5fk8H7DKIJSxY9PM03LseZI4UANRC6Xi/r6erkD98uiQPRlxFeGMF0uF9u2bTuovN2R1n+duD9RFGlpacHhcLBw4cJDKsyH93U4UnvBYJC9e/eys3MMt6hg78jBzbIlKaTJmZ6gjYrs4OCNSNlGHb85dxYjIyPs2LGDqqoqkpMjO0dvP72Ca5ekYx4dnnI/JWl69FoV82PUVcvS9SwvTSFBpyJJu38Y0Wq10tDQwNzyQuLr95GbHCePBwBUZiZwzXFFaJSQNamB5Jw5mRjj1FRlRZPc8pJk7jm7klyjLiJCMejUvPadkJKtIAhy+tZsNqOwWumyKHGMN7xMdAz5oivxbouba56rZ1Z2AneuKpH383GbhQ9bzGxpt0YQ5neOK8CgVXLlMZFduOfMyYwYiQlDpRBCQ6Ax8MxnPfzpoy5+emqprCT00T4LNref3YNeTifUdfvUN+biC4gRYzrhMRqHN4BBG/kK2tFtY3uXjYsXZKOPjyc+Pp5n3xigbUSkMFvLqiI11Qk6At5uWlrGImp0bzcO4/AEOWtOBgpB4M29w+wZ8qBRwPnL4JrjCqPO47MOKz99pZFlJSZuX13OumPy2NJulbuJY+H+TW18tM/M15fkRhhh2z0BeiweLK4AN55cLD8bSToVSgEaBxx80OFCoxJoHnL+WwlzMiY2EG3dupXCwsIj0kAEocDk3yW68O/GV4YwdTrdtGpXR8uSy+12U1dXR1paGvPnz/9CL8nDUQ5yOp3U1dWRk5PD98+soODTJqrSD97Ftr3Lxg9frEelEHjr+mOjUnEHU/qRJIl9+/ZhsVimrNMKgoBadWDirchMYPMPYrun69RKHrggJEn3ySefRDUyxcfHs/kH0VGcQiFw1fLCiDR3GGplpIfk5OM9bgqz5Ym/oVQqI9JiYceQvfs6GXM4yUw2YBon0G57kG8/u5uFBUkxm1sefL+dTY0j3LemSq7r7R1wMGj3Mur0IUn7xz6OL0vhzNkZUZq2s7ITIiLL6UCSJK5+rp62ERdPXFZNrjEUfWzrtOH2i+zqscuEecvKUj5sNVOstsvXYXJdMIzHPuni0Y+7uXp5fsQYzS2vNDHs8GGMU8vpzEsW5vCv+iFWzs6hMDWewsJChsc87GwbJCiO0tbWhiMg8POPPSgEgfIMPTOzErhsUS6i18WxBdELvaAooRBCXb9BCQbHQovHlVXprKyKlNBzeAP8/LVmkuPU/HRlKcWpcXzWqaBw3Pey2+Kmz+Yhx6jDGK+mODUeYfzafdxm4SFH3EQAACAASURBVGtVaXxzaR42t5+2AQslafooQf3/FMLNOZMbiMbGxjCbzVENRElJSQc1nvhvFS2ArxBhKpXKaTV6HI0Ic3h4mN7e3pjR1aFgumMlbSNOHt3cwZp5WSwsNDEwMMC+ffuYNWuWXLM9oSRpWueZmaglTqMk3xQfEZ2FMVHpp23EybqndzIrO5EHL6jG5/Oxe/duDAYDNTU1KBQKHnqvjV6rm5+tmiGr/IT3EyZeUZTwB0W0U3hQHgiSJFFfXw8QIWk4nQXKVGo+B8LObhtvtPuZNTcYodgDIRm291tGOaEsBWO8Gq1Wiz45jR8910FAlHj64mwCfgeNjY182OXG5vazpd1CIBCIeim9unuIYYePbZ1WmTBPmpHKT04NhF7QE445Uafi56sPrN27b9hJgk4VkRrttXr4xlO7yDfF8edLqlEIAgFRYnffGKIo0WV2y4R562llfNJmiegqTU/Qct68LJqbo9PYLl+QF3f2U5OfRFVWAn3j9dy+SXXdc+dm8n7LaEQqOlYEfPOGJhoHndx4UhHnLSzH7fGytHsvFoeHkbYGdo+E7MvWVhmiyisdoy6ufrae0rR4HlhbRbZRJ5NfGNu7bAREkcWFyXSa3dT22FEIAt/zBvnGkjwuWZSLShF6Zn/w0l7GPAFuX13O09+YK9+L2l479769D51ayd8un4cvIHLF3ARK8zKjnpX/FGI1OCoUCpKSkkhKSqKoqEjOlFgsloM2EEGIMP9b07pfGcKcLo4kYYZbvZ1O5xdKwU7GdAnz1boBNu4ZwuLyY3AP4na7WbRoUcSLY3ID0VTITY7jre+FrKoONhvaMerC4QlQ22PDarXK8oLhuonDG+CJTzsBOHNOFkuL90dpEwnzuufr2NZh4ZFL5lITQwcXoHnQwSf7RlkzL1sWJXC73TicLp5vV+MUVfy+TCJh0nvpN2+18OaeIR68YDZVWftX+W5/kLV/3oE3IPL3K+dHNLoAvFI3wP993M0tK0sj0nU/+OdeRh0BKnYPcv78yHrwXW+2sqlphE/bLXLUKEkSwXH9WW1cPDlZJvLz86mcGSAjvYtUlZedO3fKDTHh9O39ayrZ3mWLIA6VQmDN+PD/dO5lGA39Y6z7ax0apYK3vrtYlvTrtrhxeIPsHXAQCEpoVAJqpYLHLqmmz+aJOO9UgybCw3IiXtzroP0zJ78+R0/KuLflP2sH+N177eQadbxy9UJ+uKKYUyvToqLgdcfms+7Y6NYtUZJ4r2mUfFMcZel6CkxxNA85yR5X1InTabn//LnyNXa5XFgsFoaGhvD5/bzTOEJpdjILS7MYHPPiC4q0jboQBCFKdGJ4zMtPNjQC8Nil1VRmGrjxpJARdOL46I9CCJFqXrKOWdkGGvodZCVpEQQBf1Ck2+IhM0ErKzFZXT6+/+Je3G43y0oF0oxOshK0LCs1RQkv/DsxnYmAyZkSn8+H1WqNaiAKKxCFI0yr1cq6deuor69HEAQef/xxli5dyvr163nooYdQqVSsXr2ae++9N+o3p9r29ttv59FHH5V1ju+66y5WrVoFwK9//Wsee+wxlEolDz74IKeeeuoRvlr/I8woHCnCDKdgVSoVJSUlh02WAGM+kQ86XGQVRqr2QOglUdtjJ98Ux5q5WYyMuanQ2tHpkmJqsk4kujFPgN+81UJVVkJM7dkDRWcTie6EslR+fc5MtD47e/fuZc6cORGpGYNWxU0rSum1eqKIcOLxdJldiFJodGUq/PifDTQPOnD6glx7QrEsJygqNXzY4Rj3QBxjQUFkRP/mniEG7B52dtlkwpQkCac3yKgzNPtncfmjCHND3RDdFjcb9wxHEMcZs9LZWNcbU+B+QX4SH7aaqZkwYpIUp+bFdTUEJSmiyUirVnHBkv1pVZ/PF2X4fGqBCYXoByJXAf6gyMMfduEf8zFvXsRH9FjdXPZkKGp84rI5KASBOLUSgZAt1sSkweJCI3edOYOMRG1E6r0y00Bl5tTZGZcvyIDdS3FqPJIk8XKTk6AUqg+G07UL8pPIT46TzZh1auWUDU0fto6yqXGEa48rlGdoP2gxc+trzcSpFLxz/RJuPa2MH51SElOUXRAE9Ho9er0eSZLYNeDlz1sHUdY5ue/4ERSiyPcWJlCSZUISRVAq6bV6eGRzJ6dWpbGwwEh5uh5/UCRFr0EhCKyeJDTxUauZX25sJceo44nLIoeqHvukm1d3D3LRghzWnx/SqrW4/KgUAr6gyKYWKxbXKHnJOpL1aubmHtjC7Wjii6j8aDQa0tPTSU8P3dtwA1F7ezvr1q0jPT0dURS54oorWLVqFS+88AI+nw+Xy8V7773Hhg0bqKurQ6vVMjQ0FPM3rr/+elauXBmxbRg33ngjP/jBDyK+v2fPHp577jkaGhro6+tjxYoVNDc3H/Fa6leGMKdbMzwShDk0NERLSwtVVVWMjIwcMU/Mpz7r58XaMYalfdx+RmQt6q29Q9zxryaK0/T89vR8VqbaDpgCnkhQWzvMvNEwyDtNw1yw4MB+n3a3n+YhB3Nzk1ApFRHflSSRdP8AEhIzp3B4uXhRtOwbRBLvY5fNp23EGWGzNBmrZ2fi9PZwTLGJtrY2RkZGWLBgAdu3b+fOM8sZGvPFFF5Yf0E1O7qtUXOmqQYNj15cjT8oxjRJ/tlppby5ZzgqNXjDScUcmzAac5vza7I5P4baz1QKNWFIkkS7xUdOcqosrhBWdanfs5f3OtxUZSeyuDQTo9HIrl4Hz3zejyiKfOtr/oiZy26LB5c/SNOgU44ai1PjeePahWhViojoRhCECHeTydjSbuHlukGuO75ATs0CXPlMLW0jLu46q4KTZ6Ty7fkJDPi0ER2mMzIMvPztBRH78/iDXP50Ld6AyJNfnytHb799p50us5vi1Hi5vllgiiNBq5THYQQhtoMJhGY1f7WxlZlZCZyQJZJv1JKWoKUsXU/N/AqCwSBF481YO3b0oFKp+GBAycY9djotbpaVmCK8USee/9+29XHVsnzSE7Ro1YqIWdiwN2k4YteqQs/0P2sHUSkEHlhbRW1DI/Vj8Vh9Id/PQtN/1gLrSMycT2wg2rJlC88//zyPPfYYGzdupLW1la1bt3LHHXeQlZXFH//4R26++Wa5lyFMuhNht9v58MMPZZNojUZz0IBjw4YNXHjhhWi1WoqKiigtLeWzzz5j6dKpxU2+CL4yhDldHA5hhm2p3G63nII1m81HjDAXFRr5sGmQpTGEo1P0GhQKiMdLZ2fnQVPAEwlzSaGJs+dmUZmZcNCFxQ9famBHl5UbTirhksX7yc/lclFbW0tubi65ubkyAQZiCIbHwkTCzEjUHpRU1h1bwDcX57B79268Ol2EqfbpszOnPI/KrISYfppAxGziZBSlxHP18mhRgImYSqCiz+ah1+phQX5SxOev1Q/xesMQPzm1JIKA3tw7zK2vNlOcGs/zV86PUHWptWl5trmZuHYbf0nV0dHRQUAUmJepIV7wk6iLJJElhUbuPquSjARNRNQ40ag6Fmp77dT22Fk7P0uut93/ThvNQ04yE7QRSjeq8fsbrnEfk6sjOzs7prEzhFxRVEoFY54AXRYPAoy7foReR9ccV8Cbe4YjrLaKU+N587rFUfva1WPjrjdbuWxRLmfMDkWBn3fZ2LzPzKftFpafk0VWUhwvXbWfrMMi5/GJyfz93TZyEtScVKagy+Kh2hgasQjPKCrVWjwBkUSdig11g3zWaaXAFMdNK4p5aZIo+/df3EO3xcPvzq1k9ax00gwaui0entzSjSAILCo0MuIMsLAgker8FLwBMapL+N+NIy3SolAoyMjIoLS0FEmSqKio4IMPPuAnP/kJDz/8MM3NzWzevJlbbrkFnU7Hfffdx8KFCyP20dbWRlpaGpdffjm1tbXU1NTwwAMPyNmqhx56iKeeeooFCxZw//33k5ycTG9vL0uWLJH3kZubS29v7xE7rzC+UoQZSyh8MhQKxQGdH6bCROWgiSnQI1kTPa4slSRXLwtjzF7OzoznrmN1ZKQmU1ZaGvXilqTQTFy40WYiYRp0Km6ZQlpvMjITtSiESFk1v9/Pzp07I5qKJEnisie20zHq4unLayhKjd0191mHBY8/yIKc+Jj3RhQl+u0espN0EefkcDioq6ujoKCA+OS0KUc0pPFVv+oQ6kSSJPF24wjZSbooEnX5gjy/vY+FBcaIz9pGXFz65E4qMgw8PilFd8kTO3H5gtx9dqXsWgGhztcBu5fX6xO4atl+MtaqlEiEXEImY2ZWAql6DQsLQp6KEErf5ucM097ezrZt29Dr9REzdSdO4QACobrl4592c+7crIjz+f6LezA7/ehUCjlKvmZ5AS/uGuCsOZHP36MXz2bU6SfHqJOvX/g+2Nx+1EqFTJ59Ng8XPb6TeI2Sf15Vw4NrZ+KbFNWfWpnGqVN0KEuSRMuwi/xkHTq1knebRtk37OJfuwdlwlxalMz587OYkWFAwDnl4qmu187GhmEUCoGLrlvEb0vz5WjeYrHQ1NTErz4Zw+wTuGtlAZcvzibfFMdZ47Xb8ELQ5vYTr1HSZ/PgDYjY3AHyxyPHbKOOlVVpqJUKRFHisVoncc2d6DU9DDn83LemksrMqRdqRxtHQ3jd5XKh1WrZsWMH69ev56mnnuL666/n7rvvls3Jt2zZwrZt2zj//PNpa2uLuEeBQEDedvHixfK2d955J9dccw233norgiBw6623ctNNN/H444/HfHccDeGErxRhTgcqlQqfz3dI2wwODtLa2srMmTNlb7swDtdEeiKmmsMcHR2lsbGRqgNo0d7xWhNv7x3irrOqOK489ZCF3MO4/fQKfnhKGQadSja49vv9LF26NCKiDYoSnWYXAVFicMwbkzAH7V6+87da/h977xnfVn23/7+P9rBsWd57xfGMYyexMwmEHQghgzLL3pCW3pRRblraUlqgZZQGWkILIUBDywoj0DACJIEQshM7iffeS7Ysa0vn90DWiWXJCSvt/f/T6wkv5JzvOTqSzvX9rOtCgBcuLwl7PY99VM9LO9u4Zn4GP17kt2/q6emhoaGBadOm8cKePtZs+4wbT8rk1lP8NcDxD+xbXj7Al81m1l4xg+kTakUtAzZ+8molC6dEc8tJGdIxXzSZ+cU7NSAIfHb73KAI+Y393TzxaTPxBhUfjIt42sx2vCJU91jxiWJQp22KUUNdr434CXXnu8/IYdPhvpDmmUVTY9h48yxMYbxEs2N1IeLqKpWKuLg4ent7mT59OmaLlVHLEDU1NbhcLsmU+LXDViwuH7efmiW9p+e2t/H6/m4a+my8cOVRibtzi+LZUjfAjHHOJ6fmxYZN2WqUclKMRx+6gfvfZrZz6dp9aJVy3r7Jby5tc3nx+kRGXV48PjHEWSUAp8fH6i3NpBk1QSL2r+3v5vHNjVRkGPnjD4q4ck4qRp0ySHRBp5LzP2PuKhMfxo9vbuRQt5UHl+ZRmhrJitJEUsfIN4BD/W6So2IpSU2F3XsRPC4GLVYUo72Ua0Xs/R7MPv+IxeaaAf6wuZFzi+J54oIiBm1uSZR9b+sQT3/WyuUVqZw0xYTD7SU5QkZUlJYdzcM4PT4a+mz/UcI8EcLrNpuN+Ph4UlNTmT3b/1294IILeOihh0hNTWXFihX+iLuiAplMRn9/v9TEA0hZqonHAlIDIcD111/PkiVLpGPa2tqkv7W3twcpmH1X+C9hTsDXIbjxKdiJXagByOXyr9XB+HWuTRRFGhsbGRgYYObMmcd0B+gcduAVRWne7FiE+UXDAFvqB7hhQSYmffBDWxAEIjT+TcWBAwcwGo1otdqQ9K9CLuPFq2fRY3FQMUkt0qhVkB2nw+H2EWdQ09IdukscdXkR8MusiaJIXV0dIyMjlJeXo1Qq6bG0IxOgd9z7Gr/brOmxIvqgddAeQpjbGwep7bHSPezglnHp1gyTDoNGSVaMNmSUpjwjipQotaQnGsDCKSYePj+f1GhNyFjKC1eW4vT4QkYJJiMgOL5rCISOwQiCwB0bqtlSP8DqHxQzrzQdr9fL8PAwde19PLO9BwEoiXIzLy+JyEg/YdT327hqTnCz1+2nZXP7aeEtvSbDe4d6efzjJi7KU5KeftRb0+nx4Rv7SKbE6Xn+iunoVPKgdGRNj5XKzhHOm5aAWiFjT+sQr+7tQgDOm5YgRahquQyRox6nMXpViN9oAP/Y3cEHlX3ceaqO2Fj//XrzYA9ur4/KzhFOzYsNeY9fNPmNv3UqOe/cNIs1l0zDbHNLoh0ej4fe/kE+rGwnRl5HZb+I2+2hZ9hGYqSapHGNXJ83DlHdY2VL3QAnTTFhdXq5NF/F/PI8Pq4z0zJom3TW99+F71IGNAC73U5iYiJpaWnU1NSQl5fH5s2bKSwsJCcnh48//phTTjmF2tpaXC7XUZH+MUx2LPhlTZOS/J3hGzZsoLjYX2teunQpl156KbfffjudnZ3U1dVRUVHxnb4v+J4R5ldJyX7VFGogBZuYmBi2C3X8eg7H5N2eXwfjzxGYcdzdJ/BhC/wswcGsjMkfsr9fUURd76gkCn4swvztplraBu2kRWu5LEyTTkBBJzc3lxqLnLaRHuaHWSczRhfkHTkRaqWcV66vkN5PuM/mnrOnsqIsmWyTij179hAVFRUk/HDv4qmcMy2BsnFkOH6ddVfNpL7Xysm5ocS0tCSRfquT0pTgDtAUo4aPfhxaLwN/80pAzWc8BEGY1Pw40Jk6GXosTrY3mTkzPzZoNrVl0M71fz9IYVIEf7ygKOgYp8fHyr/uYcTh4fXrZxI5tl9pGrAhINA+ZAeipfGUiuhoLulRYB51UpQcSUdHB9XV1eh0Oh46PQ6TKXzK3OH20jJoZ2p8sGHxiMPDE582MSM1inOK/Y0bH9f002d1sbNT5BJBICdOz9+vKkWrlAfVM6eEUYy6/fXD9Iz4MzsXlCUxPSWS0/JiSI/WBh27tCSBWRlRxE2I1ve3W9haN8APK1KkTd7zO9rptzrZ2zFKQYb/M3p4WT71fTZOGhOe6LY4+dmbRyhKNnDn6TkkRanRKGXkjb1fk14VtGlUKBRs6/Dwl71WSlMi+d2SbGbWdWEQbezcuRO5Ws8LR1wkGvVcf1ImyUY187NN9Fic3PbaIew2OzutzczJMrGsJJG6XisFSYavNfv7XcLn850QL8yYmBhWr17NZZddhsvlIjs7m7Vr16LX67nmmmsoLi5GpVKxbt06BEGgs7OT6667jvfeew8g7LEAd911F/v370cQBDIzM1mzZg0ARUVFXHjhhRQWFqJQKHjqqadOiNrQ94owvwq+CmGGEwKYDN809XksjCesF+q7aR4YZkfTILMyJtcdjdIqg/5+rOu6dl4GH1X3hZBMQEGns7OTsrIy6gbd3Pn6fjxuN+cv8gQ97CciMCKSGBWe1CdTDFIpZGQYYP/ePeTm5oZ01amVcuZkHW2CmrhOOBeKAPRqBT9alDMpWR8PNpcXZRhT50NdI7QM2jm7MC7oQfj8jjZe3NnBb8/LY8640ZR73qpmX7uF+t5R7jwjR3q9ttfKoM3NF01DIQ1FdpdXiqoHRl1EjkX4ay6ZRk3PaIhxtSAI3DVubUiW5hUHBwd57uMqeqwuLi6JIS42hujoaBQKBXe9eYTtjUP86OQMrhynyLO5pp839nWzsbJXIsw7T89hWkokWbJB6VqzJtkw2VxefvLaYTRKGY+u8Nd2P6kdlETo9WpFWKNnQBrHGX9PfrupjsZ+G3q1QpL+u3dxLh/sa2RhztHv/Zys6KB7X91tpb7PRvuQgztOyyYrRsfGm8sZ/6m6vT7u21iL3e3ld0vzyTBpUcllTE3Qo9NqObnkaClgT2Mvh3rqOdQ9yrzIIYpjjcidCtwKPQLg8MCH1f0c7LTSOeRABH63NPj78O/EiYowdTodpaWl7N69O+TvL730UshrycnJElkCkx774osvTnree++9l3vvvfcbXvVXw38JcwKORZg+n4/q6mqcTuekKdivs95kcLq9PPR+HWqFjDvPzJVcNERRxOl0Ul1dLUm+3XWWge0NMZxZGNqefSy4vCIeb3jCXF6WzPIJYxcej4fDhw8jk8koLy9HLpeT6HESqVGgVXulVvpw6B1xsmLNTgDevHl2WB9Gt1fkyT2jJLZX8dvzC6UaW0dHB62trZSWltJq8fHG1iYunpUa1q5KFEU6rV7y3eHF830+vzh2bETo+a1OD7W9NsrSIoNIzu31sfaLdjJjtEHps+YBG5es3YdJp+TnM8edQxS55sUD+ETQKGRBKdf3j/TTb3WxrWEw6AE5O8vI4W4rpRPk0k6dGst9i/02XhMzGEadkrWXT2fU6SUvIUKqu8cb1F/J59Lm8h+n1+sRFWqerWxGJgicVKBHbbHQ0tLiF7dw+FPhE1PTC3JMzMkKdu9IiFRz5exUqqqGQs7p8vh4p7KHoiQD+YkRtJntHGgfRsTfIXvH6TnccXpOyHGAVPOMHOcX+sGRPn71bh1XzknhxgUZXFaewtuVPZySe3TzdFKOiXhPX9Bx4N9g/OKdGgoSI7jppAx+eloWWePUkiZGe2abmy+bhxCBrmEHc7Ki2bQq2Onkk9oB1n7Rxi0L0/npGbkYtQrmZBrZ29DNfRsaKI0RualAydCoSAvRmCJ1vLCzE7fH9x/VlD0REeZ/pfG+R5iM4EZHR6msrCQpKYmCgoKv3IH1TSLM9iEHW2r7QYCr56WTEKnB4/FQWVmJz+eTiuXgN0r+wczwbg2Tocfi4LLn9qHwuXm71HdMqyY4qkOblpZGaurRWldCpJoPbpvP9u3bg6yxJkIhE5DJQEAIK68H/vTj/j4vsoE+Vp3iINWoobq6GpfLRXl5OQqFgrvX7aC+zz9PuGpRaH3tjX2d/GqrlVmtVay9clbI33+5sZo393dy91lT+eHs4FTzba8e5mDnSEgk9XmDmTWftSDi12gNbAwsDg8+Ecw2Dz7x6M9IJgjMSI+iuns0ZDbzd0vz2Fo3yLIJXaY3LsjgxgWhIytymTCpmg74O2aPhe2NgwzZPSwuPNpFbHV6uPDZvfh8Ii9cWeonTZWcJcUJdAzZmT01SSIYl8vFz5MGqevoR+Npp7LSLKkPxUZoQ7w9A/D6fGw6MkBOokdS0Xm3qpffvd9AtE7BRz+ew9R4PXedmYNWIQuq+4XDHRsO80XjEH9YXiClUo90W/H4fFR1+mX4lk1PZFkYAfln91nY2T3AHy846ulZ2THC/nYLVV1Wbj05k+WlSXh8YlDE6vGJ3LXhCMMOD39cWcgvz8nF4fZKn6lMEDjUNYLN5aU8w8gntf3U943yWYM5iPg77DL67NDuNjBvVh6vfbyTPKMbo9jJqhI5KXFxJGrFkAzCvws+n++42rBfFzabDZ3uPztfeqLwvSLMr/KFDEeYx/LP/CbrHQ9ZMTpWLcpGrZARb1BjsVioqqoiKysLu93+rVMoFocHp8eHzePD5T02YTa1dfKnj2qZW5jB3NRQFSA4Whue7P6a9CrevdU/QDzZ/N/UhAiWTVGSm51FnFZg165dxMfHB21OVpQl8cqeThYeY0ziWBgcdSEIAoOjoV3QMREqfKIYoqBUkmKgMMnAlFgdqnHp15KUSJ69rIQorYLu+qqgY/4yCZFkxegmTVEeC9vqB/j9h42sOjkzpNlIFEUe+qCBfquTizKPppWH7W5+9MohBEEgOUotqcko5TIi1AqsTo9UGxQEgfuXhGrPqlQqUpISSUlKDErfBhwtAoLccq0Bt0+QZmf3dbt5an8LckHg85/OQy4TKEo2EBOhZMGYHKIgCKwck/WbiGc+a2F36zC/PS+POIOa/pGjCkwB3LAgncKkCGakBf8e6/tG6RhycNIUEzJBoHbAjcPto7F/VCLMednRXD8/nZw4HTJBoLrbyo9ePURJioFHx7xGRxwe9rX5I+COIQcn58ZITVYC/t/QT984jM8HT11UzC0LM5iWbGDR1FhcHh+3v34Yq9PDH1YUolXKKUiMoHfUywvVPvQ6J/eenc9rmxsYONjNrTMsxMidGAwGaf7zeGby3xW8Xu93okI2Hv+NML9HGE9wXq+X6upq3G631JX5dfFNxkpkMoEVZf4aU0dHB21tbZLTSlNT03GPF0URp8c3qRJKbnwET18ynfrqQ5MOTgesyLbUm9nSKbK9p5ULyjPDGlUHouhtDYOMODycW5wQQp7HG5SXyWScnamkIM/A3r17yQ8zInPV3Ayumju5eMCKsmQ01i7ml4V35PjDymIOd41ID07wz3NWVVVxVZ6G68qyyU4JFoUw6VW8OG7cYjwCc4vdx3xnk8Nsc2PUKqR7ZXN5uWF9JXKZvxYZ+PzeqeylZdDOGwe6QwhzYNTNK3u7kAlQbtQSaEfSqxWUpEQyMOoiY5yajFohY+PN5Xi8vrA15zaznWG7J2T+9Hfv1/NuVS9P/KCI8ulp+Hw+hoeH6eod4Pq3j+DywiNnJ1KalUCKQUa0VkF+YoQkvTc1Xh80hjMeHUMO2ofsVGQYEQSBdV924HT72NE8xHnTEnjyomKaBmxBmq9apZwz8uPw+ERpjMcnily/vhK3x8eD5+dz0hQTt5VHYFObOHOctJ1KIePqcZ213Ra/tmzzgF16LVqn5KFl+VidXvIS/DJ5N71cyZDdwzOXTiNSo2BKrJ4hu4e4CBVGnZKlJYmYbS7sbi+tZju+sdnnRVNjaB6wExehJFojkBCtYc22Fg50WlHIBIaUcSyelYzVamVwcJDDhw/j8XikDYnRaPzOo8AATlRK9r8R5vcEAcIcb4eVlpb2jdMl39TD0uv1cujQISDUdeN4s1N/3tLE+l3t/O/ZUzl3WmiaCqAwOZLhlvDHO51ODhw4QExMDBecMoMjthqKkgxhyRL8ZDdgdfLTV/2RVoZJx7SU8PZFXcMO/ri5gXOLE1g49Wh9TxRFCqIPfgAAIABJREFUXC4XNTU1lJWV0T7iQ+/2Tkr64SAIAskGeVAkOB46lTyo8am3t5f6+nry8vLwer0MDAzw4ua9JOhkFKTGYDKZJDeGjiEHHxzpY0lxPHETaoSftLlZ33KIny/OlcTGwZ/+XPXPKow6JY+uKAxKW//t81ae3NLC5bNT+OnYaEPXsIMj3f4UY++Ii/QxB43bFmWRatRwXpj0bIxeya0LM+gbcZBrOqq3qZAJIRqnAagVMkk8vDjZIKWZnR4fFz23D69XZM2l04I2FrtahnF4fBzptlKeYZQcK7QRkcgVfchELxqNhs7OTlQuC78/KYq4uGgcDofkXCGKImt3tKOQCVxecVSC8YoX9jPq9PLg+fksmhrD/UumcrDDwmlj9d8obXi91b4RJ5c+v59IjYKXry5DpZBRnBRBTc+odO/idDKys0MFzm0uLz96pQqlXMYfLyjkD8sKSIsOTg3Pzoxm2O7G4vAglwl0DDnxiiKDo25i9CqevKg4KLNy79vVVHaO8MtzpvKH5QU4PD4yTFqe+byVDfu7WVmWyP/MUBOZnsHaL9qYGq8nI1rDWQWxCIKAwWDAYDCQkZEhjQMNDg7S3Nws3e+AGP931ahzIuYw7Xb7V3KG+v8ivleE+VVITxAE3G43Bw4coLi4mMjIb+db901SslarlcrKStLS0iSz69ZBGwc7LESKxyfM1kE7Xp84NloQHsEasP6migi1ArPZzOHDh4MivMd/ED7FOH4tvUpGeaaRYbv7mKMkb+7v4p3Kbio7LRJhBjYHXq+XiooK3tjXxQP/qmFGupHnrpgRdp3NR3pZ92Ub95w1NUjqThAEhu1url2/i8QoDY+uLA4h+q4hO7/asJ+ZcSJXnTlLugd7uj08fbALjULGa/kRdHV1UVNTg06n47E9DvZ22qntHeXB84O7N1+t9eAWB5ibHR3kWFLfN+q3xhL90eT45o4uixOZzD9SEkBOnJ5fL5mKTBCkBz74x1x+vOioFN3Ee3/d/HSpGWwizDY3Hx7p49S82KDzP7q5kVf3dnFWQRy/G3s/cplAtFbBwKibKG3wo2H1hUXsa7OERLgapZy3bpyF3e2VuledTieZmZmMjIywt6oat8tFUqyRYXT8eWsLAv56cMbYe0yP1lLdYyXe4L++0/JiJbIcj92tQzy/o52bT8qgKMmA2e7B7vbicHtxePylhYDYeQABQjPb3PzrUC8Lc02kGrV0W5zU9o4CMGT3hBWCH7K5uez5fQC8eGUpqy8sYtTlJTfeL+p+95vV1PaOsvrCItKitXhFEEX/OfMSIvyG0kN29GMi93qVnFGXyINvV2Nzebn3rFypJjsR491qwF9PNpvNdHZ2MjIygkajkdScdDrdN97Qnwiln9HR0f94SnbDhg38+te/Dnrt4MGDvPvuuyxevPgbr/u9Iszjwev1cuTIEdxuN/Pnz/9O0iBft+knUC+dSNa/fKeaw10jrJwiZ85x1vvFuXks70xiRhjx8XB49KN6Xt3TyY3lJooibMcVQZgImUyGQibwl0vDpy7HY8m0BCo7LCwda9AYr0FrtVqRyWSoFP7h9GNp0D69rZnKTguv7u3gvnOPEpjfpcTK4a4RqrpGsDo9kv0X+Lt9n/rXHrY0O6gf0nDDEpXUYZpi9MvvTYnTS24Mgdrdgv4WGvodpMv8NbzAw0oul3NxnoJuojl9wkO+JCWS2xZlEa1ThnRC3n1GDmfkx4bYWy0pnrzJB/wjK3ERqpBO2MrOER7dYeWO6OEg9ZyHP2xg06FetjYM8uQ4MonWKvGJBHUbK2QCb944C7dXDNGBPdZ4jloh48Uv2ylNjWThmOKOTqfDIdNy1yttKGQCL10Sg2gZYnqsgFwA52Anw/JYDAYDz/6wBLdXDOq0tjo9PPCvOrJj9dywwG/39ez2NrY3mjFqFDywNJ+p8XqevLAYnUoe1AnbbXHwTmUvZxfGSQbJf9vWyj92d/J5o5mnLiomO1bHb5bkoZALJI7VXkccHh7/uJH8hAgunJmMb6xmKYrgEyF3zMFkyOYmUqugpncU+5hTS1q0lt8tzaPf6iItWotPFFn1zyrMNjcPL8vnrII4TDoFaze1MjjqRS4TGHG4uXzdfspSI/nxoqxJG+LAX09OSEggISEBURQlh5DGxkZsNhsGg0Ei2K9TkzxREeZ/mjCXL1/O8uXLpf9/5pln+Pvf//6tLb/+S5hjCER1KSkpDA8Pf2c1g68aYQZGVlwuFxUVFSHnX5gbw4jTQ45JPC4BR6gVQbOJx0P7oA23x0OH2cZVi8on/QF5vD4eer8OhUwIGncZbyJ9PKSZdPz5Un+qsK+vj9raWklSMCBttXR6EmVpUSQcQ+3mZ2dP5dU9HVw1N9g7URAESlMM3H76FBIi1UFkGdCfXVaWRpdngKUlwenq/MQIf5OKELyeXq/n2kWFXLuIoFRZU1MTCoWC2XE+pk9PJWLCqItMELhidvhGKZVCdszZO6fHx/ZGMzPToyQy2Nk8xKpXqlArZHz6k7lBKd51O7uo6vfyzOetrEk/mhGYlxXN1roBqdkmgBsWpLOiNDGEyJVyGcfKgn885u+56uRMqS79/pE+1u5oR6OU8cUd86WozuPz+s3ARRFDZBRpibE8O3UKbrc7bLTU79WQFG0gIVLNntZhNtcMsLlmgKvmpKJSyLjlpAyiNMqgLubS1NAM0F+2tvBOZS9Huq1ckeNDEAQWZEezrX6Q0/OPbmqmpRjY12bB7vabf+9uHWbT4T421wxwwYwkTHoV68bS2oH79ONXD1HfZ+PRFQX86QdF9FiczBrboGiVctxekV0tQ8xIi/TP6Ar+e6pRythY1cP2Tg9ZJgNXzU3jhS/bOdQ5wuCoi8srUkJS/ZNBEAR0Oh06nY6UlBREUWRkZMTvZlNVhdfrxWg0SvXPY0WQJ3IO8/8Kamtruf/++9m+ffu3fq//JUygs7OT5uZmKaprb2//ztb+KqmS8apBk42sXD0vg6vnZbB///6vXRMdsLpQK2REaEI/bqvVynlJVs7MSeeMspxjfqEa+21s2N8JwKUVqZLA9MQoetTp4WcbDhNvUPHzc/LCCqI3NjbS1NVPSlZhiP4u+In1WJiZbgxrLi0IAjIBrpxApAHLtWnTphEZGcmz+RlBxwSECxQyvwxf17BDkkMbD7lcTqQxGkOUkSlTZDidTvbs2UNLSwvbmy3IVFqWlCSF7PQb+kbZ126RpN8C8PhE/rK1mTiDmovHaaY+/nEjr+ztYnamUeq61av9ouxRWiUTg5Fr56QwYh3hloXBTVFLSxLCjqYIgkCcQY3D7WXI7g6R4msfsvPoR40sLUlg0bha833v1mJ1eMiJ1XPxLP/1zs40UpgYweyxtGaAMFONWl65bgZKuSBFsu1Ddq556SC5cXqevNCvXmS32/m8upN7P2xEKYM158aRZYzm/JI4psQZpC7uaSmRPBSmNv7cF22s39XBb87LY25WNGcWxHGo28o5RfGIdr9TyNxsE2/fFLxpuP+9Or5sHuKK2ancsjCDiowozpuWQH5ChDRj2TPiYsOBbq6YnUqGScuI0z+Xand7KUmJJD1ag1cEheCf2b3ttUO4PT5+vWQqKUYNyVEaUqM1PLe9jVf2diIXRUozVMzONKJXyXh1Xzen5sV8q1lMQRCIjIwkMjKSzMxMvF4vQ2P2ZY2NjSFm5ON/jyei6edErPlN4Xa7ufTSS3nkkUdITw81Jv+6+F4R5sQHdyAFG6idjY/q/l1zUYEHeTjh9nD4uine5gEbV6/bi04l540bK1CPCx/cbjcHDx6koqwEg+H4AtA5cXqunJuOQiYE2VFNlBw80j3CZw0DAPxoUU5Q2s/tdlNZWYlGq+P+HU5s2w7wtx+WUZoW2tQhiiKPflSP2ebmvnPygq4d/EIEj31Uj1cUueMMf8Q7/lrazHZ+8241FQkC043uEMuz5gEbbx/oYllJPAmGo6/fsP4g1T2j/PrcqZxbHCwIYXF4WLZmNz5R5K0bZxGlVaNUKolLz2XNxp2AlaxYC9Gdnfh8PqlR45Z/1tIz4u+gvLziaNS5t3WYtTva8flEziqIk4yrs2N1eH0iOeNmOYuSDHz0o9l+A+gJ382CRD3/M0sf1Ek6Hgc6LNhc3iDza4ArXzhAfd8oj19QxMJx9bQ39nWzuWaA2t7RIMK8bl4aW+sGOXmcQEBylIa/Xz3BuRo/gTzxSRNur8jDy/LRKOU09vu7cPe2DfuJRuaPljJTk1DIezDpVSQlJjI8ZGZxvBWw0tjoCGrAeu9QLwc7LPzo5Ez0agWf1Q9itrnZ0zrM3Kxo5ueYmD9mgbd3b6t0r0adHlZvaaYwMYKlJYmUpBjY326hING/MdKrFdxz1pSg9/DM561sbzSjUcq48/QcnrqwiH6rS9pMrf60mXcP9fKbJXnMyohiaryeNrMDk06J1ycyMOqisW+UomQD8bUqZphkXHl6NlvrBznQbqEsNZI0Y6gwxbeBXC4nJiZG6kGYaEau0+mk7+WJiDD/L+EXv/gFRUVFXHzxxd/Jet8rwhyPQAp2vH9jAIHO1hO5SwqMbVit1uN6V45HYExFFEVGnd6wUWPQeUT/ULR3rB4TOHdNTU3YWu2w3c3hrhHKM4whllhymSA5hky8pvEkPj01iusXZBBvUAeRZSAlmp2dTVx8AtoPB7G7XZOqBPWOuHhhRxuC4B8ZmRhRNg3YeOFLfxp3WWmy385pHGFuPNjJ1roBDnfI2XrnySEPhvvfrWZ7wyDN/aM8eP5RezOFzL9GODEGm8uD1em3f7M4PFJaMlqnpCAxwj/IXphNpEYhWRn19vYy1eDBYoMEhSOo7b4oKYKKDKN/NGFco82FM5JZOi0hpEv4eOM5r+ztpLHPxk9OzZKOHRx1ce1LBxEEeOGKUgoSj3YwOj3+lKXLE7wJW16aSG3vaIiV11Vz0iRT54l4+2APncMOyjT+NbssTj6tG0TAL0yRlxDBgpxofnVOLqlGTVDNLjdez6ZVs1ErZKgVMmJMfmIfsjp4c28rOeYO1N5aNBoN92+y4PGJTEs2cG5xAr85L48dTeaQhqRt9YM09XmYPvbb3t5kZsOBHjZW9XLetASunZfONXODO+A9PpG7NxzB6vTy2MoCrpydikYhY9lY+j5KqyRKq6Tb4iBGr6Kh34bL46NtyE5FppFHVxRKm+0z8+P43Qf1/GJjLa9eN5M5aXpqa2uJ0at4amsL7WZ/U15mjJZHVxQes1nu20ClUpGYmCiZkdtsNsxmM/X19QwPD1NfX09MjF8O8dvOZH4TickThU8//ZTXX3+dvXv3fmdrfi8Js6Ojg5aWFqZNmxY2sgqQ0okiTIfDwcGDB4mJiaFkeilrv2hDrZBxxZzjj68EyOnpbc2s39nO/5yWw4qyyW1ssmP1rL+2HI1ShkbpF4I/cOAA8fHxaLXakFrpPRsOsatliJsXZnHN/GMbJgNSU8V4wlTKZdxycrAST3d3N42NjUH3/K2bZ2NzeUMcUQKIN6i4aWEm5lE305JDI6fMGB0XzUzBMy4SCxDm6Ogoad4uFuVEsmJWZthd9PLSJFoH7ZIWagBrLi2hd8QZtsklMVLD2sunI4oE/V0pF3jpquAoS6FQEBcXR1xcHH+aOhW73c7g4CD19fU4HA5pzu7JHxSErZkHCG+iXZjD7eXKFw7g9Ph48cpSDAFlHq/IbzfVI5f5zYoDsnx6tYLESDUjDk+IaPkLV5bSbXEyNT44/ZwWreXJi4I7TgNYv7uDdTvauX/JVGaPOdGMOj386r1aBOBHpSpmAalGDXecno3b45PWlwkCi4vCyzhGahQc6LAwbHNz0hQTgiDwwu4unt/Rw7RkA89fXo7dbueiaY0c6LCgGWqmutqvPrS0OBa792hmqLHfxs/eqsbj8XDGHDdJUXIqMoyclhdDYWKE9DtzeHy8f7iP0tRIMmN0DNnc7G71Cxa0DzmYmR4VYkH2cU0/D2yqpzwjil+dO5WaHmvQv9nRPMSBdgun58UQo1eRE6ujd8TJT18/RIzSzZ+mCVxWnsx7Vb30jDhJMKgxHmcj9F0hUJPX6/Wkpqayc+dOkpOTMZvNdHR04PP5pPpnVFTUN34G/idUi8bDbDZz9dVXs379+q+UPfuq+F4RptfrpbKyElEUwzbWBPBdmj5PRH9/PzU1NdLYRn2vlX/ubkcQBM4uSpDUUiZDIPrtGXbiE0X6RpzH/PfgN7GFo76ZBQUFmEwmurq6QlLPaSYdu1uHJDPgySCKIle/sJeGPhu/XmgkcZKdpc/nY9u+I2hFR4j4g0YpP+acpSAIIcQ7HnKZwL3nBBtfC4KA2Wymq6uLhbNKWHLq5GNB55UkcV5JEm63O4jw1QrZpB2hcHxJupZBO26vL8iVQxAEVBot+8xKipKnUGzSSM1D7+9r5E/7nJySFcHPz54SVGf6rGGQVa8c4tyiOH47JkZutrmp6xtFwD90HyBMlVzgiooU6vttQQ9wtULGOzfNkq5jPCI1ihCt1fGo6hzhvUO9XD0nVWpK2VjZS7fFySe1AxJh6lRyzpuWQMeQg+woJ4LgT4+Pr8tORLfFQe+Ii5KxNPKIw8ON6ysBeObSaZSkRDIjLYrX93czPztaanb5n8V+Ivf5fFgsFgYHB3l9ZyPPVblYkKHjl4unEKvXkWHS4rCNSmnuKK0yRND9jf3dPPFJE7lxev5+dRmxESoeOC8Pm9srkfzDHzRwpHuEh5YVkBip9svo4dc/jtYpQ5q3HtvcSLfFSYZJy7or/Jurw91Whh0eRmxevCKcXRiP1eFFpxS4YEby1zI4/y4hCAJGoxGj0UhWVhYej4ehoSH6+/tpaGhAoVBI9c+IiIjjEqHP5/uPkyXA008/TW9vLzfffHPQ6/fccw8XXXTRN173e0eYJpOJ5OTkY36oJ4IwfT4fjY2NmM1mZs2aJUlfZcbo+MHMFDQKWcjuPxwC0dzdZ+WydHoixWEir4kINNk89VkHqohoZlYEW3yN30X+7KxcfnJqDlrVsXeWHp9ITY8Vr0+k3+4LW1d1uVw8uXEn6w45mZcTQ8Ws8Lvoz+r7eWpLE2clepk7Se24a9jB4a4RTs6NCXm41PZYeW57Cz+sSMU6PIzP5wtKc9tdXjZWdlOeGR027eX1iRzptjI1Xh80ylLdbeXed2pYPj2BH1aEdrv2W11sqRsgxn00shkYdXHh3/YgAq9dNzNonvLtg908sKmeKK2ST26bQ3R0NNHR0WztVeHyNXOwx0lbWxtWq5WIiAhMJhNHOh0IQFWXVVonKUrDH1cW4vT6yI0PJuWfTiJgfryH2LDdzR8/bqI8wxgUcf/y3VrqxuYVA24nD5yXx8c1/awsOyptJwgCvz7XL6+3a9euIE3WW8fGK/566TQppSyKIhc/tw+728fjKwuZlx2NRikjJ05Hv9UlzXQuyDGx5Sdzw17zZw1mFHIZ87KziehTIZM3M+QS6O7uxmKx8LOZGkZGBHxuJ6LCXye0ODz8cmMtmTFabluUxfSUSBIM6qD67UlTTIw6PXhFkCHyUU0/DreXut5REiPVnJEfy9R4vTSOArCr2cybB3u4bn46l1ek8mWzmaKkCK77+0FcHh9/vqiYuxel4R4ZQCETuOONw+xoGkKtkFGYHCltGv7dmJhCVSgUxMbGSh6VTqeTwcFB2traGBkZQa/XS/XPgCDFePxf6ZC95557uOeee77zdb9XhKlWq0lJOb5Q+XdNmIIgSF6OM2fODEoPKuQybloYfig9HALpYq1KzowwXaITEWiyccvUbO8WAb9pcklqVFjCFAThuGQJ/rTruitn0mVxkOgbCPnhDQ8PU1VVhSk2FpmsC69v8trGuh1t7G8bRuWWc8UkhHn9S/toG7Rzz9lTubg8mLye2tLI+4d6aers57YyJbm5uUG1mOe/aOHJT5vIitWx8dbQh+8TnzTy0s4OFhfF88B5RyPWT2oHqOsbZf3uzrCE+av3avmsfpCKRDkL5/hfU8llqJVyPF4fWmUwsefG+Ql55gRXkkvLUzBqlcxIjyLDpEUURUkmrVRn5rpiBTPSIxgcHMRo9KvsBOYdJ4MoivRZXcRFqILuZ3W3lbvfqmb59ISgWuSH1f1sONjDu4d6gwjzsvJk/r6rk8Xj6oPZsTqyY8N3HHp9Its6PHhjh5iTFc2Iw8Oe1mEEAZoH7UxPObppio1Q0W52SLVbpVzG368KbR4Cv8F057CTU3L9qdqGvlHuetMv1PDmjbO4tDyFgqQIpsZHSBGzzWZjz969vLOjmgS1m8SYKBpGVexoMvNlyxA3nZRBcbKBd24uDznXLf+sIsOk5bkfTueRFQW0DNiZO2aZJggCKUYNu1uHKUyMIEqr5PkvO9jdMkSKUcstCzNYWpLAsN3NiMOLTxRxeHwY1DKqe/xSfjE6JQq5gFGrIPU42Zz/JNRqNUlJSSQlJUmlDrPZHKQnHNj4KZVKRkdH0el0DA0Ncd1111FVVYUgCDz33HPMnTuX1atX8+STT6JQKDj33HP5/e9/H3LOyY698847eeedd1CpVOTk5LB27VqMRiPNzc0UFBSQl+f/7c6ZM4enn376hNyP7xVhflV8l4RpNpuxWq0UFRWRnDx5eqq6e4RNh3q4cGaqlEINh3Bdsp83DNA97GBZaXJQo8rIyAiVlZVkZ2eTmJjIT70dmG1uisai0mN13HYPO7hp/X5y4vQ8srI4LIlNTYhgakIE9fVDQeu0t7fT1tZGWVkZ87Ra5uenHjPF+dPTp5Bi1DBdY560aWBqvN8SKiNMhHjh9DiaOvv5YXkSsWH2EGXpRvRqOQtywpOMUi5DFP1kNx4Xz0rG7vZyal744yoyjHzZPES+6ehxBo2C91dVIIqEDP9PS4nkyztDrbbVChnLS4/OhE6USSub7qG1e4C2br+Un1qtltJkI145G6t6OTUnuM72t+1tPLmlmQvKkvjF4lzp9U/qBmjqt/Hy7s4gwjx5ion306OYkxV8A1eUJrFiEpH0D4700dhv49p5aVJkvr3RzEtH3KyvOcTnP51HtE7JYysKGHZ4KEkOVmR6+eoynB5fiJ6x1enh07oB5mVFY9Kr8PhErh6zTHt0RQHzc0zEG9SkRmvQKGRE65TIZQKz0o00D/ibcGIjVOh0OrZ2y3inyU5ZaiQPFSch9PZzRrqMeC20tzRJtbphhz+CnJke5Xei8YkMWP0+qdNTIkO6j1/e3cnftrdRkWHkkRUF3LggnVSjhqUl8XzZZOZ379dz4cxk/nRhIV6f39nnlpdbsTrdZKSbkcnh5CnR/OrcvOM2cv1fgSAIREREEBERQVpaWlBKvLW1lZ///OdkZ2fj9XpZtWoVZ599Nq+99houlwubzcYnn3zCW2+9xcGDB1Gr1fT29oY9z2233RZyLMAZZ5zBgw8+6HcuuvtuHnzwQR5++GEAcnJy2L9//wm/B98rwvyqufXvgjBFUaS5uZne3l6MRiPR0cc2iP3rZ81sqxtAIZex6pTJ63YTSc7h9nLPhsN4fD7STTrKx2pKAR/JgGg7EGIDdizCbOj3uz50W5x4fGJYo+QAGs1udh/p5oZTouhuqZcsyAKRa278sXUl8xMN/GpJAV9++eWkhPnoBcVj1xFMan19fdDfyItXzyQqKoq6urqQNeZkmdj5s1MmPf+qkzNZUhwfQurROiW3nzb5Z3HF7FSumJ3Knj17gs6pPY7+rccn0jxgIztWJzXz+ESRa186SNewkxeunB6k5NNpcXPx+jo0Sjnvr6oAj0uasXvsy2H29Pj4rFbLT0qPPniH7G7kgoDZFuzMcsnMsU3ABMeXOIOav15WEvZ6Rxwe/rq9lTmZ0ZIxtcvj4+43q5EJkJcQwaKx9XLj9Zg0AgUpRknTd7Jo2C+S4P88x9fSH/+4iTcP+OuWf7qwGLkAhUkGGvpGpc/IoFHw2nUzg9Y70m3lhvUHUSvk/OvWcpRyGdFqAUEQSYhUExUVRVRUFPfn5uB2uxkaGpLGuh7Z66ZtROTHC1O5uCKDpy4qJt6gDnpmvLy7g31tFn52Zg7pJi1KmUB2rP96SlIimZZswCfCR9X9DNk9HGi3sKI0kd0tw0Rp5JQm66jvHUUQ4N3KPkRgSfEwp+WHygD+u/Bt6o0ymUyqf2ZnZ7N+/XpefPFFvvzySz7//HP6+/sZHh7mqquuIiYmhr/85S/87Gc/k8pRE83gASwWC1u3buX5558H/B2+gWzRmWeeKf27OXPm8Nprr33ja/+m+F4R5lfFtyVMt9tNVVUVGo2G8vJyDh48eNz1LpmVilIuY3HRsaXRJoq5qxUyzi6Kp9VsJzc+Ap/Px5EjR/B4PJKPZDj0W524fJMr9MzJMvHLJfkkR2mOKVEHsPqLfhrMToaHhrllYQbp6ekhP8Q9rUPcvH4/ZxcmcP/S8G4iHza7eburmnvPKQiJzgRBkEjb4/Uhlwk0NTUxMDAQVK/ss3lxK1yE68O0Ov2WTbMzo4MszQRBIMOkHVOnCTVL9vpE3j7YQ5pJw6wJafCt9QOs3mnnvkQr01KORnmiKHLnhiO0DNr5y8XTggbTf7mxlvcO9XLVnFRuG9OItbu87G+3IBP84zLjCdPp8eETx/7rE9FrtaSkpJCSksKFsh4a329gdpISi8XCnj17MJlMXDMjhkW5MSGuI0adkttPnXwTsKtliC11A1w3L10aCXqnsod1O9rZsL+bbbfPA/xKRcunJ1DXZ2N6ytFzJEaqeXCBhvLy0A7bYbubjVW9LMgxSTqyAKteqWJ3yzDPXlZCUbKBackGNlb2UDY2mysIAn+bhMwdbi9b6geZmRaFWiFDQEAzLhVenqjg8jPKiFAHf5/aht38ffcQF81MpSIvjymdh2i3mhGtg+za1YvBYKB+SI8lJprseP/7W/tFO1anh12tsZxVEMf8VdFBv42bdY5OAAAgAElEQVR736lhf7uFh5flkx6tZXpqJOt3dbB+dycnTzFx67xEmrrNyGQCerUCt9cn6ef+J/BNTCGOBZPJxLx589i9ezdGoxGDwcATTzzBvn37WLNmDbW1tWzbto17770XjUbDI488Qnl5cDq8sbGRuLg4rr76ag4cOMDMmTN54oknQqT2nnvuuaDmnaamJsrKyoiMjOSBBx7gpJNO+k7fWwDfO8KcOGQfDt+GMAO1u5ycHBITE6X1jvflnJUZzazMY0eh4N/Vud1HfQEFQeB/F/tz93a7nZ0795CUlER6ejptZjuC4A6JnBr6Rrlq3V7UgodXc72EU32UywSWTOJ0MhFnZGtwVjtZVpFDRkb4GnFNtxWn28ee1qGwf3d5fPyj2oFM1sup+Qmclh8X9t899Wkjf9naxFXTdJybbwyqCbcO2rjl7Q4UcoGP/ychSBYP/CMzn9b2c3F5KvcuzgtZ+6WdHfzhowZuXZgp6ZcCbK0f5IFNdYjAzrsWBBHqmm2tVA/6eHl3ZxBh2t0+PqruRyYTONJtDSuyPZ6W9WoFT19STO+Ii4qMYFLOjdfzz2vK0KnkIZZci4sSWFyUgMPhoLa2lvz8fP/sZ08XXouFOouOqGgT8bExQfrAr+7tZNPhPn517tSg78c9b9XQO+LEqFVy3Xz/PViQY+Ktgz2ckhv8Hu47J9RDM4C9bcPU942yfHqiRCp//byNl3a2815Vb5DQweEuK15RpHnQTlGyIawZtE8UeXpbKz5R5JaFGVJk/uz2NtbuaGd6SiTP/rCEt2+ahVopCyKyCLWcD6r7STCoJNeTv37eyqbDfZhtbh5bWchvzy/E4fGhVfp/q4da+7j7jToEsYX752lIiTexal4CTcM+FoxF2Zur+9nTNsytCzMx6pQ09ttweny8sa+bVrOdzBgduXF69Co5hUkG3B4vv9vWj1M0c/+5uRQlGUg8jnn2icSJ0JG12Wyo1Wr27t3L6tWrmT17NrfddhsPPfSQNJe8Y8cOdu3axYUXXkhjY2PwHKzHE/bY3/zmN9K/+e1vf4tCoeCyyy4DICkpidbWVmJiYtizZw/Lli3j0KFD39o4Ixy+d4T5VfBNCFMURdra2ujo6KC0tDRoR/R11vN4fdT0WMmJ04cduZgsjdrX18eHu6vJz80hIyOZ7mEHF/51FwLw9i1zgnQq3V4fiCJeEXzHuS6P18c/d3eQGq3l5KnBqaNA2rk00s6SFdlkZ0/eUHXBjGSMOsWkXb0qhYyVeRpGZAZmH0Nj9WCbv87Z41JRUBAcqcoEAUFAGmmYiNRoLT4RUozh66lHukf8hsI9I0Gv5yXoiY1QkRuvZ2Jm+vbTsvnzh1VcNTv4vetUch5bWUjHkENKYwbw6yVTuWZuKlmxwfXY2cfYMOXE6dnVMoTN5Q0r2fdx7SB9PW5KSo6KdPt8Pi56dg+NA43cN7eLZJ0ozdg9tbWFwVE37x/p47p5RzcHP6xI5u2DPZw8Lo2abtLyz2vDu8YE0DfiJFqvkjYTN79ciccnEqlRcHahP96fnxPNu1W9ISnItZdPp75vlFPGzimKIk980kzPiJP7zslFq5TTMmhn7Rd+kYqzCuKk7uD8xAjkMoFpyf60f4RGwcbKHjJjtBI57m4d5tfv1iIIAh/92K+UdNHMZAZHXVwyJu/n30j7dXHL06NIiDGi16jQqeTMKC3GbrWQ5xkk0TtMffUIJpOJ1Vs6Mds8lKVFcU5RPI+vLKR9yMFTW5pp7LdxqGuEIbubrBgdpamRjNrMdFu9ODwe/vftGrJiday5ZNp/rIZ5IsRZRkdHiYuLIzU1ldmz/f6nF1xwAQ899BCpqamsWLECQRCoqKhAJpPR399PXNzRzXFASGbisQGsW7eOjRs3snnzZuk3rlarpTTvzJkzycnJoba2llmzZn2n7w3+S5hhIZfLcTqPP98YgMfj4dChQ8jl8qDaXQBfR87u5V3tPLe9lTMK4qTI8VhriaJIQ0MDR9r7eeYwyKobef3GWFQKGUq5nzhUE5R08hMNrL+unI7mhhBN0onY2Wzmsc31yASBbXecJJG4x+OhqqoKlUpFZmam/zr6RhkYdVER5sGvUsg4p/jYEeuyPD35+VPRTmJq3d/fz8o0O2fn53DW9NAuzdRoLc+uSEen0UizieNx15m53HJydsjfAj+8e86awsLcmBD5uOQoDe9PYn48Mz2KVTN0ZMWEkvCpYSyqwK8klBOnx+XxMWhzBY0niKLIk1tacHl9/GRRltTEtbdtmFv+UQUCbPufuUGbqZoeKz9/twGfKHJmhYOkQNQiCLSa/TORxuQsZmQbJY3RS6bA3j4Fs0werFYrer0eQRAmVfIRRZGnP2tlYNTFXafnBH2nNh3u4563qqnIMLLm0mnSez/QYaEw8Wi6dm5WNJ/8ZE7I2hkmLclRajqHHaQaNVgcHl7a1YEALJ+eSEWmkbRoLeeXJOATRTLH3evT8mLZcUeM9Bluqx/koQ8akMsEtt3u74jONGlJiFSTatRIqlKlqZGsuTQ4zbtmWwv/2NvFgpxoHl1RyBvXz0QuE1DKZURo44iOieW2Vw9hHnXyi5NFlmXLqer1EOvuo6dHpMuh5L1D/Vw9Nw2Lw8MZ+bH8eWsLVqeHtw/2UB4Pl5ca+azdRW3PKMN2D8N2z3+MME+U8HpCQgJpaWnU1NSQl5fH5s2bKSwsJCcnh48//phTTjmF2tpaXC6XNL4SQGJiYthjATZt2sTDDz/Mli1bgkZX+vr6MJlMyOVyGhsbqaurIzt78rLDt8H3jjC/65RsQO4tIyNj0pGVr7NelE6JTAYmXfjaxvj0rsvlorKyEoPBwPxZZTxzaB9alQztmCDAv1bNA4GQLkTwK7lYu4+fKs5PNFCUFElGjE562ATMtdPT00lJSaGzsxOrzcEVz+3G6xNZc1nppCMvlR0WHvuonmvmpXNSbvCPJeB60jfipHfEKXXzBiLZvr4+Tp5bLu0mw8GkU6JUhn8ICIIgkaXPJ0o+mYGHbYRawVkF4VPBAYiiGNJ8JIoif9vejs0jctsEmyaLw8P/vl3N1Hg9Pz4leHzo8nX7qe0d5YkLCqXGmFazg2e/aEMAFhfGUTgmkhCjVyEIEBehCplDTYrSkBylBq9LGtIHf8T9j2tn0DHkkAb/Axqjubm5OBwOyaB4dHSUyMhITCYTdSNyntvRwU8WZUnzgQOjbtZ81opc8Ed45ePSxiMONzIBRsYkA4EQz9Dx8IkivWMKN4F7f9eGI2xtMHPX6dlcNDOZH52SSbfFKbmRKGQCPx/X7TseZpubdV+2s2hqDHkJepKj1OQnHhVQjzOo2XBDaLTxr0O9rPmslTtOz2ZBjonCZAOagz2UjH3v1Aq/bq1SLuPMgjhGnR6OdFv9DUp6E9eelUZtj5UYlQfr8BB//riL/X0ePPYRzilOpHPIzg0L0nl1bxfP72hnl0HOb85I5uSiSNrMdtJNuqA53X83TkSEGZB9XL16NZdddhkul4vs7GzWrl2LXq/nmmuuobi4GJVKxbp16xAEgc7OTq677jree+89gLDHAqxatQqn08kZZ5wBHB0f2bp1K/fddx8KhQK5XM7TTz8t+Yh+1/jeEeZXwVcluIDLyWQSewF8nQhzaUkSC6fEhhj4jl8rYDFVVVVFbm6u1G325s3+KCgQlRxPZ9bsEHlgQx2zs4f48anhB95NehUvXH20G3Gi6weMpUARSTFq6LY4j2lT9I/d7exoGsTrEyclzAue2cmIw8Pqi0uYnRElRbKzZs2SdsRPfdrI5w0DPLJyWtAYTmCNX2+spmvYwSMXFIdsGB7fXM/fPmvhgaUFLC9LDtlAPf5xI182D/HYykJpgD6A2149zLaGQZ6+pFhKoQ44RNZsb0MmCJyeHxs0grCzeYjPG8xsrRvkpgUZQZGZ3e1FEMA+Tsc11ahhWUkCTk+wKEGGSctnt89DLhNCNG4jNQpevXoadXV1IWn8rBgdcREqmgbsZE9IASNX8sgXZqI0Kn6+uBDrmEXUHz/ooMbs48+ba3hoaS6RkZHE6JX8sCKFQauLaRMaiS4oSyI/ISKsKMSuliG0SnlQ89HvP2zglb1d3DA/nZtO8ssvun0iAuDy+u/FpbOS2ddmwTvhs3nog3qqu608sqJQaqR6fX8363a082ndIG/dOIs3bjhqCh6A1enhDx81UpgYwUVj6kPvH+mj1Wznk9oBFuSYOKsgLmjD1DRg5+EPGkGA4mQDyVEaHltZwIjDS16Cnhd3drD2izbOyI/lf8/O5SeLY3l1byczE5X8clMzgujj1ycZKIqKpCBBS2mcgj9t72VvZysi8KOF6dKG6D+BE1HDHB0dxWAwUFpayu7du0P+/tJLL4W8lpycLJElMOmx9fX1Yc+5cuVKVq5c+S2u+qvjv4QZBscjzIDLicfjOabE3lddbyKUcoFNh3spS40KaQoQBAGLxcLw8DBlZWVBqQm5TKDNbKdvxElZWtRxW8ZrBlwc7rbROtQxKWEG4PJ4+funlcQrnZwxN1gsXiaTISDy6vUV+ETxmDJf187PwOsTuaQ8VAggEP0bdUosDg8K0cOuXbtIS0sjNTX436/9ohW7y8vnDQNB4zKCIDDi9PLKng5kAhxsH2behPnLyg4LggCHu0dYTihe3t2Jyyuyo8kcMoNY1+cfC2gZsEuEGaOVcX5xPDaPSH5C8AjNvOxolk9PIDdeH5Iaf+mqMjqHHOSPE0OXywR+dW74ZhqVQkb7kJ02s4M5mcagz9ft9bGu0kaupYUb5wd3KV/83D46hx08vrIwqDZ5uNvKJ7WD+ESRG0/KIHFs7OLnS2N4/otWLiqOpLu7m9raWrRaLZfkmzCZkkJIWRAEWgbtbK0fDGqWqumxcus/qwDYdGuFpBk84vAgjP03gEeWF9AyaJfk6FZ/2szLezpZlBvD75f7a9Uen8jr+7tBhAPtFqkWenJuDJ/UDrBszMasZdDOwx80cG5xnNQtvaNpiH8d6uWDI338YEYSMkHgztNzmDFWfwzg45p+/rm3i9tPzSLDpGVGeiRK+VEVrrgINU0DQ1idXnRjrjEBoY/iZANxEZl+YYKDQ0RplRTlZWMZHuLGIhlDw2aOeOQ4PV7cXpF/He4PEeH4d+JE6GXb7XaSksLP7f7/Af8lzDA4VkRos9k4cOAAKSkppKUdXywdvj5hvra3kzXbmpmdaeTxC4/WWbxeL42NjTidThYsWBDyZff6RK5/cR92t5dHLyhmVsaxu27npOsZ9siZPfXY6kdut5vVG3fxQpWddJOec08OThcHiE4mE/B6RTYd6qEwySD5ZY5Hdqyeh5YXhT1PYLf76vUVtHf30tVST3FxMVFRodZfDy8vZFfLEIuLg8dwBEEgQiXjnrOn0j3sCHsPHllZzI6mQU6Z6o8mfD5f0Bzg75fls6/dIjWrjMdzPyyhumc0SEpNJgjce1Z2WKcHnUo+aTdppEZBZGL4GdUXvmzn5d2dPHh+fpBJ8qXP7cfm9vLQ+fn/j73zjm+rPvf/+2gvW5blPeIdryR2EjuDQAgJM4ywN2GvUgi0FOjtbWlLW0Lb214u3FJKGWFeNgGSlBFIE0ZCpkeceMd7SpYtWVs6vz9knViWDIEmt/dX+LxevHhFlo7OOTrnPN/neT7P5xNhhlzX62Bzh5cPOzq4oCItIssPJ6SyKdfqnMx4Lq/OIEGjiLA3q8yK5z8vOjwWEna4sFqtPPJeHVs7Pdy5OJHKvFQSEhLwiQI/fbcJYSITC//qZn2INKNXySPGhO5fOZOL5mVQnn742DVKOWnxaj5qtHBcgYmUODWiSMR+KWQCD60qoWXIGcE6npmi58VJrNv3DwzxefsIvaNufloVuqYW5SVwRnkKpZO8LjMTNJw004xl3Ctlqy/s7GFv9xh/axhizUl5/OeFkdfqg++1sKtzlMExDzcdP4MTChNJntjHrS1WHtjURHVOgsQClgkCGq2WX/zdQuewnKoMNfcuUbOtfYwl6R5aW1sl8YT/bZutY8WSjSWZ96+Cb13APNIA5/f7o14fGBigpWX6h/h0mDoK8lWYkxVPulHDovzDD4VwoE5JSUEQhJgrQ5kA2YlaDlmcJBm+2r1do1Rw8RwzSSkJBIKx7azCakHzCtJ5s6WL+TnRxz15gfF2bT+/2thIWryaTbcfN+13N/SN8VmrlUursqTSsSAIBAIBero6GB4aorxiLnJF7F7uipIUTpqZjNMXuRAJl2SvXDi9WWyiXsXKWWmMOL24x+007K9HFEXi4uJCs2S5iSybGZuwk27UkG7U4PEHpfM1uS/+WZsVly/IiimEH18gyHNf9JBn1klD/hDKmm58oRbLuJd1qyulHuRre/vosbn5qHE4ImBmmTQ0DjgiAglAWZqB6nQl+RnJUWbE/3PdPCzj3qjxIoUslGXFgscf5Nd/ayYlTs1tS3Mkh4sP1/fSbxepG5GTa7LR3t6OXC7nlAI9/U6Risw4micmh5IMKj5aswiByPtOpZBRmRVPy9A4Zr1KOuafvNPI520jXDwvg3tPLeDs2SlRwvDLi5NYHs2FwxcI8ti2DlIMalbNSaVv1M2KmYkIY51AqD/9iymZ+7jHz9XP1hAIivz1ijkUpej5wfJ8NjYMSmXbgTEPv/uwjeMLTJxbkcbSokR6R91U5SQQEOH5L0LGCWtOysMfCAIhq7TGgXHerRvg0qoMkgwqRpw+rK4AG5vHaRqF25YWMztdT9DtYGhoiJaWFlQqFYmJiZhMJomEdSxxrAJmWCjlXxHfuoB5JJg6NxkMBmlqasLpdLJgwYIIx40j3Z7b7T7i98/OiOc/LpzFjIkHXLhvOGvWLNRqNTZb7FlGQRB4/PJKAmK0Ik4syGQyOqxuLnvpU7JNWp69Zr5EhIHDPdqwWtAnc2Jr3oaDFIT0UtUKmUTpnw4/fK2eDqsLUYQbT8gFQplMS0sLer2e8oq5rHx0Oy5vgLduXRRTLvCWF/exvX2EP11WwfGFZmlfwsHr48YhHvxbE3euKIhi6H5xaITrnt1Dll7glRurJB1Mi8VCXV0d3kCQV1sFUhP0/Oi04ogHy3M7uvndh23csCQ7gsgzMObhtpf3IxPgxWvnUjypPLu12cojWw4RFEV23nO8VJ61u0NmynKZQLvFiUkXOm8Priph88FhrlwQmf0/f00lXn8wqiyqUcq4o8rA7NmRBsgQluhTsaXJwoLchIhsz+b0ccv/1JEWr+YPF5RJ2de+7lHeqRskKIpcXpUhlVN/fXYxmw8Oc/WSGdJrHo+Hu9KtWK1WGuv24vF46O/vJzExEZVKRY/NzYF+B8tmmiVC1M4OG7e9XI9BrWDzHQsRBIGSVAOftY1QlBLKUeM1Cr7osJGVoI1wz9nWYuEPH7Vz+4m5EhN5X/cYz3/RgyjCGeXJ/GzlzAkBkciy9f0bmlArZPzk9CKUchlGrQK72y8JG5SlG9AoZZK+7WftI2xtsdA44ODcijQunpfBxfNCwbTD6mLD/iEALpqXzkkzzeQn6UiNV/OLDc1sbbGgUcpYvTCLG5ZkU9/SzTstboYdXta+38LppcmsWZ4vMUVdLhcjIyMSCSu8gAufx6ONY8GSnez1+q+I7wJmDEwuobpcLmpra0lJSaG4uPgbrfq+DukH4LGth1hf08tVC2ew2OzGbrdLajZer/dLtyWTCQQDIh8cGKQwWU9eUixZgsP7NeJy4w+I9NjcId9FBMlg2u12R/VofYHghKybXsqwZDKZFKTmZBnZfu+JX3mMqyrSeaumj8UFoSza6XQyODhIZmYmxcXF2N1+nN6QcLXLF7uc3WMLOXkMTrI4mxww19f00Wl18fre3oiAKYoiB1o7EUQRr6BGp9Ph9/uJj48nPj6evLw8vmi38PHm/QRFJxW6UbLMIfcQs9lM67ATuUygdcgZ8Z0JOhX5STqc3kDEqAjAnMw48sxaCpP1ETKDJp2Shy8sZ8TpY+6kTLI8PS6mjZhMENAo5fgCQRyeQAQrFkJEonfqBliYa4pQ07l/QxMfHhzmjLJkfjOJwXpwwEHT4DgH+h04PAEpo6vIjGdleTIpceqI76jOSYhgyMJhge7/3jlKfZ+cm4qVuN1u6uvrCQaD/PhTDxZXkHtPyeeSiX6zSiFDhAhVnu+fmMvNx8+QFntbW6zc+9ZBdCo5H605PI7ydt0gbcNO3qodkAJmeXocJxQmkhqnJl6jwOkNcN9bB8HjpqIy5CfabgkRfEQRrj8um6wELa9cP49AUJQWIOtrB/j95jYW5CTwhwvKWD7TTOvQOAtzQ8fcbXPx/Zf3U5SiZ+2qYq5dnIUMpID+7I5uhse9XFmdiVopcOasVH74xgH2do0yK1nJz06ZgT2g4PV9/ZRO+X21Wi1arZaMjBARzT5Bwgqfx6PhUzkZx4Il63K5olR5/pXwrQuYX6fnODQ0RFNTk+Qf+U3xtUk/stAQdW93J7LkNObNmyft95EE3w8ODvGLdw+SbtTw5i2x5wfD2ypLVvPoZXNIMqhRyGV4PB5qampISkqipKQk6nw99F4zb+7r5epFMySiUKx92nxwCJ1KzuL82OftlqV5kktL2KczPPIAIa3QN25ZiNsXe1AfYN0182gZGmfBpD7l5P29+5QichJ1nDuJuOP3+6mtrWVRlp5nr62K2WcFqMw2cdbsVExaJaeekCv18BoaGjg50UfR4gSWlqRGyRS+fuP8mNtLjlNL7M2pWDYzttaqzenj/g1NVGbFc+3iyNnIS5/aS9uwk79cPjsigD2zvZvHt3UwI1HL27cclh3LM+sIBMUosYTqnATWLMsjLV4VUf7UKOX8+pwS3DEWK0N2Dz9+u5EFOQkSyScoimxqCOmjHrIrWZmbS25uLn6/n+LmOj47ZMc71EVtrYXExESKEhPZ9L0F6FTyiN9s3fZu6nrt/PKsmRNarpBtmsJUPimXHJOWs+cc7l8r5QJL8k2UpYW8RJsGHezoGCUYCGJz+kjUqyhM1nHTkhmoFDIyJ8h0SrmMUZeXHYdsHJdvkgT4w4sao1bJ3ZPK1r02D2NuP/W9dkDgqgVZeCdYzv1jHtbXDiACZ85K5f6VMwmKIZ/SgCiyf8jLU18MUJVr4s+XzY6Sf5wMQRCkBVz4PE72qVQqlVL2+U3Lt99lmF8f37qAeSQQBAGn08mhQ4civCu/Kb5uwLx4TgLpvm4WzSkmdYpA8ZEEzMJkPWa9KioTmIrwtuZOrJ6HLVYaDx6guLg4aqBYOhZZ+P+Hb9Cps62NAw7ueSPEjtx0+3ERuqiT8dGBQZ75pIXz8mHl8VV0dHREbCcrQcOWpmH8ATFqNQ6QZFCTqFPRN+Ymw6iRFH7C5yfLpOWukw+XKMN94Nzc3AgmX6x+tUoh44GzDjfLwi4NM2bMCAWBkRG2N/dzsLkVreCjr6+PtLQ0tFotLl+AvzdbWJCTIJUtwwiKIlubrRQk6yJ6isMOL3e/0UBllpE7l4cWEttarfy9xcrfmy1csygr4qFoc4ZmH8fckfs+NysetVLOkinqQt9bmsM1i7KiHtJymcDVi2IzNbc0Wbjz9QZOKUnid+cdVlXa1mplV+coNT1jUsCUCQKPXDyLlqFxiuiV3qtQKHj0srmh6sXEfWW1WvmstpE/fuGgIl3LXctyMJlMyORyHtvWgQh83j7C6WUpbLlzcZTwf1aClkurMhh1HeYFbKgf5DfvtZKoU/L+7QuZnRHPtQsy8Dus0m8gE4SohQfAj948wIF+B7cvy+WK6kwqs+MjvGm7bS7ufesgVTOM3HlSHr88aybp8WqpjH77K/vJTdTy8IWlnFGWjNXpY1GukVteqmPM7ecHy3NZWpDIKzsPMTjuY2uLlVNKk7/SiHwypvpUTp2h/Sbl22AweNRLvd/1ML9l8Hg81NbWAlBVVXVUGu9fFuREUcQXEEMlKlGks7OT3c09vNOtxWvycVFK9La+SnihKMXAhu/HNt2NtV+iKLL27X28Vmfh52dNHywB7jl1JpdXZ0c87KceX1q8mmyTDr1KTsI0KiaBQIDf/m0/HaMBymfM4Dy1Oirwbm8f4YevhdRttt9zYtRYBsD97x7krX293LG8gBuPz436vdqHx7nlxRrmZeg4K308JmHrtb29dFqc3HZiblTv95NWK2/u6+euFXlkTUjqKRQK6qwCP982QoJGye+X6hFFUfIIfK1dxvttTuZnG3nyyoqI7W2oH+T+DU3EaxRsmWSM/Hn7CPt6xtjbPcaak0LHcWKRmdNKkqjIio86rpevn0ffqJvZU2ynFuWZYlqIAbQMjfPSrl5uXDIjaibznjcPsP2QjScuny31XjtHXMgEaBt2Rrz3lJJk9vc5mD8j8jwelx9yM9m5sy/qu5/4pJPWYSc/W1lEVlYWuywK+sabsRxyc/3YGB0dHchkMm6cn0D3uEzSa1UrZHzePoJWebgv7g+KXPLUHpzeAH++bDYVmfGUp8eRYlCxpCD0OblMYHV1Ggebx6VgDaH77f4NTXRaXfzxwnJMOiXFqXqaB8fJM2vxBYI8/XkXaoWMu5bnI5cJ7O9z0DbsxDru467lIZGDMKzjPrz+YKg9IJPxq3NK8PiDeP0BBu1eLONefry+kTSjhvOLtaSnpWP3CxH97W8CjUZDRkbGP1S+PVYZ5ncB81sCq9XKgQOhDKupqemosdS+LMP8+bsH2d83xq/OLsEzeAiFQoGYmEPjnlYUDYNRllxHCqc3wIa6fuZkGae9OWUymSRxd3DAgSCT02H7cjavLxDkiU8OkRav4bZleQiCEBXEjVqlJKIQCy6Xi5qaGm5dnM62bp80kzl1O1kJGjRKGblmXZSDSBgOtx9BEHBMZFqTM0wIuaR0WZ0Mjjr5+dknRFUL7G4/v9jQhFwmUJ2TwJKCyBLyg++10DniJiNBww8nWX0ZtUpEMdSDVCgUpKenS5TI5RgAACAASURBVF6ATf5D/K3FiVlwsG/fPsxmM4mJieh0OrJNGoIiUQFrRXEStT1jzM44HBzjNQoeOi+2s0uSQRXBhp26iNrTNUrrkJPzKtOkc/fbD1qp6bEjAmunKPF82jaC0xugoc8hXS+XV2eSk6ilbMroS5xGEeGxORmv7unlv/7u4lcJFmnm0+MP8tgnncgEOKM8hZNmmjmtLJn+MQ9zMuMpCFuGeb1kWK0h4tW+3cTHxTEq6PnBhh5EYMOt1SQZVAiELNRc3gCaiUVUUYqet26uQjEpG63vc3DHezYqG+r582UhyT63P8gHB4cRxVAlZFGeiftOLeTukwtQyASaB8fZtH8IBLisKjM0elJkxrEiX5oRFUWRn7zTSMuQk4cvLOP355eSbFAhEwRqe8a4762DLMpL4OGLynllTy+v7+2nrsdOt0Vg/U1GEuKObsnym5Zvj6XSz78qvnUBM1YQFEWR9vZ2hoaGmD9/PhqNhqampqP2nV+WYfba3Li9AbbvqeOUyjwyMzPJ8waQCQJzvoJp+mXYUNfP2veayEvS89pNC2K+x+cLlRILCgr44xXF7Ose5bhpeo5h1PWMsWn/AABXLcrGqFVGBakwuqxOtrePsHJWquSyEe5XlpWVschkYtWk90/dTnaijs/u+XIC0drzymgccEgyepN/32AwSK7MwhWzDZw8ryhmad2glnN+ZTpdI66obA1CRJTX9vZx3hT3jPkzjHy8ZhE6lZzGAw1SwJLL5Vx5XAGXLQplJ263G4vFQltbGy6Xi/j4eDZdV0xSYmTJVKeS85PTi/AFghEZEcADm5ppGRrnPycyosl4+ON2Pmm18puV+dKxi6LIDS/UgghxGrk0T3rDcdk88VmXJDg+GU9cPpuGfgdnzT5c0lDIBE4oTKS2x45erYgq5z67o5svOmz8fOVMKXhv3D/EqCfkCRkOmGqFjLtOyqV12CmRZ7RKOdU5CTQOOKjOMaKUy1CpVKSlpfGzjwap7Qnw6PlJGPzj6ORB1HIY7OlAlmzGaDTywjWVuHxBiVx1oN/BDS/UUpJq4MkrQ7PLg3YvItA14pL2WauU89CqEnpHPVLLom/Uzb1vHWRudjx3LMvj6kVZqBSykNwgoX6mViWnZyKjD4iwq2MUbyDIIauLIbuXAbuHdKOGIYcXXyBI67CTH68/iFmn4K7luTy7o5s8Q0ASOTiWONLyrc/nO+oZZiAQ+NpTBP8/4VsXMKcirMeq1+uprq4+JsPDX5Zh/mBJMvuaDrFy8WypVKhTyTlvbvRD7etgbraRwmQDp5bF1kYdHh6mpaWFhIQEZswI9aHMehX1vXaqvqT3WZFl5JL5maTGayTR6OnKxHe/sZ+G3jGGHB5uXZpHR0cHAwMD0qIE4IOGQX7ztybuXJHPnPhonV9/IMjj2w6REqeOmW2rlXLmZBnxBYIE/KIUdN1uNzU1NaSnp/NvFbMZtHsZ9/ij7LEEQeCXZ5dM+/A4ozyF08uScfmiFwRGrZLWoXE6x/xMlXoO93hFuVLyrgwGg/QNj3Dra03ICXDvAi0pSSGik8Fg4JDVxUV/3UN6vJq3bq5CLhPw+IO8uqcPuUxgT2e02fCLO3tw+4Ns7xhlbtzhYzqhIJH6Pjulk8TPl81Mmna+tCw9DqVcxiGLK0KSb932bh75+yHmZhmlQBTGI1sO4QsE2dpikRSRfnnWTJ76YC+3nRQ5gnR1DEH3779Sj9cfxKhVcuasw4G6ZchJUBSxemWcWprPR8WhzH1kZITh4WFqDrRw/3YPMpmMF1fPIj0xnkG7J9TSmBQcT8iL457FRo6viBzcjNcq6LC68AWCyGVyanvtNA466La5uWt5PhfNTWd/n10yTu8ccfPQ+61ASJghK0HLwxeV0zfqxqRT8u/vNIbOYVocFZlx/O78Ug5ZXPxyYzP+oEjrsIubT8ghw9f7vy5OANHlW4fDgcViYXh4GJvNRlJS0lFl3/4r41sdMMN6rIWFhaSmRivGHK3B3lh+mOHZTpfTyYUrFh71VVlhioHfXzgL3ZR5vXA2PTw8TGlpKQMDoWxxyO7hey/VgAgvXF8VUxcUQmSYe06LHACfLsNcWmim1+ZmblY8dXV1yGSyqEXJhweHGLB7eG//EBXHGaMC5u5OG3/ZdoggcHJpMqYYovQOj58zH/0cjz/Ic5cX4/F42L17NyUlJZjNZmq6R7n6md2YDSo+XLMkZpXhuS962NY6wq/OnklafCQr867XG9jSbOWPF5RFiA4M2j1c+tRegsEgL+W6mTmlFPXKnl4e2NTCFdUZ3HdqITKZDFtARceoHxFIzZ2Jyuegs7MTh8NBh1uNKIr0jXkkIQm1QsYDZ82k3eKM6an52/NK2dU5yumlZga6HdLrD18UW01pyO7hjx+1c3JJUoSbSt+om8ue3gvA27dUSRq68RoFQZGY2sY/Pq2ALzpGI0QashI0rMxTRoknAHzcNMzzX/RwzykFFKcaOK00md2do8zJjCS/PHNVBYesLo6f6Ef6AkEe29pJSryaS+fPJD7NTeDz3fj9Is2th+hu9ZBsMHD/yRmUZEUuEGelqEnUK3H5Amgn7oV73jzIiNNHvFbBqjlpLCsy8/0TcyVZw5++28SerlFuXBJybkmLV1OVY0QuCKRMCILYnF56R93Mn2FkYW4COqWcdTu6OGRxhca5zFpWlifxaZsNq9PHY9s6uaRAZN4/IWBOhiAIxMXFERcXh9PpJDMzE7/fj8ViobW1FYVCIbUQvi779qu4Ff8K+FYGzDC5pre3N0qPNYxwVng0AmZYMD0Mt9tNbW0tSUlJ33i286vQOjTOjc/vRaeS88bNC1EpQv3Kuro6NBoNVVVVOBwO6SI3qBVkJmjw+oNRZb+p6LA4uf3lWqpyEvjZmSXTlpxvW5bPdQvTqampwTQhJTgVPzylkNI0A6eWpeIfG4y66coz4lmYZyIlXj0tgcjjC4bYoiJ0DVhgbJhFixZJv2uY9u/1BxFFiHW6//JpFw6Pn82NFq6ojsxku0bcyIRQUJkMtUKGRinD6xfRxnBIaR92opAJtE4izZSlGSSPx5ykOCCO9PR0RFGk1G5HpepB7nVQV7MXk8mE2WzmnDmp9Iy68fiDUcSnE4vMnFhkxumMJOb02Nxc8cxechK1PHNVhXSNra8b4N36QT5rH4kImCGHGxmISIEF4IK56ZxQmIhZHx0Az56dSlq8Okpyb9AZ5KzHdjIvO55fTmIaP7atk4P9Dl7Z08dPzyiaVjM326Shw+qix+Ym26Rlb9cYz30R6mOeUZZMWryGZ6+uRBAE8pN0BINBxsbG0Fit1NQfoFUlkJduRqVS4Q+KXPTXPdhcPp67upJsk5azZqfwWdsIcydaHmqFjNkZcQQnLr2CJB01PWPSHKtKLnDtomySDCqJnPezd5tw+4Nkm7SsXpBFvFbBuu3djHsDbKgfZNzrZ1GuiScun817B4Y4MDBOinb8n5JhTodgMIhSqcRoNErjXB6PB6vVSkdHBw6HA4PBIPU/j3Ra4FgrFP0z8a0LmIFAgJqaGpRKZUzvyjDCAfNoZH6TL6AwsSic/XxTTNY+jQWlXJAyFDhsQ5abm0tGRqjcOznQaVVyXrkxdq9zKvb3jdE14sLm8kkBM9bq0mq1sv6zemocBtbEyI4AUuLUXHNcDsGgSPcYUYHXoFbwlyvnxvxsGGaDiv+5vormtnb0AQeG1NSIRVB1ronXb15Iol4ZoWQ0GT87o5DP2m0RQtxhPH7ZbIkgMhlGrZIPbl/IgQMHMcXIwO5cnk91TkIEm1QQBCoy42kadETIEYaJG+UFcg70OygrNmEftdHb28vbnzfwx70+4jVy/nbr4XJ2GP/29kHePzDEvQsNhBVg24ad2N1+anvsEyzs0PecVprMp60jrCyPzMQSdEo237EIURSjVITMehXvHRiiKFkfUa7909YO1u3opnrGYR9MURRpHQ0JYQzaPREB80cn5/PKnj6umqRetL/PzuaDw1yxIFMKyh82DvOTtxvRqxR8fOciZmXEcXxhIqlxh2dFC5L19I66GbKH3HESEhI45JDx0O5+tEoZz16kZXCgH4ttjGG7QBAB65iTbJOWW47P4axZqdJ8Z/+YmztebQDgxWsruXN5HredmCMxpnd1jnLf+oPoVXLW3xxizp8/N52GPnuoP/t6A2qFjOeursDhCfD4Jx181GihrneMq5+roSrbyH9dVM7uXbv+TwWTWOLrYRGK8CLO4XBI88d+v19i3yYkJER99rsM818QoiiSlpZGWtqXmxl/3dnJI/ne9vZ2BgcHI3p43wThEu+X9RtmJOp47aYFKOUyRixDtLa2RtmQTc0MP2kZ5sG/NXPTCbmsqpjeceCU0hTGPQGKJ9iTU0uy4Qy+v7+f9/s01PRYMcd18dMzY3skdlqdXPLXnaTq5fzxzNgzgaIosqVpmOQ4NbMyIsk5Pp8Pe3cjGcY40CQheKKlA1Pj1Gyo7+f4QjOZCdHi0AVJOlQKuSSJNhlJBhUGtZHeUXeUHmtdr521n45yu9LOstJINumg3cPLu/tweAKcM2nI/rrnaxhz+3H5glFkoutfqMXu9uMPFnJ+ZTopKSk4dDaEfXWIIjQ0NBAMBjGZTFLfaVfnKP6ASPvo4ev1+AITvzyrmMwEdURWmm3S8vRVkaMuEBLuv+u1/VjGffzl8tkRpsYb9w/y8w1NaJVyPvnhYX3gdKOaYFAkIyHSAHthugJzerbEKg2jPD2Ok2Z6I7b9s3ebaBkaJyCK3LU81AnOMGqQCQKFyaFFj04l5w/nlzIwQeIRCBF5Ln96L3KZwNu3VBOvUaBVyhAAvVpBWloqapUSg2GEJ4tM9A5ZEUe62DnYxpMHRGoGvNy1PI+L52cSr1GSHq8mKIqYtEpqesb4xYYmLpibzhXVmZh0SuSCQLpRgyiK/GFzG0PjPh46t5QRpw+VIuRm0jfq4RebmjmhwMQfLizj1d19fNg4TH2/A5vzyLWk/7fwVS2nyeXbnJwcAoGAZEDe1taGQqGQsk+DwYDb7Uaj0WCz2bjhhhuor69HEASeeuopFi9ezCOPPMKjjz6KQqHgzDPP5Le//W3Ud073WavVyiWXXMKhQ4fIzc3llVdewWQyIYoia9asYePGjeh0Op555hnmzZt3zM7Zty5gKpXKrwyW8PXl7L4Mfr8fl8uF2+0+KsSicIn3ywKmyxvgvjf343aOc/McNdXV1VHZ8tRj3Npiodvm4qODQ18aMJVyGRfOy6BxwMGoyxfxAAwEAjQ0NCAIAtXV1dxmtvLcjm4urprexqh31I3XH6TLFrI9ioUvDk3MZAKf/mipxDYMZ875+flc8kIzNmcP/3Z8AhMm7RIe2dLGczu6qMiM56UbqqduntXP1uALiDw4xQUkjGufq6FxYJyHzivhlJLD2dmzO7ppsvp5flc/y0ojz9nG/YN83j5C89B4RMCcnRHH9nYbhVP9KYH52fF81m6jOOVw8K3OSeCtm6swapXEaxT4/X5GRkYkjeEfzlPR5dIzN/GwiIEgCMzJjGNv1xhlaXFRpdxBu4etLVZOK00mTqNg1OXjs3YbMiFEupmcFRclh6zJKrMjFyoXz8vg9LIU4tSR16FCJnDF/ExEUYyohPzhozbe2NfPotwE/nTp7IltpPM/u3s5eVJ5uDw9jk9+eByT9Qqe/LyLx7d1cuHcdO49tQC5LCRSIZcJkhtLcaqBDd+rRqOUI5McdGQUZpoxxhmwjnuZnaLjhbY6RNFDV2cHe2WDmM1mnrxkptSz23nIRveE8P0V1ZkUJut54ZpKEnRKxr1BNu4fIiiGrN4qMuN59fp5yGUCHzYO02Nz89fPunjui17uOyWf8ow4shM0JOpVtEb92v9cfN2Wk1wuj1DjCpdvOzs7ueuuuzAYDAQCAW688UZOP/10XnvtNbxeL06nk48//pj169dTW1uLWq1mcHAw5nesWbMm6rMAa9euZcWKFdx3332sXbuWtWvX8tBDD7Fp0yaam5tpbm5mx44d3HrrrezYseMfPznT4FsXMI+0JKJQKI5Khhl2+1AqlZSWxp6p+7o4kmDeO+JgX4cFQSZjxsx5MUvLU7dzywl55Jn10zIpJ+OjxmF+/NZ+Zph0vHZzqJTrdrvZt28fGRkZkvXZkgIzyXEaSUg+Fhbmmnj44jkInjEUQuyAmWXSolXJyTZppTLz4OAgLS0tzJ49G4PBgErRiiBADH0DFuQm8PqeXkmkfSpmpuhp6HdEybCF4Q+KIIQyscn4/om5+Fzj3HBcNKt51ZxUGgfGOX0KU/mRi2dNW1L/44WxyTrZJi1tw066RlyUp8eRnJxMcnIyoijicrnY19xFffcIAd8XUvb5vVfa6LK5GR73ct0UhZt73zrInq5RGvrs/GzlTBL1Kn57bklI03ZKYCxJM/D53dOLIWxuHOb6xdkk6lXScTm9AS54Yjduf4A3bqzCpFNSnh7HG/v6IwyoL5mfITmDTEan1cUz27u4ZH4G5elxE/O2YPeEFgUZRg3rJ5jEkw3CO60uNuwf5JpF2cgn9sUfFLnimb14fEEeubic3104h75RD9kmjfTQb21r5+X6MYx6NRfOTcdwUi6LJ3xUNzcO86tNzZxWlsx9pxby49MKsIz7mDWh1NNjc/Hy7j5OL0vm9hNzefD9FuxuP/+9rYPHL5sToen7fwnhBcU3xeTy7aZNm9i4cSO//vWveffdd2lra6OhoYEf/OAHZGVl8dhjj3HfffdJfdCUlOjWx9jYGFu3buWZZ54BQKVSSUpE69evZ8uWLQBcffXVLFu2jIceeoj169ezevVqBEFg0aJF2Gw2+vr6jpkn57cuYB4pphJ1vgkmu32E1YOOBr4qYI6OjtJ+sI57lueQbDaREaMEGWs7iXoVc7ON2N1+0r9iBFQ/kVXETfSU/H4/u3fvpqysDJPpcK/vzb19/GpTI3OzjTy5OnapRBAEji8009/vY3x8POZ7MhO0fPqjpUDoRm9ra8NisVBVVSXdVG/dspB+yyhOS2/U51eUpLDr36Jv0jCevGI2gkwWRWAJ45mrKugf80Tp2hanGri+0kBiDKJUWryGP1xQFgq2MY65bYIUNGPSA3Xc4+f2V/aHAth5JdL+2Jw+LnlyDyLw0rVzpV6iIAioNVrufG8If0DkkYsLSEwI9Y+ztV56bCJJMhfj4+PodDopSC/MTaC2Zywikzy1NJn+MTfWcV8Uy/WZ7V28WTPA2lUllE4SMvj3dxrpHnGjU8q57cRcKWCOe/wMObwIAljHvZh0Ss4sT6EkVR8lpPFfW9r54tAovz+/RGIoP/FpJxvqB+kb9fDEFXP4/rI8VhQnRXzWpFMyaPdQ1zPGojwTcpnAg++30tDvQKWQceUsQ0hYQ4BEnZL+MQ9xGgUyQeCTVismnZIzylPIyMhgRIhjS28touhmRaETtcPGvtpeAjOS6B4SCYow5PACofGcup4x3L4AerWC+zc0U9MzxqdtVk4sSuL6xdl82jZCglbxlQS6fya+igfxdSCXyyksLCQzMxO9Xs/MmTN5++23GRwc5PHHH6epqYlt27bxk5/8BI1Gw+9//3uqqyMrPW1tbSQnJ3PttddSU1PD/Pnzefjhh9Hr9QwMDEhBMD09XcpQe3p6IsiEWVlZ9PT0fBcwjyamSrDFwj/SwwwGgxw8eBCv1yu5fRzrMZUwenp62FbfzhMNIjNTHTy2MNruKYypAbPX5ubG5/chAK/etIDU+OlZcYvyEvnb7cdhUMvp7OzE4/FwwgknRPVm1RPsUbXyq+e7Hv20n0/bbfxldSpFKbHViTbW9bG38RCrSo3Mnz8/4nzq1QrUShkfdbsoKglEkVcgJOjgC4qsmpMW8bAIPVhD/548gjB5272jY2xvH+GSqkxJPcflC7DmfSu+4AgvXz8vSsHno8Zh7nq9gXNmp/LA2YcJMF0jLi5+cg8CsOm2BVKAauh3sK97jIAoYh0vkF5XKWQY1HJcvqC0SAlDJkCSXknfmIckgxqz2YDZbOZPRUW4XC6p5+RyuTAajSQmJnLD4kxuOSEnYjtdIy4ueGI3CpnA376/MEKM/eXdffSMuvm4aTgiYF5Rnckb+/qj/D+T49T89YrZ+AKitMj4t7cPsqXZys1Lsrnx+MPf/eLOXrz+INvbbZw70dO9ZH4GfWMerpnQuZUL0DnipmfUHWHsfev/1NM94uLHpxVybkUaVy3M5JXdfawsSyEYdExY2Lm4bWkuVTPiMWhCPcpHt3YAIovyTJh0Smam6DmjPAW9Sk5SWhr3bbYgAH/K11BlHkVVLpCf5KOnp4cPO3w880Ufi/JMPLiqhBMKTHSNuDCoFTz3RTcKmcB/XFDGsqJvTur7/xEulwu1Ws2ePXt45JFHeP7551mzZg1r166V2gjbt29n586dXHzxxbS1tUXcg36/X/rswoULpc8+8MAD035nrOf4sSRWfSsD5pHgmwbM8MB8amoqpaWlUS4jx2JMBSKDdE5hCeL+gwxPrIinw9SFg0EtJ16jQCaA/ggUSQbH3Kz9qJ7TC3TodLqYRKYzZ6cxNzsh5lzeVGxpG8Xm9LO3czRmwByy2bnnjf3IBIHl84pjnsufvNvCrs4xLEIr950eObbQZXVy35sNCEJoyHxmDMnAl3b1sPb9VlYvyOSHk1wqfIEgd7y6H0GAjARtxDxm+BQGY9y8zUPjyASBgwORmbNGIUMhE5DJhAhx8XnZRm45IQezPnKWUaeS8973FxKMwWIVBIGXVpfT0nYoIgMTRZFndg0yZPdw36nlKGSh6oPVauVg6yEe2O5GqVDw1GWlpCSGMs2w2fPUR86Dq0rY0mThsqrIkZsrqjMjxnAmZy3Ng+PU9dopSTUQp1GglIfY1MopNfO1q4qp6bFzaunh0vWczDi+f2IOhRP2dO0WF7/YGFLfKk+Pk8hXuYlaemxuyV7rtNJkZiRomZGoZdRiB+CG52vx+IP8ZlUJSwsTKUrWMz87HrNeJc2XKmQCJ5ckkagL9Yn1agVqhYys9FRGXYl8VteCR68nIxjENzZEwO9D5R9neHiYM8qSSE/QIAPu39CMKIq8UzvwrQuYDoeDpKQksrKyWLgwJI154YUXsnbtWrKysjj//PMRBIEFCxYgk8kYHh4mOfnwb56VlRXzswCpqalSqbWvr08q6WZlZdHV1SVto7u7W5oCOBb4LmBOg28SMCfLvk0uS07e3mRvyW+KqZlhOEj3BuJ4Zp+bKxe6+dPlFSQbvnxuaupKTK9W8F+XzCHbpEGl+PKA6Xa7+enru2mwBNAbTZyS6Jz2vRkJGhoHHMSpFTGNoMN4cGUeO9sGWVURTcoaGRmhcf9+lhclMuwKRCjYTEZJqp6dHaPMTI22BEuJU1OZbcQbCEYYEk/GIYsLuUygzeKKeF0hE1hRbKZ5yBmhrapVynn4tCQSzMkUxrAhu25xNvlJOiqnyO4lx6n54PaFocXJpB6cXCZw0/Ez8PqjJfJUChkb9w/iCwQ5Z3ZqxO9X2+vgoa2j3KUZlrK9EaePP2/rQCYInFaWzMJcEyZT6D9ZQjrDH+9GxEddcwdxgpu4uDieviiPpERTVBZbmRWPIByeaZ2MZ3d0817DEL8+p5h0g1xaiD34XisioR71mbNSeODsYr63NIesKed+aZGZBJ0yonT9xr5+HvqglcIkHS9dN48Mo5o5mfHIhBDjOYzfn18qmRcAvLSrl0e2HKI6x8iPj09EJpNRkmbgQL9D+l6dSs5D55biCwSl81vXa+e+tw4iE2DjbQt4/cb5CBO/x+bGYXZ3jtJpdXHNkmouSU7nlCoPBsHNyMgIv95ykJqhIHIBLpuXSkaigXkzItWy/i+OXBztTMzlcpGcnEx2djaNjY0UFxezefNmysrKKCgo4KOPPmLZsmU0NTXh9XqjTB7S0tJifhbgnHPOYd26ddx3332sW7eOVatWSa8/+uijXHrppezYsQOj0XjMyrHwLQ2YR7skO7WnFmvA92iybsPbaugbY0tDL0XyYebNLmVfwxhDjtDNfdH86Vmp0+E/PmjmzX19XL8khxuOz532fSMjIzQ0NHDVohxerrVybmUG450j077/QJ+da9btQSkX+GDNkmn1NHMSNdR0CFidPtKNh9/T1dVFT08PVVVVHHecmjG3n4RpekNrluVycqqb6rmxZfSeuzbar3Lyg+Ou5Xkcl29iXrYx6j2/OacEpzcQ9d1GjQyjRs6uThuVWcYIoXilXMYpJckEpzBGIdT/dfsCfNZmZV62Ucoc63vtrH52H6WpBl649vAMateIi5+8HZJhK02LixjbeKN2iI6xAOt2dEsB06RTcvXCLAbsHiqmBOw8s44/XFCGIITED0RRZGxsjL7BYZ76sJaZiTLm5iZjNpuJi4vj07YR7nq9Aa1Szt/vXBRxHE9v78Y67mVLs4XL5qZI5+t7S3Oo6RmTrMYUMoGsBA31vXbyk3TSQuGNff089H4rM1P00vEmG1QgIlnDaZRy/nzZbPyBSPGGR//ewfrafn53Xilzs43ETWwzTqNAFEWcPpFz5qTy67NnkjChEuX0Brjwr7vx+oM8f00lafEaUuLUGNRy0uI1qOQynvysi783W3jg7GJOKjIzMOahIiseXyDI9c/XMOjw8vMzZ7KsqJDlVg17PmzDF4TXagb541IbviEb/UGzZLd1tKpLRwthBvPRRFh4/ZFHHuGKK67A6/WSn5/P008/jV6v57rrrmPWrFmoVCrWrVuHIAj09vZyww03sHHjRoCYnwW47777uPjii3nyySeZMWMGr776KgArV65k48aNFBYWotPppPcfK3wrA+aRQC6X4/V+eUkTQjOAYS3aqqqqaW+KoznXGd7Wr99ppHnIyd2nFGE2m7ligZFcs4552V/ugzkdjuT2CSskzZs3D61Wy+LiTOI0CvZ0Tv8ZppuC1AAAIABJREFUnUqOTBYSIZBPIxwA8OyuAV7aM8KOgXqeu7ZKKjP7fD6qq6uRy+WseaWWjxuH+e355Zxenhq1DUEQeL5+nKca9/G7C2ZFMCghpID0o9frWTkrNeaiQKOUc2KRmd5Rd0T2J4oi5z+xm4ExD09fVcGsSUxPQRD4xfsdbO+0s3pBluRnGcaoy8eqx3ehkAm8dXNVxD49sKmZjfuHOKMsmd9MOIj0j3kQBIEOa2SWmxKnZnZGHL5AMCpLu2lxBqLHyc2T/D8FQeAHK/IZdfmixkq6bS5e2NXDygkGryAIGI1G3jpo538avSQbVDxfpqO7uxu73Y7FF5q5TNRFPzJ+ddZMtjRbWDUnDVEMSAtSo1bB6WXJEQuMV/f28dsP2ihPN7BudSUAJq0SEUjUH37fiUVmNq9ZhGGCXBYIilzy5B4G7R7Wra6UesXbWiyMOH3U9IwxN9vIOXNSWZSXgFmvoq+3h2f2WNncamdFSRK/OadE2pYvIBIIIo0x9Y26uXtFPstmmhEEgfcODNFjc1PXO0ZCQSLxGqVkOj3uDTDm9vPizm76Rt302dxcXpXBtlYr1y3OZkFFmqTXGrbbio+PJxAI/J8JnP8oQzYWnE4ner2eyspKdu3aFfX3559/Puq1jIwMKVgC037WbDazefPmqNcFQeC///u//8E9P3J8FzCnwZEEuLGxMerr68nPz/9fF0Job29nWbaKtMQ4FheFVvUGtYLTyqKDyJHithPzWVGcHFVOglCPNDw0Hw5eezpt3PZSDTlmHXeUTc+6yzHr+OCOJagUspielmHMzYxjfe0QSwrMeL1eampqSEpKiugFD9k9CAJYpunPOjwBPmj3oJD7qOkeZUlBZB9pa/MwB/rtDDk8UsC0Wq3YbDbMZjM6nY5dnaPc8lIdSQYV730/1E8RmTBrFkLatVORoA1prsZiRY44fdg9ARBF7G5/RMBMi1cTCIqkGw9XJVYUm/mvC8si2LMQknB79urKmMedHq+iwKSIskF7/8AQ97x1kBMLEyP0ZTftH2JHu23CAuxwCWt+thGTXsXJJcmkpqaSmpoqKb4Upg4zPmpl9+7dEcIJSwoSJdZu+Bpo6Hfw0AdtiKLI/BkJkqtInFqBKBJBKFpRksQHMyJJRq1D49z1egNLCxO5++QCgqLIsMNLUCTCOPq355Wyu3OUlRPi7Y0DDv79nUbOmpXKSRlBSpI1bOsYj8iw4zQKnr+6Em8gJG1nd/u58/UGRBH+FKdmVkYcvzq7mP19dk4rTeaxbR28vLuPnR2JrD23hF+eNZM39/VTNcPIn7Z10m/3oJIJ/Pmy2ZJxQXjgP2y3NTg4yNDQELt27UKtVkt6rVqt9p+i/nOsvDD1+ui2xL8SvguY0+CrAlx3dzddXV1UVFQc0UVytEqyTqeTnp4ekpKSuHHhrK/+wNfATS/spdvm5pFL5lAxyVos3CNNS0tjxowZ0g3u8QcRCYkkhLOK6W5+pVzg0S1t5Jp1nD+NE8uS/AQeOCmRhWWJ7Ny5k5kzZ0aQAgAeu7ySpgEH82MEdQCjTsXqWVoU8SlU55ii/n7+3AwG7R6WFoXKluGMOTk5mdbWVtxuN52eUKCaPHMpEwRev2EeQw4vZenR/dMfLcvkrpOLSI6LLsfnmnU8ftksSS1mMm5flsdVC7IisrDQ/GoiQ3YPbcPOCObt+weG+N2Hbdy9Io/TJrFF36638GTNOG+21PHRmkXS64N2DzIhlLVOxjmzU2keHOe00sg+0qyMODbcWs3kuBtWfEkMKnmvU2RlmZl4hVcSTrAHVfx4qx2FTMZb14ccTfLMOsrTDRjUCsyTMsczylOYP8NI4iRtWn9Q5PcftuINiPz67GJUChn7usfosbnZ1DDE3ScXoJTLeOHaSqzjPuZMBL9AUOSJTzsZcfqkWdfP20doHXbydt0Ay9KTOK04gdVLI91KAD5qGuajRgs/W1lEhlHDnIw4LOM+qbctlwmIIgRFWJJvYschG8tnmvnT1g7equnnlJIkqnKMXOnO4OEth4CQ12YsKBQKTCYTw8PDzJkzB6fTidVqpaWlBbfbLTGXTSbTUeE4HAmOlRfmPyL3+f8DvpUB80hWdNMFzEAgwIEDBwgEAlRXVx/xBX40MsyhoSGamppIS0s7Jq7m4fLU5EAR7lfG0r6dm23kz5dXkpek42Dtni/tiWxvH+G5HV0IwFmz02Jmms/tGuCJz4ZZUD/Cf15eHfMYPf4g62v68PiD04oQrMhRUV6RT6zqr1Gr5J5Ti7A5vZI+5vz58wkEAmRnZxMMBsmz2ojXyFH7Ig2glXIlL+7qZWFuAmfPPpzJh68nrUrO5oPDLMpLiCDyfHhwiLXvt7LmpDzmTumN2t1+7nnrAFkJGn56RlHEYmTV47vwBkTWXVVB+UQJ+J26AQbGPLxTPxgRMEtTdWgUAovzIhcSl1dnUpSij1AOAkiNV/Pb80qjmL3DDi+rHt+FUi7wzi3VEeSfR/9+iHfqBqnpGWPd6kpJOKGlbwTE/QQCAer370crF3HZbTx+aTlKRWQZfmuLhXvfOsil8zNYM2EB1jfq5v0DwwC0W5wUpxo4c1YKbn+QOZNK35lGDTXddty+IAtyExh1+djcaAGgechJZVY8F1SmExRhUW4CosfCuC/I1c/uI06t4D8vKpcy8Bd39tI35uHz9hEunpfB7cvyMOuVUoXg/g0hyb7//KiNTJOGp66swKBW8NsPWnH5gqyv7eflPX2kxat5bnUFviBUZEX7qYYxuQSq04VY5VlZWQSDQYm53NHRgUwmk0T34+Lijln2eSxKwy6X61/aPBq+pQHzSBArwDmdTmprayOUbI4U/0iGGSYVWa1WqqurGRwcPGoEovD2B+1eHjinFL1KTnZi6KIPk23C/crJCARFLv3rTizjXp5aPU+aM51u1To328jxBYkUTsisxdoHm82GKEJSUtK0C4I39vTyxt5ednbYeO+O46L+LggCrSN+bvrdVvKTdLx+88Ko9/zhgyae/KyLKytN/HjVPERRlH5rQRC48fU2emxu1q2uIN+klKyP3mtx8O5BP+8fGGJlWVLUsf5iYzObG4c5oyyZX59zWDf3bw1DDNq9bKgfjAi0ALU9Y+zqGGXHIRt3Ls+XypIyIcSK9QYCEWMY955aQGmaIUJqD6AsTc/vliewaG7kKI1MEDBqlLxV289Fc9MjAvm+7lGuf6GO4/ISeOTiULXC5QvgCwTxBwW8gchrbNWcNOp77Vw873CFQBAEijISeefWBSjkAqqgh+bmZlp6hvjBe/vRKmT8+dxsMlKT0Wq11HSP4fEF+aLjsN5vVoKG25fl4vEHpdKuRinn+IJEemwuqXKxu2uUBzY1A7D5joUk6lX89IxCbC6/ZBEWp1FQkRlP36iHPFWQAbuf1qEQg3vU5ZME3u8/s4gd7TZOL0thT9cod79xgHiNgjdvmo8gCJxWkoTN6aNv1E2HxUX3iBu5TKAoWcddy3J4cVcvbRYX1nEfCoWcspQvrzJNF6DCATLMqvd6vVitVql3rNfrpQXbkbqFHAmORUl2fHz8u5LstxVTA2Y4uysvLych4euTar5phjmZVBQe1JfJQlZdRwOCIOBw+7hm3W68gSBPr55HMBiMyKKnC4JObwBRBLcv+JULgoGJMug5c6J7vWHbsVXFOuamyDhtSWx5OIDTylP4omMk5nbCxzPmCe1H/6gnqkzscDioa+tFLhNwyXRRjGkRGB4PKdSMuv1oNHGSAXRarptm5wFKTAJ79uxBqVRiNpvx+/0Eg0EKk3W8f0CMGi+5++QCipL1nDk7WmloQW4CVy3MJMOoiejhKeUyNn5vAS5fIMJaKytBy60n5NA76okYO9nWauPH749QXl/D89dEurvc9XoDvaNufAGRG5fMkF5vt7hAFDnQf9hHM9uk5YVr5qKQC1GWXgtyE3j6qgrJxWMyNjcNs6F+kJ+smIFarUaflIUfC3afiF8UpPLjicnxmJamc+Ik3V1BEIjTKAg6faGZViE007p63T68gSAPnVvK0sJEchN1ZCZoSItXS0zrM2elYnP68E84stjdfu54dT8A/36Ciaq8OO4/swi9Si4dz7YWC/+15RDfOyEnNHOpkiMIhz0/e0fd9I15+OVZRdT22EnUKSlO1XPeX3bTbnGilMt4+qo5tA05MajlFCV/dVZ1pBmdSqWSzCFEUWR8fByLxRLhFmI2m/9hs+djUZJ1uVzfBcxvK8JqOqIo0tLSgs1mo7q6WpJh+7r4JhlmWId2KqnoaMj2heEXBXyBAGqFjKAIYsDPzp07SU1NJScnZ9osWi4TeOn6akbdPvKT9Ozu//JRnTtfqaN7xBXKTKsPj7w4nU5qamrIycnBLmr468e1WFXdrF40I+Z2arrHOL7AzNlzYs9aCYLA7CQZf71qLhlGTcT+hxc9v7t4Ls0jgZh9UJkg8NoN8xi0e6VeWRgp8Roev6KSoBg6frfbjcViwWazMTIywrJkM2dfX0JqUmSpOC1ezcI8E82DTjKn7NOoy8+g3RuhrxpGQ7+dl3f3cdvSnAhD78c/7eTPWzs4tyJN8pR0+gIIQoj0NBVnlCXzdt0AC3Mjj/ec2anEqxVRM6s2l48PDw5z0/EzIoJm14iLi/66B7VCxqbbFqCbNB709OfdDNo9/L11hBNTBIpTDTx+2Sy0SvmEmMKMkGl69xD793ejcVsYTlSRmJiIoInnN39rQRCgMtsYmvkk5CDTNDROxgQhKsmg4sFVJaRP8uDc32fn5hfrSDdqeOX6uWhVcmZnxDE87iVFJ2PcJ/J522iEPu4HB4dpt7j4oHGYolQ9z3/RzRllyVx/XKhqtL52gDdq+vng4BBapZzbT8xl0O5lbnY87RYnoigy4vTFnBeeDt+kBCoIAgaDAYPBILmFjIyMMDw8TGtrq7RgS0xMjJA9PBIcK9LPdyXZf0EcaQ/T6/Wye/dujEYjVVVV/1A/4etmmP39/bS1tUVZcsE3L+8GgyIiSD2l/lE3P//MSWpDPc9cPR+HfZSOproj8uocdni47aVaMhM0/P6CWV+5TycVJ/F+wyBzJvV5wkIPs2bNwmg08synbTRb/TzzeWfMgGkd9/Lzdw8AAhXZxpgBLyy2/dH+AYxaJbedGOqTdXR0MDg4GMqYFUo+2dnC9vYR7lxeELWNUZefV/f0oVXKI/wfAW5/ZT/bD9n46xVzqMyKJzMzE5fLRUJCAi6/yM2vNiOIjfz7cQbSU5JCGaig5PoXapEBf7l8dkQf8+26ATbuDxk6n1wSSXD63QdtNPQ70CplEb6S454AMkGICI6nzDSh8thYOm+KTQtwx0l53HFSXtTrcplAeYaBz9pGSNQppXLtz95tomfUTYJWya1LD0vY+QKh68cXFKN6n784cyZbmiycVWbGYeknKIq8trefYYeX/zi/FL1agUwm49X9Y2xsGafHFc8Tx5ditVoZGuxmbjK4RTkmwYnPp0WpVPKDFXmYdCqJiLO5cZh/f6eR3EQtL10X0iV2egMRzGW5AD9ckU+2SUPXoTa+6BpnQ/0Af2+xSOXw7y3NpTBZzyklSTz5WRdv1w1iUMk5Lj+R4/JVnFGWzCGLk9YhJ1anj4c+bMXh9nNmeQpPX1mB2x9gcV40oezLEAwG/+F+pFwuJykpSRr4nyx76HQ6iY+Px2w2YzKZvtLH91iRfr7LML+lsNvt2O12KisrYyrrf13I5XI8Hs9Xvi8YDNLU1ITT6YxpyRXe1tcNmL5AkO+9VMOoy8djl1ViNqgY9wbwBUIZxfBgH0N9PcydO/eIVomHLE46rCH3DI8/GOWJORlNAw68/iB/uXIuuWZdhF/mZKGHM8tTqG/r4YIl0Q99gAStktPKUrGM/z/2zjs6rvra/p87RVOkkUa9995syZbcwLRgbDCJ6SSUQAg9gSQPkvALL4SXBiGQAiQkgZhQEkpMC7YpNjYYMMbdqlbvbaQZzUjT2/39MZprjYot23IeL7DXYi0zM7pz29zzPefss7d7moB3EIIg0G728dL+PkTgsspkhrtbEQRBmpM9PDjO87t7EMUAazZjikPJ77d38FHbKGNOb8goBkCTISBx12UKkEyC3ykIAk6Zhn6rDxGIT89B5rHS2tqKze4gLUKG2SWSqAu9nquL4/mkfZTziqc7xNy6MoNnP+2bJkd38+kZnJ4bTeUkJrMI7Ox10uTo5nvnZIcoBLWP2PnGc4coT9Hx+JWhzOq7XmmkftBKh9HBXV8K+FFetzSNN2oGp9mc5cRp2XDjIlQKWchojMfnZ3enmbwELXqNAiuBRcdb9QZkgkDjkJWqicXNVxYk0jBg5erqVMntYmuPj/RkNTdXx2EbG6Wmpo/DJh+PHXChCVOw5Y6lKOQy1AoZokiINGB1pp711ywkNlyJIAhsODDAb7d1sDRLz63lCpZm6lhdEi8JUXzaMcp9m5q5vDKZ5Cg1F5YnUNs/TnFigNX7223t7Ou2cP8FAQLWgMXFfRubcHlF3j08wpkFcSGyiHPFqZh71Gg0UrvA7/cH5mWNRkkqbjJ5aOp3nwrSzxcB83MIURQlsotWq52XYAlzywqDs4cxMTFUVlbOuiI9kQzT5fUzOObC6/NjcXqIjQgjNz6cu5ZGEK5SYLOMsmTJkjmvOhel67lvbREJujA0YXJkMtmsJdnH3m9n2+Fhxp1efrGumIaGgLv9VG9Qh1fE5xenybIdOW6BX10ye38TAsErK0rGheWJ6MJkdB2umVZezo0PZ93CZJRyYUKTNLTP+fWlaYy7fFyzZLpa0PprFtA6bOeMvJhp7+UnhPPLdUUoZQLZCVFAlPQwe7ogYLzb11KPYaKUFhsbS0qUhievXjDjsZyVH4teEzA3DsLu9vHlJ/bg9Ph59ebFpEyMqXSanLzV5kRo72PdgsSQPmrLsA2ry8uebsu0nu7SLD0twzYqJ2X+Vy5OZmVeTMhsaBBRGiWb6gyszIsmbcIFZ0+Xmef39OEXRVZmlCEIAtFaJfeuycdoc4cE9oWpkfz2shJJP9fh8fHoxFjG2QWxnJ4bCNrqPjOyQ3VEKPzs3bMbnU5HTkwM/7xhAUn6I8fm8fl57eAgaqWMO87KIkwuQwTC5AJOj58nPxogKTqcdRMkqcZBK2NOL3t7LIy9186eLjMPXVwsWXDtaDVhtHn4weuHcXn9lCbr+OaKdPotTuxun0QuOl6casECmUxGVFQUUVGBc+3xeBgdHaW/v5/x8XE0Gg0xMTHExsaiVquP6ad7IvgiYP6HYrZA5PP5qK+vlwSC59OI9FglWYvFQl1d3Yyzh1NxIj3MCJWCx65cgMPtI2dC0NrlcqHzW0mNSaWgoGDOJSNRFPnBq/W0Dlt54qoKaZ9mC+JXVaUy7vRw0YIE9uzZM22eM4i/7+nj7Q43Q2818Y9vTjd5Dn73r95podNk56GLS4nUTM/AlTKBe85JZ8eeGnJyCqedT6VcxrfOzMbq8iKXCfin2G8ty46mKlM/7XUIkGJaDDb2dptZmhUoy00mDpWn6NjTZQ5xPJHJZLzf7eKZXaPcv7aAongVRqNRIsI80wR7Btz88cpSqibNjr5RM8TP3mohI0bDazdXAYGZRacnoDPrcB+5BzKi1ZyVoSImJiak3wlwbmEc7rUF5MRN73PNVK599P1Onvm0l6uqUrn73JyQ9/7wQScv7evnrfpISURhYWoky7P1pOrVaBQCYxPnI0IlJ0EXETJW0jPq4KvrD6CUC2y8rZoIlYJbTsug0+QIkSMsTdXzyk1VxIYrUcplWK1WXt/XzWOftrAmS8F1VYnExsbS75Dx2qFBIFBNWFuWQE6chsLECDbvqmNn1ziy7nG+uTwdvVbJFYuTSY5SUZEWxW0v1jJiddNssBImF/jDB11cvDCR2PAwXtrXz7DVzTuNw+xoNfHuHaHCCseLf7fCj1KpJCEhgYSEhIBEoN0utUA8Hg8KhQKtVjuvgdPhcJyScbfPEj6XAXMm2Gw2ampqSE9PJy3t+HVYj4WjBZSgCMJcy6En2sOc/CA1m83U19ej0+lISUk5PsKAX2RP1yhun59Oo53kCSLLTBlm+4iN9Z90s7YoGu9g81H7o2vLktjZ1M81S9NnfB8CmfKLe3uBwFhEUIBgMjweDw/8az+bOnxc7R3lh6tDA6bD7WPdE5/i8fl5/obFFCVGSCQImUyG0+Pjor/sw+r0suGmRZJHI0DDwDg/fP0wIvDOt5dMY5J+b0MDzQYbvaNOvnVmlvT6S/sG6DA6eKveQOWa/JBS2o92BfZl277DKEY1UvYZVA+a/B2RagUv3FAZYpsFAZ3W09LVZGXFT1P7aRqy8n6LcZr1GMCfP+zi+T19PLiuiNNyA1mz3e1DYGZFo+XZejbXGzhzkhOHWinnooVJ5MeHIwgBBabDQzb++82A7u3m25dMc6sRJv4DuG5ZGiabJ4REtOHAAL/e2s5lFUl8f1UuOp0Oo18DggwTOiIjIxkcHMRitnB2hgJ9hJYYlcj9m5rZ1mzkv9fkkauXc/WiRJJjItBrlZjtHr759xo0ShlnFcTx4EXFtA3bOCs/lv/3xmG2HB4hpl3Jt8/M4qqqVLRhcn7xTitZMZqQfTsR/G9K4gmCQHh4OOHh4WRkZODz+WhpacHhcLB//34UCoVEHgoPDz/hXqvb7T5m7/T/Or4ImMDQ0BBtbW2UlpZKJY35xkwZ5mS5ueMph56satDkAN3e3n5c26rps/Bpu4mHLy3D4vBIWdZs+/Re4zCftJvoHhrltduWHXVBUJqi4+riMM6cIQgGoVbKuf/CIrpMDpZlh5ZFg/OqbrebiJh0hM5+zHbPtG3IZQLaMDnjTjHADp7Yb5/Ph8/nw+70YrZ78IsiFoeXpElk2ZQoNQm6MHRqhST0PXmxUJ0ZRYfRTvkUhu39awt4q8HAtdWhZV6ZTMZTVy+kbkKGzedxS9mn1unk8TWx5KXGS5nAQ1vaeGFvPz+7sCBEfL3X7OLBnRaEXXW8dvNiqVwK8McdXXzQasLl8U/rYW5vMTLu9LKr0ywFzLvPzeHCsgSKkqZnC0uyonniq+WUJh957+2GYf5nczNRGiX/vLYQQRBI1avJjNEQrlKEZGZpejXPXVcRQjL61kv1HOob46GLiqRAbBgP9PsHx4/0/W9ZmUlpio6qDD3RWiXx8fE89XE3Y/5Rri7Q0dTURNeAFb9PZNBkwYWHgTGBCxYEroXZ4cHi8DDmELC7fQxYnAyOufD5RVaXxLO/20J5qo7fbe/A7vZxdXWqJI14spgP0s98QS6Xo9Fo0Ov1JCUl4XK5MBqNdHZ2YrPZAqpOMTGScPzx4LOgk3sq8bkOmH6/n5aWFqxWK1VVVTPeHPPlSj41YDocDg4dOkRycvKM5cljbevEWLJHPDODKkVbOhyMt3dz1+rikKH22fDTTU20D9v44ep8rqw6konP1MMURZFynZ3zslRceXrRMbPnVw8O8OBuJ1sNtTx1beWMn7G7fTQOjlOUpAsRQPD5fNTV1aFUKtFqtfxgWQHnlyVROoOMXZhCxuZvL8fh9qCbEPdWqVT4/X78fj9RWhnPXFOGze0jJ0aFzxeQ/pPJZOi1Sh69vJS4iLAZBRjuPjeXu8+dzry1urz4/OKMM4y1/eNsOTxCVYaepEi1lH0+9XEXj2/t4rJiJ6vTOlEqlRzqDsyIto2E2qnpNQoiVTLCwsLQTylT37AiHafXzw0rpmfuv764mA9bTSGCCp1GBz97u5ULyxL4+tLQasud/6znYO8Y3zsnW/LBzIrRIBMESpIipN9LpFrBP29cjNcvhmS8v3inlTdqhvjvNflSX9HhCWS0Ds+Re/qm0zKoztSHXL/XDg7yVsMwuXHhRGuViKLIM5/24vb66atIYXVFFn8sdFPTZSBGsPHTbWMcNoPgd3PP6gKyJhxa/CJEquX8/O1WbG4vz+3uJTtWyzVLUllTEs8PXj9Mj8mB3TN/2s+ngvRzMpjMklWpVKSkpJCSkoIoihJ5KCgcHyQPRUZGznoMn0X7slOBz2XAFAQBl8tFTU0N0dHRLFq0aMaAFcya5qPGPzkDO5pv5ly3dbw9TJfLxaFDh4iPj5fEzD0+P5uabfhwcP6Ccaqzpu9Lj8nOd16upSRZx8/XlXDFohTebTSwdEp2N5Ul6/F4eH3Hfv5a6+SK6kyW5syeNQYRMSHMHa6a/Xx/1Grkpb19CILAVxYkS/OQkxcfO3fu5M6XaqjpHeOZ6xeRN0UWzu/3c89rdbSN2PnLVQtInCi5BkuyAPVDDrYeHuZHq3NJ0gXsmXw+Hx+3j/KDN5qJjwhj0+1Lpu3fzvZR/mdzM99cns4Vi48o4vz4zSYGxlzoNcoQ8QCA37zXjsnuoTItkm8sPxLUukedyGQCY2iori7G4XBwr26YXa0GCtUGmps9kgZpeJiMX5+jp6ioKGTh0zZs4xdvt7KqKG6aZVkQnUY7fWanlFHuaDXSNGRl3OmdFjAjVHKpPxlEcXIE//hGBZkxWsYsRxR8nv20l0ff7wwJroMWFwIBIYsgnvhqGf0WlyQA4BdF7vxnPT2jTp66eoFUDn15/wDdow52to+SE6dlzOnlR6vzaDbYWDmRHTt9sLXdwcq8WNYV2UgwyDkvT0draysj4w6erPdjcsK3zszkysXJbGsaodfspG5gnEN943zUNsr6axbQa3ZJ85/zAb/f/2/TiZ0LZpvDFASByMhIIiMjyc7Oxuv1Mjo6yuDgIM3NzajVaok8NFX9K/j3/8n47FzBfyOcTid79+6lsLBwmonpZASzwvkImHK5HK/XS0dHBwaDgcWLF6NWz26mfDQcb0k2SCiaerxKuYxrK/SM+ZQhYuuT0TPqwGhzc6AnwLC8oiqNK6qm93gnZ5hWq5WamhoGvZEMWu1saRzmhtOyjrqP7zQYaB6ycv8KNReePbseziTPAAAgAElEQVSo/JKsaM7Ij6MkWYdcJkjHNrU3erDXgtPjp33EHhIwRVHE5fHxQYsRkUCvLTFy+nX4w45ORqxutreMcv3yDCn7VCsVARamQobH45EeOsFjf69phMExFxvrDCEB85olqbxZa5hxJOHuc3PYeniEC0pDGdn3nJfHlwrjqM7UM+708lH7OBnR0VzzpTSahsaJVnkZHTWxs7aVTwf8bO3xonpvN2/eVk3URJb5SYeZtmEbFoeHW1dmTvvuP3/Uzb9qhmg22PjbhN3WZZXJjDm9IX1KgAM9FipSI/neOdlkxhypFjz+QSfPfdrHZZVJ3FQVIz00Dw8FFISaho4oCT14URENA9YQIYGX9w/wr5ohfvGVQkqTdXh8IrV944gE7r+g08n9awv4pGOUryxIpGnIyq0v1pEQEcYLN1Qy5vBitLnZ2T7Kv2oHeadxGK3cz28uLaE8PRBM32kw0GJqwieKbNrXjiATqE7UcfmCNLrHPGyqG8Hh8eETmbHfezL4rNh6BTHXREChUBAfHy/pBgdnP5ubm3G5XERFRWEwGCSjZ7PZzI033khdXR2CILB+/XqWL1/OY489xuOPP45CoWDt2rU89NBD074rKysLnU6HXC5HoVBINl9XXnklTU1N0vb1ej0HDx6ks7OT4uJiCgsDM8rLli3jT3/603ydopnPxynd+mcUKpVqTqo982nJ5ff7GRsbQ6vVThunOF4cT8Ds6+uju7t7VkLRaZkR6HS6WW23lmXH8LOvFJOqP7oNUTDDHB4epqWlhdKyMjJ8CuJj9DNmrpMhiiL3/asRj8/P9SXyGcuWEHjw3r2hlvNLE7n1jGxJ3GHqsVlcIg9fWobV5Q0RAwhqxipk8PiV5fSanZyeO308BODeNflsax5h7USpMph9npYfz6bblqBTy5HLBXw+H1arFY1Gg8fj4dbT00mIULKqODT4FSYGxnemPohFUaTf4qIwKYJ4Xej96PH52VRnoNfs5Iy8GLpNDjw+kef29LGlcZhrl6Ty3XPyMSnj8Bm78fotiB4vu/bsJzspYL/15bI4xp0els4yaH9JRRLNBhtXTxqhUSlkxIWHMfVq/9crjVicHiI1ipCA6Z0Q7ff6Q02yf7Q6j3MK4lg2IQg/YHFy3bOHSI9Wh4zSbK4z0D3qYHenmdJkHSqFjD99rZyhcRdVGYGFnMPjY1PdEBkxGiLVCjqMARUuq9uH3y/yi3daMTs83LginS8VxlHTN4bZ5qXT5KR8ImlfkRPDFYtTiI8I462GYVoGbOzsM5OmG+MnS5WUL9OSFBeDAj8wvyMXn7WAeSJKP4IgzCgc//e//5277rqL0dFRzjnnHNauXcvLL7+M1+vFbrezfft23njjDWpqalCpVBgMhlm/Y/v27dOSmJdeekn691133RXCM8nNzeXgwYPHdRwng89lwJTJZHNqZs9XwLTZbBw6dAiFQiGtxE4GR5t5DMLv99PU1ITL5Tqqq8qxgq9MJszIRJ0KQRAYHBzE4/FQVVXFw+918GbtIPeeXxjC5pwKt9dPr9nB9cvTqe0fpyDaOutn93aO0mlysLFuiFWpPsbGxliyZEnIsY07vfxkpwPZ7jpevqka2UT/LBgsRVHkud29/GlHFz++oGBWQ2ulXEZpsm6av+XhQSv/taGO80riuePMLBoaGggPDychIQG/349hzEm3yYHD5cbnUwVK3364/cU6AOIjwkKCdJfJwZ8+7EIQBM4piAsh8uxoNbH18Ajbmo1cuSiZ80sTiNIoONhrQQRJT7UyLQqdPIUzUwVKCvNJ16swm80B2b6ODmK8AgaDB3usMmRh0TAwzhM7urjzrKyQfXqrYZjfbu8gIkzOjv86InC/tiyej9pGpzmufOfsbC4sTyQnTotl1ARA67CNm/9Ry+KMKEkAodfsxOr20TRkk7RfAX7xlUI+7TRz8YTUnNXlZf0nPaTp1ZwzkZEf6BljY13gQXtpRRJyQeDhi4vJitUilwnEaJV0jNj46eYWrl+WxtXVqeyuOSx9d/uInXtebwz0pWO1XFudwov7+mkYtJIeF0l1dZnUu6upqQGQSo/z4RryWSL9wPwo/QSF43/605/y3e9+l8suu4zOzk7a29tZuHAh69evp7q6mieeeIJ77rlHEig50dl2URR5+eWX2bZt20nt98ngcxkw54r5CJhBz8CysjLq6+vnac+OjqAAQmxsLEVFRUf9oc6HT6fP52NwcBCZXEFsVjEKhRKjLSCkbbLNbPQcxP97vZ6PWk384Lx8/vi1HHbu3DnrZy+qSMbr86FzDePz+WbsPQsCkq1XUO0m6HQfJO7s7rTg8PgmjIenG247PT6+t6EeAciM0bJyko3Y3m4zvWYnb9YMsiLCSFpaGikpR0qvf97ZxPvNRixOL7+7tDjwvaLI0qwoOk3OacLsadEa1pTE4/WLZMeG9oTOyIthVXEcC9MiUchlZMRosLt93Lsmj28sTydtQjIuTCEjJVLFo80OXuto4beXlkgsx26Tg28/tQ9RdPArr41wwS0JeL+4b5hPO80Bq7RJAbMyLZI0vZoVOUey0l6zA5kg8MglxWRPmfPcVDfE79/v5O4v5VCdKEMQBFpH7NjcXvZ1W0K2+/MLC0iOUksVjeCYznkl8ZJgRf3AOJ+0j4IAt67MRBsmpyItkrVlCWREazjYO8aP/tVEtFYpzaf+YFUu39tQz86OUR55r520aA0/rJBJ1Yr6gXE6TQ4cHj89o076x5z84/pKTHYPCToVMpkQMvg/1TUkIiJCGr04ET3pzyLpZz73x2azoVKpyM7ORqlUolAoeOKJJygpKaG5uZkPP/yQe++9F7VazcMPP0x19fQ5a0EQOO+88xAEgVtuuYWbb7455P0PP/yQxMRE8vPzpdc6OjqorKwkMjKSn//856xcuXLejmkmfG4D5mxzg5NxMgEzKNpusVhOSrT9eHE8AghwYgHz1QP9vF0/xA9XF5Cqk3Pw4EHCw8N5rc3H1h0HuG5ZBj/7SjGtBiulKbN7BMKRoHas367D7ePq9XsYs9r54yV5FOVO78eJosjHbUauKQ7ja6uWEROuDAmWweD6PxcW8lGbkXMKZ86cVQoZ5xUH9ESLk0JZtpdUJGOzO4h0GsjPz59G2rpuWToWh5dvrMiUmLfjDjfnFcVRlR5JrOZI71MQBHpGnZSlRPKV8oRppegojZJfXVSMZ8Jmq9lg45N2EzV943zYZuI3l5ZIgc7i9LK734VM5qFl2EbFRE9ar1USrQ1DLoMlFWVolDIp+6yKsGBIUXBRWXiIcHZmjIZHLy+VVIQAntjRzca6QK/zT18rD9nPD1pNmGwedrSaqEqIxSeCXq3gv9fkUzxBJPL6Ra7520EMVjfPfH2h9Lc1feMMjLnY1jTCHRNzq+UpOi6uSKIwMVwi/IzaPSREhHF2QSzjTi8yQSBhYrbT7vbxVr2BVcVxJOhU7GgxEqVWgOimw+Tg7lcPkx6t5ltnZrKlcYROox2dWoFMNt3QO4ipriFWqzWEORpckByNOToZ/wkl2aPB4XCgUqnYtWsXjz32GEuXLuU73/kODz74oEQc2rVrF3v27OGKK66gvb192mL3448/JiUlBYPBwKpVqygqKuKMM86Q3n/hhRf42te+Jv1/cnIy3d3dxMbGsm/fPi666CLq6+uJjDz6M+dk8LkNmHPByVhy1dTUoNPpWLx48b+tFNPf309XVxcVFRVzlqg6EcbtxtpBGgbG+ehwH3myYUpKSrDZbMg7DCAEypnaMDkLZiESAZjtHv7wQTvnFsVz59k5pEVPZ9xNRq/BSI/RhlwuRxU1c6Cr6x/n3tcb8fq8XHUeElFncrCEgA7s3i4LVRl6ItXTB627TA4KEsL5wXl504QJrGYjCb5htKlZ02zednWM8pONh/nmigyWTJpPfeqTXp77tJeK9Cj+evUCiXEL8IPXGmkdtmOyuSSh+Mn4+VstvHpokAfXFZERrcYvBqTufP6ALVcwYMZoldxcGYFGHx/ishKpVvDWtwKr+eA5CD7szWFmSvwj5MWqJNUhvV7PJ0MCj+8cYnm2nkcnfDIvWphIy7CNKxZPd4n54apcFqdHcV5xPKLDwj9qLGxuHeDsghhpXMXnFxkcD8w8jto9E5KEsG5hIgq5IOnyjju9XPbkPmxuHw+sOyI4/8cdnYG+47CdyrRIfrwmV2oVvFk7xG+2dSADEqNUXL88jcsqkzl0YD9v1g7TbnTQYXSwKEPP89dXYnF4UMpls/bKp0IQBHQ6HTqdjqysLLxeLyaTicHBQZqamtBqtVL2ORuR77MWMOdbfN1msxEbG0taWhpLlwZmVy+77DIefPBB0tLSuOSSSyQFNZlMxsjIyLQFfbBSk5CQwMUXX8zu3bulgOn1enn11VfZt2+f9HmVSiWVeRcvXkxubi7Nzc1UVVXN23FNxRcB8yg4kYAZtOTKzc0lMXF6ue9UINivdDqdR+1XzgSZLJDxHA/uWV3Ae4c6yMAomUvb7XauXhjNDWcVh2Qms+H95mFe2d/PB80jvPud04762b6+PlpbOvntpaWoNWqy42ZeDKRHa8iI0SC4bWgUzBgsAR7e0kZt/xiRGgU/WJU3bTv/s6mJvV0W+ixO7rsg8NAWRZGuri6MRiMP7/fh9rWhDFOFZKnvHR6m3+zkzdohrlh8hESzIDVQUq3K0COXy5HL5SiVSvx+P+cUxGK0eVicppOuQzD7lMlkNBtsCEDrsJ1VRXEkR6lZXRLPod6xkO8eHHfTNurlugX6EOF1URS54+V6mg02/nrNAilQAfzw9cOM2j0kReVyxeJAIDebzYy3dePz+xk0Wuju7iYuLo5UvZqbTkuXxjcmw+ryYbS5cfv8KESR+PDA/ZekO3IfuLx+frwmsACZHNA3HAgwZEsmMtE+sxOT3YPH78c9QSby+UXOL4mnfcROTqyGP+7oIkwhY2tB4Pgr0iLJjtXg9vrpHnXw5Mc91PVbcY+7uP6cON5tHMbq9kml5KgZ5BSPBwqFYkbZucbGRrxeL9HRAcKVXq+XguRnMWDO5/7Y7XZiY2NJT0+nqamJwsJC3nvvPUpKSsjNzWXbtm2cddZZNDc343a7pxF7bDYbfr8fnU6HzWbj3Xff5b777pPe37p1K0VFRSEqbMPDw8TExCCXy2lvb6elpYWcnFApx/nGFwHzKDhegYD+/n46OztZsGDBv01TcbJg+7H6lTPheEuyfr8fr7GbFYl+ysqOqBPJZDIQRVL1R88UXz/YT5fRzuWLUllVknBUmyRRFGlubqa2b4zf7PWSoOvitVtnVl7pHXXw881NXLMkjTRvH63NTcTFBey1pq6kbzo9gxf29kskk6lYW57IwJiLVRMM26ChtiAIVFZWUthQQ8PAOOlTXE5uPSOLxEgVXyoMXTmvKk7g3KL4adfmozYTfWMuXr6xivgIpURMCl4Pp9vDnWdmMOrwcmZ+HIIgoJTL2LC/l5r+MSrTo6SRixf2DfFOu5N+Zyt//8YR0Qe/CHu7LfhFkfYRe0jAvLwyifdbTBKDNkjiWFos0Dzeh0IQGbL5MZtbued9CwY73L4iietOywk5p7/e2sYn7aMYxt18e4me4gQ1L92QKwmaA3zrpTraR+z8+Pwj/SeATXUG2kfs7Oo0kx2nxexw85ML8lHKBb5UGEe/xcmNz9eQolfxo9V56NQKDvWPU5IYgUwIKPbs6hhlbWkC5ckRPL+3n16zky2Hh3H7RNIOG3n720tDzLbnEzPJzgU9K1tbW1GpVMTGxuLxeD5zpJ/5DpharZbHHnuMq6++GrfbTU5ODk8//TTh4eHccMMNlJWVERYWxjPPPIMgCPT393PjjTeyefNmhoaGuPjii4FANnnVVVexZs0aafsvvvhiSDkWYMeOHdx3330oFArkcjl/+tOfiImZmfU+X/jcBsy59DBlMhle73Q9zamYnOFNZW2eShxvv3ImHE/AnEwmCoofBDHT+bQ4PHzUamRZdgyxEWG4vX4e2dqG1+9nSXYMv7p4ducRr9crlbVz8/IR9tZI4wsz4f3mET5uM9JptPGvW6sZGxtjZGSEjo4OlEql5COoVKlpG7axbkEi+QnTFzXNQ1Y+7RjloYtLWJAaKZXX4+LiyMgICA589+xsUvRq4iJCB9s/7Ril3+IkNiI0g9nbZeb2F2v4UmE8D1xULL3+wDut9I46yIrRcsvKLAApEPn9fn7yegPvHjZy04pUzsqLxu328lGrmef29OHxiXzYauLyRYES6eqiGA72mLl2aaj0niDA41eUMmx1szgjig9ajOTFh5OqV3PbGVlcWpkcogz0TuMI929qJkwhY3VRHLkZKcSEZ7Gkp4l3GoeJUbjYv3+/ZF4cGxvLJQuTGLa6A1ZZg2Z+snUInWaUzbdXS6MpUZrAbyJSfSTQ+vwiP1qdx4HeMS5ZmMTvtnWwsc7AV8oT+OF5gczfMO7G4fFxeMjGHS/XUZIcyZ++eqSHuvXwCH/Y0YXD40cAnvhqKWUpkdy3sZnGXiPVmYG2wKkIljNhqmel3W7HZDJhtVqpra2Vss/o6Oh5dwv530TQqaSiokKan5yM559/ftprKSkpbN68GYCcnBwOHTo06/b/9re/TXvt0ksv5dJLLz3xnT4BfG4D5lwgl8uPWa4MKujExcUdM8MLzirOx8rO6/VSX19/XP3Ko+3TsRAsNefl5c1IC58p8D71cSf/3NfPmpIE7v9yMWEKGTeelkGH0RFSlpsKu90eOKfJ6fx+l5GkyCFe+GYVeq1yxvMriiKri+PoGLZyRn6gRBMdHS0RchwOB0ajkebmZg702XjikAeZIHBmXgw6TWiP8ulPunmr3sC408sj6/Kk8npwQbKtaYQfvtZAgi6MTd9aFvK3P93cjMPjozwlkosrjvT6Oox2vH6Ruv6xkM9/+6wsNtYaOL90euleJpOBIJv4d6CE6/P5SI5Usq4sHpfPz7kF0dL9pFbKODdHwzkFoaWu721oYFfHKL+7vJSGgXE2HBigPCWSm0/P4MNWE3e92kBJko6/TRBx4ieINEWJ4dyzOk8au7l3TT7fOiOLeJ0q5Jze/3oNNQYP96yMJU/np8EaCNJ6tSJkjvORS0ow2z0kTrIqu/2lOpqGrDzx1XJ0agVp0WpkAqTq1VhdXn63rYPkyDDuOCsLRJHHd3QTGx4I7s0GGxtrh8iP15IXH05d/zgIMDzuJkqj5PeXl7Jnzx6qZ5mz/XchOLc4MjJCUVGRFEA7OjrmTfT8s4DJpLH/ZHwRMI8CuVyO0+mc9f3R0VEaGhqO6sAxGcGgcjIBM2gw7fV6qa6unlGe6ngwl5nOoDj90UrNMwXMJZnRfNxqCpHRu275dHbrZPh8Pvbv3095eTmtFpE9nW3IBPjOObkz+mSKokjfqJ3bX6whLz6c03KnXweNRkNaWhpJKSnEpDv4eKSRKKWPuoP70Gg0UkagVqu5Zkk6Y04vl5XpqampoaysDJ3uCFM2KCQeEz6d9XzbGVns6hjl9ClemRctTEKvUVKSHHru8uMjOL+UGX0nh60uzsiP4etL0yhO1tFqsFE3MMbKvFhKUl2M2d1EqBWSYPw9G9sZGncTEzfAVycZTvdZnIgExMxPz41hWXa0RLBxegMauY5JmqmlyRE8enkJFWlRyGWBqoEIfPvleg71jvHwJcWcnhsjndP9I71YXAIDbhWpJhPWYQN3lMuoLojD4XCg1WrxiyI3/r2GfouT9dcslEq1/RYn/onRI59fJCVKzZNXlVOUpGNHq4m3G4ZxeHxkxmgmjKIXkDHxtz94rZG2ETsKGbxy02Kcbj+D4y7OyD9+c+d/B4LSeEHCFQQUx0wmkyR6HhkZKQXQU12lmu/gbLPZ/uO9MOFzHDDncsPMRvoJmkz39/dLpJe5ILi9E/0xTO5XRkZGzstNf7SSrCiKtLW1YTabqa6uPqp1z0wl2ZX5caw8huhB8G8EQaCnp4cGg5PX+7VcJoxy/fJM7jg7hwSdatZg6fP5aBu2MjjmYsTqwSeKKGY5L3e8VMeBHgu/v7yMpdnREmFjZGSE+vp61h+yU2MU+dEZsYTbB1m4aJHEwgsiO07L+msrpjl5+CeO48z8WOKmBNN+i5O36odQK2UhPd5bX6jB7AhUML6yILSfevcrDdT0jXHT6RmUpkQybHUzavPQZrDxwNutCAJUZ8dSlhyBx+OhNBocHjnFCVqpXyaTyfjNJcX0mp1UZerZWDuEw+0jK0bDlsZhLE4vz359YQhJ676NzXzQauIby9P45vJ0fr+9A5vbh8XhQRDANsmD0+Lw8IuvFNJldHDRwiRE4LbNwzg9fh5I9mIdCzBvdZFR9I3a8U4Ex8wYDaIo8vtLSxhz+ViYqmPL4RF+9lYLcRFhvHZzFUWJWi5aGJDAazLYeO3QIPu6zagUMn5yQQFn5sfQYbSjkstQK2TkxIVTwomZO/87MNNCWa1WS6Lnfr9fEk7o7u5GJpNJwgkRERHzHuDmWyzd4XCQnDydQf2fhs9twJwLZgqYPp+PhoYGAKqrq4+rD3Eyc51jY2PU1taSn59PQkICBw4cOGnBAZg9YHq9Xmpra9FqtXMajTmReU6Tzc2t/zhItFbJtytU+DxuhjwqRqwePm4z8c3Tsmb1xhRFkWd3ddE0aOWuc3O574JCUvVqFEfJ3o3WgIhCMEhNJmxkZmZy365d2NwOaruNnJ6qoLm5WSIOhYWF4fb5uezJvTg9Pp66uoLSlCMP6KYhK4+934EoiqzMiyVVfyQIvbJ/gHcbh+kyOUJEEJZlR7OrY5TCxOlZe3mKjvqBcYom3lucEUVOnJZItYJVxfFYHB7yE8LxeDxs/OgA+sgIXl5XSKIuTCIO1fZauO3lBnJiNTywrpB/7Oln1O4hJlzJr7a0IwBPXlUeshjRqRUgikSpFXh8IkPjbrw+Pz+7sACPT5TmKgfHXFz19AHC5AKv3LQ4oK3r86OSC3h8AhmpyWTGaPD7/bT1DXPdAidKrwOZqZMeYnim1sY7h03cuyZvguUaOLbKtEge2drG83v6WZYVxQ9X5dI+Yue5Pf00D1lxev385eNuHr6khBtXZODzi+i1n30PxmO5HslkMkk4IScnRxJO6O7uxmq1npTl1vHuy4ngi5LsF5jVkislJYX09PRTzkgNYmBggI6OjpB+5Xwo9My2nWAPMTMzM0TF5ljbOd5V64jVjcnmxjBqhcXpLCwpxurcw8LCVBZnzdx7cnv9IAbmK//8YRcen8i5RfGcXzq73Na408uGA/3817k5aJTyaf3TEaub95uH+UaJnDFfLNecVYpSLsNqtTIyMiKREaKiYwiTgROmae/mxGk5PTcGtUJGYuSRB5rN7eWcoji6Rx1cUnEki/zHnt7AjOS3l6JShC662kdsaMLkvHnbEmmwXq2U80HLIL/e0sptZ2Rx42mZWK1WDtbW8lqXgkN9JoSwLu67oFDKZGzegIB596gTm9PL6qIYOk0OFqdGsCJbz5jTS/YkbVuPz895xXFctzSV9OiAdvB/nZOF0eblv15pRK2U8ex1FWiUcrx+P35RxOOHIBdLIRP47op4YsMVUtlVJpNx0yvtWBxe7js/j4ICPUajkY7BUTweLw3tfVQnyvH5lNy/toCqjChue7EOvyiyq9PMFesPsKYknme/vpC/7eplW/MIX6sK3JMzVR0+yzie58VU4YTx8XFMJlOIcEJQtu94WzynYsTF4XB8UZL9vGNywBwZGaGpqYnS0tJpA+snsr25IDhWYbPZprFvT9QTcyqmBsyg9VhZWdlxmWnPlTw0GakRAtfmixRk51JeFOhtqhUyzi+Nx4ecT9pNLEyLktReBswObnjuAHqNgvXXVnDXubm0GGzHFHd/7dAAj27vIDdOy4abp0ty/fKtw2xrMrK6UM+vLj8iCh4cVs/Ozmb9Rx1s2jrIbRVqtKIT93AnQ2Ig+3zl0BDP7url/rUFIfvi8fm57C97GXN4+dt1FRIr1+728Zut7SDAGXmxrJhCTPnl2y182mlmxOrmJ2uPDO+3GmwAtBhskmNEeXk510bZ8ezs5svloWXd4uRILqtI5s26IR7Y2sFPLyxke9MwX3u2hm+vzODyyiRAlPw+f/9+J//cP8Dq4jh+emEhG2uHeKdxmHULEhl1eMABNpcPjVKO2yvy8y8XUpAQLvV1N9Ub+OW2QfJjVTxTmCUtoILrKFEEFGFs6RG59vR8wsNkZEWIDBiMfGvzIGYXaBTwnTMzWZal5+UD/QyOuRkacyEIAt9Ynh5if/Z5wWTLraysLDweD6Ojo/T39zM2NkZ4eLgUQKe2EGbCqQiYX2SY/+GYaw/T6/XS3t7OyMgIVVVVc7ohZ8PxjnDU1NSg1+uprKyctr8notBztH0SRZHu7m4GBwdPyHrseDPM4ALk4pULQ6SsgoH3zx/38OqBfr68IInvn5ePKIqY7W6cHh8jfhGvHy5aOHvPRBRFNtYOIZcJLMuKpiAhXHIegUDPcXvzCDqZl2TBQnS4krNLZ8+mX60x0DvqwqZOZ111KhaLhZGREbq6unhpt5Mes4/3GoeoytSHXCu314+IiGfSSIw2TM4Np6XTY3KwMH06W/jiisCYxpqS0Kz51pWZrMiNJk3lpK2tjcrKSn77fjd1/eM8dmU5Cboj9+bmuiHu3xTQW02NUpOgC+PFfQPsajfh8Yo0DNkl5m3wnozTygFRkpxrN9ox2jx0mRz85pISNEoZcRFhGG1uvvn3GhBFnr2uQvpOnUoBIkSEBTwzr/7bQYbGXPz2smIUMhnlKTreaRzhyY+7AciND+eqqmRW5GSTk+Bgb7cFmwf+sKOL/7ckjF+dpWfYp6Es88TEuv9ToVQqQ4QTbLbAAqqhoWFW4YTJmC/Lwsn4IsP8AoiiiMViQavVUlVVddKrsrlmmMca4YD5Lcn6fD7q63PS62gAACAASURBVOsRRfGErcfmmmFODswzLUCC5KHsWC1hChlZsVqJ3JMXr+X7q3JJ02ukrHPydttH7CRHqdGGyekyOfjVu60AvHTjYl66sUr6nMXhpWlonB+/0Qiij7dur+b7uul9RFEUefXgAB6fyM++HHDU+MqCJARBQK/XM+xWcGhIyT3na/m0dYjKGAe7du0iOjoag0/LxqZxHriomASdiqzYUJeQAbOLG1ZkEh525Cfo8Pi49m/78fhEnr9+UUjJ8YMWI99/tZ6qZBU3L1AFhOdlMl47NIjb62dnu0laQJjtbh7e0obN7SM8TMFjV5ajUgj8+aNuChPD+caKDL5UFCepDgF0G20Igoxfryvkpf0DPP9pD9csTuJgj4W/7uzhtpWZXDthJq1RyolUK/B4/ROm3yJ/3NHFqN3Do2tTSIpSY3f7aBy0BjSVDXY0YXI21Rm4ujqFirQoGgfHOTxo5Z43mkiNUrPp9mqe/7SHfx4YIitWwxlLCvA4rKiNRrqa6hgIC5MYpHPJZOab1PJZhSAIREREEBERMaNwwkyGz6ciw7TZbF9kmJ9nWK1WyZKruLj42H8wB8wlyAU9Ho+lFjRfAdPj8WCxWIiPjyczM/OEyQBztRxrbGzE5/PNGpiD21lXkcwF5YkoZIJky7Wv28LvtnWQptdwZVUyNrefL5cnIhMEtjWP8LPNzZQm6/jDVxeQHKViaXY0chnS7OAfP+jgrXoDqXo1VQkCkWEiRSmxREbMvDLutzj59ZY2AJ67fhG3TogLBPHTzQEVohtPy+CO1WXSMZrNZh58rYl9/U7GLKPce24mDq0gPbCe2NHJ+y1GPH4/D150xO5tzOGldzQwBmK0uUMC5pDFgc/nZ8jqYeHCJdJ1umJRMm/XGxi0uPCLIoZxF05PIKtN0oVxx9lZ/HZbG9kxWjbVDeHxiVy9JF3S0LW5vHzcbuKlff3s7TJzem4MFqeP1hE7UdowMqLVHB6yER4WuA69ZiePfdDNHWdmcUZeDGEKGaN2Dy/u60cEFsXF8fSBQWIix/jOWZl0mpx8uTyBFY98gk8Uqe0bY01pAt85O4snP+7mvcMjeP2B++baZRlcuyzjyAnWHBnBCM59trS04HK5JMcVvV4/Y7Z0Kogt/xcwm3BC0PBZr9ej0Rzd2/ZEYLfb/23qZv+b+NwGzKPdMMG5w/Lycurq6ubtO4+WYU7uVx5rhCO4rZMNmBaLhdraWlQqFVlZWSe1rWNlmMGRmLi4OLKysmY9/5O3I0PE6w2cL5lMhjZMgVwmoFbK+P22DvwilCTpyE8IR62QIwiC5BGpUsj57jk51PWPS4H87YZh+i1OZD43ysQINt+5csagPWp388qBAUasLvITwknQqciIOVKidnoC2dOXyxNxenycOWl0JjgO8N3zSnjm0x6+XpWIKDpobGzE7Xaji4pmbaEOr9/PVdWBjK3P7ODXW9pYXRLPqqJ4WkdsWF0BhanmISu/fa+NxXo7938pmaTEBB7e2sZFC5KI0ipoGBgnPiKMxZlRbG8e4fWDgzg9Pkx2D1UZetL0GrY4RvigxYgoBog5k7VU3zs8wpMfd5EYqSIjWsPXqlKJUCloMlg5/4m93L4ykzvPyUWvDtxvbzcY2N48Qp/ZQaxWwYdtJq5ZksYtp2dgdngxOex81GVFEKxcVpGMy+tnxOahKDGcdqOdwXE3rxwc4KuLU3j4khIO9lpI06uP+QAPzn2mpaXh8/mwWCwYjUba2tok+bnJ2efnNWBOxVTDZ7PZzMDAACaTiYMHD0rZp1arPanzFZy5/U/H5zZgzgRRFGlpaWF8fHxOQet4MVvA9Hg8HDp0iKioqBn7lTPhZHuYQWeTysrKo0pSzRVHy3itVis1NTXk5+cfU8IvGDBnsuUqTdHxjxsWoVLIeOrjbqwurxTITsuN4eUbF4fIvN335mGaDTbc3jzWLUzip2vzeG/fYVbkJbCkNGfWstT6nT0892lPQLFGo+SGFekhTNZfvdvK2w0GrqpO5Z83VePzi4xY3cSGB5SIavrGeOz9Dq5fnk5FZgxNQzayCpNQKwQu+8sexp0e7qlSgrGTg7YoNrXaeb85YDt1fmkCJrtbknJ7ZX8vH7WZsCRr+ccFhTy7q4c9XWYUAuzpttAwME64SkF1ZjQ7Wo3IZFCZHoXJ7uGM/Bh8fpHbz8jika1tVGdGcWF5Eg++28LV1amckR9HcpSKwTEXSrmM129dIin7vFk7hMXuYV/PGJcuSsVs9/BJh5lzixMxjHtYVRTLw++10zJsRxRFdrSNEq1R8M2FWvJjVURoVLzTOMyY00uPycELNyzC4/PzyoEBUib5YVYcxdFmNsjl8hABgJmyzxMl5p0KfFbKw8HFHARYuOnp6ZhMJtrb23E4HJJwQnR09HHPittsti8yzM8TJpNsZjImng/MFFRO1N3kREuyUzPZ+VIUma0kOzw8TEtLy5wF6YMBczankWApcSYrrPgp2q4rcqKxunwUJkZgs9lw9DURFqHnrU4PedmekM/3mR08v7uXtWWJrMyL4cPWgKeiH4jRhmF3+2gYGGdBWmQgMIIkUPDS3l5e3NfP8uxo7lmdzxuHBtjbbUallBGpVvDY+x2kR2u4+9xcHF5AJqegdAHxapFbXqjF6nBTGC2QrBNZnq5hVVEcv9rSRkqEggqtmQtLYylI0rP1sIE1pQnERYTRbLAiAGUpOr5WnYrR5iY3VkvjgBW318+Gm6q4f2MTbxwaJClSTW3/OAq5QKpew+CYi8aBcVbmxRKmCMjqBd1GvG6RVw4G5Awr0iM5LSfwgP3Ne2283WBg3YIkPH54+eAQFy1M4sO2UYqSdDy7u59Wv0jzkJU7VyRw0ZJcfr89YMlVlRUIikq5LESFaL4wNfs0m80MDw9LbZWg5u3JqmKdKD5r2a4kpzhFOGFsbAyj0UhXV9dxCye4XK7jJgr+X8QXAZMjIuZBUYBTBblcjsvlkv5/rv3KmXAiATMoJB4ZGTnnTHaumFqSFUWRzs5OiV08l2FrURRRKpW0traSmJhIXFzcSQ1p33R6FjednoXRaKS2toWS0lJefrebD1uM7O8Z45srMiTN1w37B3h5Xz8tBhsRKgVVGVFUZUaxKD2ahEgV9715mPeaRrhmSSrfOjOba5amS9msye6hfcROp9HBquJ4rl+egUohZ93CJMLD5OhUcgbMTp7a2c36axfSa3aSptfwdqOBFqMHtVLOhZUpbGkw8Of32yiI8rO/y8NB4PZbKlmp0vLjNw+zpwvKUiIJVyn45/4BFDKBTbcv4a87e3j+0z4KE8MZtgbEyr1+kfyEcN5uMNBjcqDTKEiMVHNheSKLM6L484ddbKofZv21C3no4hJiwsPQKOU8t7+Hx7Z3kBsfGBf5tGOU65elkx6jJkqjJE2v5pH32vH5RQ72jPHIpaUszoji3cNGPmk3gt+PS1Ah+v3ceWYGd56ZMS+SkHOFXC6XHvIul4uCggJJR3guvc9Tgc+itdfUY5fJZCGZ+YkIJ3yWjvFU4XMbMIPBoq+vj+7u7pMWMZ8LJo9wnGzpVy6X43a75/x5m83GoUOHyMnJISlpZlurk8HkAO73+6mvr0cmk7F48eI5/ZCCTNicnBxsNhtGo1EqFQdJDCciEdbb28vAwACVlZWoVCq+VBjH7k4zfWYnT33czfmlCaiVctaWJ9JlcrAoI5J/7htgZ5uTDQcGuG5ZGt/7Uh7JUWpkgkBSZKDfZrZ7eOrjLtQKGafnxvDivn68PhGtUk56tIYfnJdHw8AYH7dZuGVlFt947iCH+sfw+0U+bDUiCJCm1xClUXB2QRwXV6ays93M7kEb4VotF+eK5CRG0dHaDILAgpgwmi2wpcHA1iYjApCiV9FlctBhtNE+YiMiTEZOnJYrFqVgtnsw2z14/SIKuYzSJB0D4y7GHF4q0/UMjDXjF0VMNg9+UeQP73dgGHdRnhpJcZKOirRINtYNSSLnSrmMDTdVkRyl4u1GA60GO0qZgNXlRRAEvrdExxWZLrRJOZSlRSGbuKaTzbKD854ymeyUP1yDWd1M2WewDBk2wbw91dlnsFryWYHP5zvm+Z9JOCGw8KxFFEUp+5w8EmY2m7npppuoq6tDEATWr1/P8uXLeeyxx3j88cdRKBSsXbuWhx56aNr3ZWVlodPpkMvlKBQKyfHk/vvv58knn5RaOb/85S+54IILAHjggQf461//ilwu59FHH2X16tXzdYpmxec2YPr9fhoaGnC73ccsTc7XCjEY5Pbv309kZORJlX6PJ8McHh6Whtwn3+DzieBxuFwuDh48SFJSEhkZGXM6vsmzgHK5XBrSzs7Oxu12SzZdNpsNvV5PXFycZBw7G4KlZ5fLxaJFi6TPXlCaQJhcxoEeM0VJOtTKwOt58eH85rLSiZGWcB7d3kbbiB1tmIIN+/sZsbp4+cZFJEcFHqx/+rCTdxqGCQ+Ts6tjlAfXFZMWrZHGR57Z1c3vt3egEODrS9O5ujoNp8dHcpSaTpMDr08kWhvGm7cvIUIVWDBZnF4cHh8+t5t7rjpNuudcLhfbtzTxXrORT9pMaMPknJsbyYBd4G+f9CCKAYPmeJ2KW8/MJjY8jD980EmLwYpGKac4KYLWETtGm5uBMSc9Zgc5sVrWlidQkBjBQ1ta2d1pxusXaTfa2f3DM/CLIgvTojDa3Dy0pRURUMoFwhRyXrihinGnl36Lk/x4rWQKvKyqctrvZLJZdvA6BwXjIdQsez4xUxk0mH0GjRImu9icyuxTFMXPVPY1U4Z5NEwWTsjOzg4RTrjzzjsZGwu48Nx8882sWbOGDRs24Ha7sdvtbN++nTfeeIOamhpUKhUGg2HW79m+ffs0Y2mA733ve9x9990hrzU0NPDiiy9SX19Pf38/5557Ls3Nzae8avC5DZherxedTkdaWtpRH+pBNup83PBut5uBgQFKSkpOOsubC+knWBYdHh6murr6pDUojwWfz8fevXspLCyc8cafaf+O1q+EwEp3cp/FbDYzMjIisSOD2efkDMHr9VJXV4dOp6OgoCBku80GO1sOD5Mdq2XdDMIHgiBwWm4MxUkR1PWPsTRLz0V/3ovD4+OsgjgpYF68MIkRqxu/X6S2f5y/fNRFYWIEa0oTWJyhp3HACiK4/SJ1A1a+vyqXYaubJZlR/KtmkD6zkxtWZNBtcvLkx02cWxhLZYwPr0fBJcsLcXpFbn/xAH5R5OdfLuLgkAeNUsFli5MpiFag9tl45KNh0iNlZMdHsrYkjrMK4xlzePjJxiaGx10IAmTGaBged1GRFhno5yZE8PVnDjDu8iIT4OKKFL51ZjaR6h4O9Y2xqjBw3WSCwKriwKq+KiMKbZiClEn6uDq1gvwwLfX19ajVakpLS4+plRr8DQWrLKcy+5xL33C27HMy83Y+ss/PYkn2ZLgLk4UTnn76afbt28eNN97Ixo0b6ezspKWlhZtvvpnc3FyeeOIJ7rnnHmneer5aXm+88QZf/epXUalUZGdnk5eXx+7du1m+fPm8bH82fG4DZpAldiycrMNIEENDQ7S3txMTEzMvJdFjZZhBMQKFQjEvogvHwtDQEA6Hg9NOO21Ope25BMupCBIRgky/oNNIY2MjHo9HcnHp7OwkIyNjRveEcJUctVJO9DEEu402DxtrDYzavdy9KpeWIRtVmUeYl7HhYWTGaKlIjyQuQkWYQmBz3RCGcTeLM/TctSqXmHAlfr/I+WWJfPulWsadXrJiNXz/3DzajTZaDTY6TXY+aDbyQdMwfgTS9Gq6TU7y4r30jjoA+Oe+flqHrUSoFFxSkUJWrJY3agYwuo0MDons7BslWiMwOGLEJ1MSrVYwPA4+PxMWWDLuPDuec4riaRu2Upaio3vUwdUTQgSV6VFUps/OVs2bwWg7aPA92Vh7rgjei1Ozz2BZPvhvuVx+wtnn8ZZBj5V9TlbPOd4s5rMWMH0+30kplk2GTCZj4cKFqFQqysvLyc/PZ8OGDXR0dPD000/T3NzMhx9+yL333otarebhhx+munq6PKUgCJx33nkIgsAtt9zCzTffLL33+OOP8+yzz1JVVcUjjzxCdHQ0fX19LFt2xI82LS2Nvr6+eTmmo+FzGzDn+mM6GYcRCASG1tZWxsbGKCsro6en54S3NXW//j97bx7dSHmmiz+l3ZIl2ZIl77vdttu7e+8AgUCzBm5gGJKBZCaXgcxNDudCJjcZfpcbJplDQpPJzEA6JOQmdAcYmp2GbiA9BMIFQm/Q3d73fddua9+q6veH+aolWbJKtmSr037O4UDTUqlUqvre733f532eWAHT6/Wivb0dhYWFvDYFawHLshgdHYXNZuOcP/i8hyyKfINlNMjlcpSUlKCkpATBYBDT09Po7e2FWCyG2WwGRVHQarVhPeLi7Az8zyvLoVGsHDAXPEvkGaPDh//WXIqra8LHYUYtboyaXSjMkuFnt23F7KIXWRliXPu5nJ3R7seg0YVsuRgNBUrU5Gbi1JgNg0YXPho2Y8zigctP4/YmLQzzFM5bhHD4aOhUUly3VQeNQoJ/+6t6MOySl6Xg3CwWPUG8+NkMHryuGuNmN7IyxNAqxOidd6JCr8Jdu4owOm/DNk0QBz9zY9YjQG2uElp1Ji6vXtpkHOmYhzZTgh9cW8Vp2yYKYppeWlqaELM7FqJln6Fl+tVkn2stg0bLPiPnPvlmn+kWMJN9Pm63GzKZDOfOncOBAwfw/PPP4/7778f+/fsRDAZhs9lw6tQpfPrpp7jjjjswOjq67Jn/5JNPUFBQAKPRiH379qG2thZXXHEFvv3tb+OHP/whKIrCD3/4Q3zve9/DwYMHozLy16NPfMkGTCC6h2Mk1hIwCStVqVSira0Nbrc7Keo8QOwMc2FhAT09Pairq+MyMT5YDfWdpmlO+KCtrQ2nTp3i9TnJCJaRsFgsMBgM2L17N2QyGRwOB0wmE+ctSEq37fM+HOsy4MYGfZjgQCS2lWRBq5AgVxV9J355lRYauQRVuqUNQoFahv+1r4r7+1JtBvaUZ6NMK4dQIMDjf92ACasbHwyYcVNDLmYWvRift0Jhn8T+v27FQlCI9/rN2FuRjc+Fb9D8+Yyin2awpzsb3XMOXPF54NtVng0/zeJr2wsgEwvh9ATQb3Tja3sqkSkVodE+jiyLC/UaCh8NG3HwHROursvFjnwprAF5WHk1EbhcLnR1dWHLli0J3V98EZp9AliWfQaDQe7vV8o+kznKsdbsM916mHxIP4nA7XYjKysLRUVF2LVrFwDg9ttvx/79+1FUVITbbrsNFEVh586dEAgEMJvNy+axiSuSXq/HrbfeijNnzuCKK64I25Dde++9+PKXvwxgKaMMTT6mp6d5OyutBZd0wOSD1QZMMqwfykpda7Yaimg9zOnpaUxNTSVkak2OlSgRYDVZbGjWkKwHlvRpbTYbtm3bxpXOCUmhsrISPp+Pyw66xh1YWBTCbJWCprNjfmcBRUEiFGDS6kFtbjg7l2FZvNNtgMMbRGNhdNPiTKkIX2nJR4b4wvFLNXJ8c89S+XLeYAS9aMC2va3IyMiAEkvkoJ8eH4TdG8R3r67kjJ19AQZikQAtRWrU5Crh9AXBssA9XyhBtnypL334zDS6Zx3Ilouwu1yDv99bigDNoH16EV02CsVlWVCphPCbzZB4FjE6YAnz+uSDhYUFzslmvYbUV5t9pnL2MdHsM91Ysok+6/HgdruRnZ2NYDCIgYEB1NTU4P3338fWrVtRWVmJP/3pT7jyyis5clgkv8HlcoFhGCiVSrhcLrz77rt4+OGHASxZG5LWypEjR9DQsCRBecstt+DOO+/EP/7jP2J2dhZDQ0PYuXNn0r5TLGwGzDhYTZALldZTKi8sqMmy5AKWj3EMDAzA5/Nh586dCT8MiQZMMrfKN4tdTb+SDwjTWSQSoaWlJWYQlkqlKCgogFeixr58Bld5XGDcC/j0008hk8m47DN08DpAM/jVR+Pw+Gnc/6WKMPF0T4DGiVEbAjSDa2p1KMpevjmZsnnwxJ9GoVVI8OB1Vdx3ZlkWI6NjOPDRLCQKFVpdDCo+fztFLflsCgUUxIIL10gpE+GBL1WAZpZ6p291zeNI+zyurs3h5PWurtUhJ1PKmVELBRQYlkK5Vo7/eWUFtJliiAQCzuHC6XTCZDJxoztarRY5OTlQKpVRfx+TyYTR0VG0tLRs2IB6vOwztPdJAmmqwSf7FIvFaRcwk51hyuVyHDhwAHfddRf8fj8qKipw6NAhKBQK3H333WhoaIBEIsEzzzwDiqIwOzuLe+65B++88w4MBgNuvfVWAEu98TvvvBPXX389AOAHP/gB2tvbQVEUysrK8Jvf/AYAUF9fjzvuuANbt26FSCTCk08+uS5ztZsBMw4SCZikX7m4uBh1vjJZllzkvBiG4TRaNRoNamtrV/Vg8ilNE8zNzWF8fBytra28XSNSESyJMpNer+dFOllwB/D7k1OgAPzjNZVQZyxl/S6XC2azGT09PQgGg9BqtdDpdFAqlajPV8Lg8EGrCM/AFBIR/n5vMdwBBoUxSpsCigJFLY1iEDAMg/7+frAsi51bijGz6A0jHwkoCt+9uhIBmglzMflwyIz3+kz4291LmfzWPCWGClxoKbowItRQoEK1XsFJ2wHAkfY5vHZ+DvX5SnxvXyVEkqVFkqIozuuzoqICfr+fU3hxOp1QqVTQ6XTQaDQQiUSYnp7G/Pw82traki4XuRaslH16PB6wLItgMLguc58E0bLPmZkZ2O12uN3uDVcdAlJTkpXL5WhpaeHmJ0Pxn//5n8v+X0FBAd555x0AQEVFRUx5zueeey7m5z700EN46KGHVnnWq8MlHTCT2cMkrEGFQoFt27ZFDQzJchghx/L7/fjss89WtAHje6x45xVKXuIrqZeqfqXT6UR3dzeqqqp4ja8AgFwqRLVOAUqAMGswQlQqLS1FMBiExWLB9PQ0bAuLyIIce7boIRMuv0e25qvgp2Nfs8IsGR6+sQZS8dKcYTAYRFdXF7KyslBWVgaN3QelTIRMabi91//9eAJSsQB/v7cEYuHSojZicsPo9GNmwYtqfSYqdAp89+rKsM+zuPx48sMxaBUSfPuKMggoCgIsuZFY3H44vXRYEA6FRCJBfn4+8vPzOUs7MvsaCAQgFApRX1+fNBnFVCA0+zSbzZidnUV9fT0ArMvcZzSQ7JOmaahUKuj1+jDnkOzsbG7ucz17nMkuybpcrkvCCxO4xAMmH/AJmNH6ldGQzLKM2WyGw+HAnj171txPihcwyWIvl8tXFFsgGxDy71SRe4aGhhLuo0mEAnxj98q9VpFIhNzcXOTm5uKTEQuOnZxAr3UeX8ydDrNNksvlmLJ58euPxtFUqMTffF4WjYTDF8T7/TZsL1JgdrQfJSUlyMvLw5jZjQP/bxS6TGlYudbpDWLC6oaQouANMFzAvL0tHztKs1CTt/R9/TSD505Ng6KAr+8qgkQogD/IwB9k4PQGwbAsggyLIk0GfvTlGuRkSmOSlyJBvD5VKhX8fj8YhkFWVhbGxsbg8Xg44Yjs7Nj9343E3Nwcpqen0dbWxvVmo/U+QzPPVAcrUgINdQ4h2SfxrUzm3Gc8pCrDvBSwGTDjIF7ANBqNGB4eXtavTBXIGIfVaoVCoUgK+WKlgOnxeNDe3o6SkhIUFq4snE30ZAUCAYLBYNJ38lNTU1xpcC0iDDa3H3KJMMyBJBK5SilyVBloq87BzlodvF4vzGYzhoaG4PV6YWUVWHQFYHTEPo8/9hnxX90GDA55ce81DcjOzgaw1KeUCAVQykRhGwmdUopvX1EOsZAK88IElizF/EEGEqEALl8Qg0YnAMDtoyGRC5CvluH+qyogEwshEgjw2vlZ/O6TCWzRZ+KJOxoTuj6E/axWqzkrNmIPFWpOHEs4YqMwOTkJs9mM1tbWsGw4Vu8znVSHQn0r/X5/mOpQss8nFaSfzQzzEgCfrCeWZivLshgZGcHCwgJvcfG1gmR6GRkZ2LZtG68xDj6IFTBtNht6e3tRX1/Pyy5JIBBwJbxkZpVE5s7v94fJ3K0G4xY3Dp6YRHF2Bu69rDTm6yp1Cvx/11Vz5VuZTMb1phiGgdVqhUw0D3gN6OhwcoEjdCC8SSfCeKYXt11WzwVLYKlc+89froFEGL4QTtk8+HjYgi/VhJeZj/ea8F89RtzcmIuvtOQjWy7hzj0rpAfKsMCAwYmmQhXyVFJkiITQKxMbUCc98cLCwmU0fYFAsGyRJ8IRfr+f+7v1LjGSTaTL5VqR/EUQ2fsM/Qe4kIElK3jyIdmsZ/aZbNLPpeKFCVziAZMPomWYof3Ktra2dVkc3G43Ojo6eGV6iSJawCSi9HxHVMgu2mg0Qq/XJ40cQjYJKpVqmczdaiAULJFxQskx0fDnEQve7jLiv7XkYU95OBOYzHVuz1Rj0upBkQJYsFnR1dUFhmGg1WrBsix8FgvuvbYVednhu2+Ly48XP51BY6ESV4TMgp4es+HDIQvkEkEYK7c+LxOjJhdXkmVZFlaXH25/EOVaOfddXj47g84ZBzKlAtTlKfGfd7dxWrl84PF40NHRwbs3HCocQdM0rFYrDAYDBgYGoFAouLGVZKnKRAPLshgYGADDMGhsbEz4/ogMnsCF8adQ6T6ySVut6lAi/d9UZ5/JHrlxuVy8uQQXOzYDZhxEjoIQ14/y8vKo0mt8kOgNa7Va0dfXxzvTSxShAZNkc263O2Fyz5YtWzA/P49z585BJBJBp9NBp9Otelfs8XjQ2dmJ0tLSpDmsLPlSVkEmXnmhcfho+GkGTm/scvzLZ2dxdmIBd+0qwmWVZSgrK4Pf70dvby8cDgfOm4HfdHXg5sZc3LKtjLuWExY3zk8vYsETCAuYX9yihUwsxM6yC79xkGHgCzL4xq5ijpHrDtB47fwcAjSDan0mbW8VgQAAIABJREFUCrOWru/20izM2334bHIB41YPvrq9EHIJv/vMbrejp6cHW7duhVq9OlNn8nuzLMuxj0M3ETk5OVCpVEkdK+ru7oZcLkdlZeWaj0uCT7TsM7Qfn2j2udaMbqXsUyaTcc4hG1UW93g8myXZTSwhdBTEaDRiaGhoTa4focQYPpicnMTc3By2bduWsvk3EjCDwSA6OjqgUqnQ0tLC6xxDyT0KhQJVVVWoqqqC1+uFyWRCf38/fD4ft2BmZWXxOu7i4iJ6e3tRV1eX9E1CZH8wGvbV6tBUoEKeOnZ2VJkjx6TVwwkMkHnYjIwMNDc3w9lrRH/7DLweD86ePcttIiqyNPjm7mKUasPLWP4gi0ypMIw52z/vxNMnJlGYlYH/fX01AEAuFuK6rTq4fHRYybWpUIUyrQzv9WVCoxBDncEvyydEqubm5qSU1iiKQmZmJjIzM1FWVoZAIACLxYKpqSk4HA4olUou+1xtJYKmac4curQ0dml9LYglGJ9o9pnMEmi07JPMfaa69xkLm6SfSwR8Fm6RSIRgMIiRkRFYrdY1u36QEm+8m5lhGPT19YGmaWzfvj2ljESBQACPx4OhoSGUlZXxzpzJnBs5RihkMhmKi4tRXFwMmqZhsVgwNzeH/v7+uAvm/Pw8JiYm0NLSsmG7ZrFQAIvbj3m7D9tK1FHvlatqdLjqc41ZIoOo0+lQUlIChmXRUKBEa/FW5CqloCiKIw4NDA5iwOCGsFALBZvLLW6vnZ9F54wdUpEAeyqWysAFahlqcjNRl3eBUEZRFL4UoW0bZBj89PgQPptcwHVbdfjqDn5l+2is0mRDLBaHeSva7XaYzeZlsoUKhYLXMxkIBDiVqfWQQwOiE4ciSUOxep+pVB0i2Sd5zjYi+9zMMDfBgWEYWCwWSCQS3mbIK4HPzCMRt9br9SgtLU25SojX68XMzAxaW1t5leMSFSMQCoWcHVDkgknGNUjpdmxsDIuLi2EydxsBuzeAw2dmQLMsirMzoo5lTFjd+P3JKewozoTGPYXKykpOI/P0mA0vnZ3F7vIsfG370tgJIQ6NezPw595JmEEjS3qh51enlkNEKTl9WgDIlovx9Z1Fy7LiSasHL342gyuqNdhdrgEFCjTLwO4N4tzkIuYWvSjVxN71syyLiYkJ2Gy2ZazSVIKiKKjVaqjV6jDZQkLaied36vV6OSP0SD3S9QTJPkUiUczskwTO9RJf55t98hUp4QuXy7WZYW5i6Ubo7u6GRCJBbW1tUo4Zb0zFbrejq6srIU/JtQTUqakpWK1WlJeXpyRYRiJyweSyroEBLCwsICMjA9XV1amfjWNZ0AzLzToSDBmdMDn92FaixmVVGviDTExnk3m7DzNWJ/wLRvzgy01h108uFUIkoMLKqwSVOgWai9TYU5GNurJsrufnHZ9DpcCC8X4rHJ8rDk06GDxzehqtReqwec9JmxvjFjey5SK8fn4OviCD/3NDNa6o1EIiEiJfHbt8T/rUwWAQzc3NGyoMTmQLQ/1OSQAVi8Vhs69E+L2mpiaMdbzRiDe2QlSHQoPoeiBa9mkymThuQLKyz82xkksEKy32JpMJg4ODqKurw/DwcNI+cyU92bm5OYyNjaGlpYXXDbga0XQCItMWCARQXFzM6xipkLmTyWTQ6/WYn59HRUUF5HI5DAYDBgcHkZmZyS2YyZRkY1gWhz+dgc0dwN/uKuJ6fSzL4oVPZ2D3BqHLlOLmxpWJRsVSP67O8+OL21qgVofP4DYXqlF1swLyKCzVfLUM/3B5KUSfs1spikJQKMXR0SAoSo0HWkrgdy1ifHwcPTOLWFwUwKFe6qWT32lHSRaUUhHkEiHe6TGCYYFFL43Lq3Ow6AmEadGGffcQokwyWMfJRKTfqcfj4TZTbrcbgUAAVVVVqyIlrSdI9smyLIaGhiAUCqFWq9dkV7ZWkOxTrVbD5XKhqqoqab3PzZLsJYxQYQDCEk2W/isQXU+WPFhOpxM7d+7kXR5bbcAkPSCNRoO6ujpMTU3xksZLpcxddXU1V0oiTEti0XX+/Hmu16XT6db0cDIsC5YF7J4AJwZAsNQbzMHsojemRizB5OQkTg/NwasogFS2vFy76Ang8KfTKM+R4/qt4Z6RRocPB/7fGPRKCe77YjkoioJMJIBGLoFAQCFTLoVYmYchpxilWwrwYBMDr8OGzz77bMlwOSMLL/e6oJBJ8D8uL8P/vq4afppFlU6B505PYW7Rizu2FYaNpgAX+qx6vT7lPqnJQEZGBoqLi6FQKDAwMIDq6mo4HA6cPn0acrmcI5JtlBj8SiDjLgDQ0NDAPS/RBOOB9ZPsi1QditX71Gq10Gg0vLLPzQzzEgWZ+ZPJZGH9ymTW/CNLsoSZqlQq0dramrBLfKLatETGr7KykvOaI8o8sbASuWctIA9oNJk7iqKWWXSFKu1kZ2dDp9MltCP+eNiCjmk7/qo1H9/YVQxvkIYuMzzYXValXfEYpJwZCAQwgxz0jNlQk6/E7ohZzblFH3rnnDDY/csCZoBmEKBZuP00bJ4ADp2YRGGWDA9cXQFgSYTdYPfhnW4jBAIKD15XBVXBUr/O4/HgRP80BmascAUYFEu9uHVbCeSZKgQZBvlqGVx+erlS0Oe9v/Ly8jXpDq83jEYjxsbG0NbWxs1zsizLiSaEiubn5ORArY5O0FpPsCyL3t5eSCQSVFVVhZ1PKsyyE0E0wmGs3ufAwAACgUDc7HMzYF6CIMIApaWlKWXehZJ+1jrTmaj7CSlvNTU1hcn4kfJRJFLlNAIs9U4NBgNvdqZUKkVhYSEKCwtB0zRsNlvYkLxOp4vr7Wh0+OD0BeHwBpGvlkGZ4O1PJOOUSiW2bNmCzHwP6gxKNBYsHzGq1ivwzT3FywIyAMjEQnz78lLkZEpgdvlhdgXgCzKgGRZ/7DMBAK6p1eFLtTlLEnpSEfw0g5c+mwEA/PW2Cqg0OZiyuFGvFWBsahavdlmgkElxa0sB9mzPgyLjwnUgWXxtbW1K5nhThZmZGczNzS1zSaEoKqpo/szMDPr6+rhSfiJen8kCKXkrlUqUl5ev+NpE7MqSqToUryKVaPbp8Xjg9/tx++23o7u7GxRF4eDBg9izZw8OHDiAX/7ylxCJRLjpppvws5/9bNnnlZWVQalUQigUQiQScY4n3//+93Hs2DFIJBJUVlbi0KFDyMrKwvj4OOrq6lBTUwMA2L17N5566qk1Xxs+uKQDJgkApF/Z0NCQ8v4IyTBJ8FrLTCdf9xPCiDQajdi+ffsy5ZVox0mlhyUhnKxWJSlUCD3S25GiqJhjCjfU52JXmR8Fccqt0eDz+dDZ2cmNMkzZPPivXiOu2qKFIgqxZ9ziwrjFjYqc8LLo3KIXT344BpVMjO9dU4k8lRR37ylGllwMpy+IU2M2UAD2Vmhwbd2FTNDjD2LU7AYAeAMMKnIUUEhEqNAr4MnNQY5pEgtODw6fMyC3ewq7C5YIMxKJBJOTk2hsbFw30+dkgJiCt7a2xl3gQ0XzSSnfbDaHeX3qdDpkZmamNPukaRqdnZ3QarW8LOcisR7ZZ6KM3ZWyz+effx5OpxMikQgPPPAArr/+erz66qvw+/1wu9344IMP8Oabb6KzsxNSqRRGozHm53zwwQfLSI779u3Do48+CpFIhH/6p3/Co48+isceewwAUFlZifb29lVcgbXhkg6YpF9psVjWPF/JFwKBAAaDAV6vN2rwSvRY8QImMVlmWRbbt2+P+rBEHidVwZKUvNVqNWpqapJy3EhvR1K6HRkZgdvthkaj4dw15BIh5JLEGYEulwsfnOmAJr8YeZ9XAnpmHTg3uQiVTIwtuUq4/EE4vTQ3fvJunxnnJhehy5QgVyXFzIIXX6jUQCSkIBYKIBMLYHP78btPJj/Xhi2BgKJwe+vS8UM1YgFAnSHGN/cs9R2zMsQ4dGISg0Ynvr6zGPUFSty9pwQzC178sd+IytxMNFdno29wBIN948hTSTA1NbXiuEa6gPTz/X7/qhi8oaX8UK/P8fFxOJ1OqNVq7jokc5SGtFby8vKSIl2ZquwzlDi2GoRmn5WVlXj33Xfx5z//GZ2dnbDb7QgGg/jKV76CvLw8/PrXv8aDDz7IrXGJtgKuvfZa7r93796NV199ddXnnSxc0gHT5/MhGAzymq9MxvAxTdMwGAwQCATYsWPHmkss8XqYfr8f7e3tcec5I6XxUkHuIVT2srIyrne6FszYPPhw2IpdZVmoDJlbDC3dEpF0UkEg+qbqbA0mFgLIU0mhUay8SbLZbOjr68N5VxY8Aw7k53hQniPHFyo1yJQJOUGB505NY87uxX/fU4IyrRzX1emQq5KiqVCNpz4ah9HpQ4E6A7V5mfj+viqIhBRsrgD8NAO3PwiWBUABOTHE0pf0YwNQfG4CvbUgEwxY5KqkMDl9eLvLgMZCFf5+bykyJALMzczg1MQi7OJC1JTlIldOcxsJ4jKi0+nSijBDxDpEIhHq6+uTcu+Fen0yDMPNAI+NjUEkEoWNraz28wiJrqSkJCn3djQkK/tM5kyoUqnEbbfdhp/+9Kdobm6GSCTCI488grfffhsvvvgiBgcH8fHHH+Ohhx6CTCbDz3/+c+zYsWPZcSiKwrXXXguKovAP//AP+Na3vrXsNQcPHsRXv/pV7s9jY2NobW2FSqXCI488gssvvzwp3ykeLumAKZPJsGXLlrivIz2+tTzAhHShUCigVquTctOu1MN0OBzo7OzEli1b4g54k4CZKnLPwsIC+vr6Vq1TGg1Gpx8Guxdzi96wgBmKUBUZMutoMpnwh0/a8f5kAHX5Ktx9eWVMhZm5uTlMTU2hra0NrjE7phe8yMlcCrBKmQiXVV4gCCllIlhcAk6jtkKnQMXn53XtVh2mbB6UaJayW+KAkquS4jtXlEMmFkAooODwBvH7k1MAgPuuLINKdiHLnLP78Hr7LCRCAb6/rwo7S7PRWpQFiUiA3jkHRs1uSEQCtBWrMTw8DKfLjT3NtTgzsQitQgJNdgY3rpGOhBmaptHd3Q2VSsVZiiUbAoEAWVlZyMrK4uQbCZHM4/EgOzs7Ya9PIjJSXl6+bkIKa7ErS7YXJrC0mTt37hwOHDiAXbt24f7778f+/fsRDAZhs9lw6tQpfPrpp7jjjjswOjq67Lf95JNPUFBQAKPRiH379qG2thZXXHEF9/c/+clPIBKJcNdddwEA8vPzMTk5Ca1Wi7Nnz+IrX/kKenp6Vt3aSgSXdMDk+1AKhUIEg8FVl2wXFhbQ09ODuro6eL1e+Hy+VR0nErFKssSjs7m5mVffiqIo0DTNeVgmc7EiQSfZMnf1+Upky8XIS8AYmeibqvWFcMlnkZ8R5Eq3hHWbnZ0NiqI4xaG2tjbYvDTkEiFuby2IqkN7dnIBW3IV+KvWPMjE4X8/bfNgyubBFyq1XKAMRZBhMG7xoi5PCalYwH0fWYRXZ45CgrbiLGTKhJCKBDjea0TXjB137ihCtV6BO3cUQZ8pRm9vL07O+BGQZuGOajnuKVpO8gl1GQkGg7BarRxhJhk6r4mClDNzc3NRVBTdjDsViLRsS9Tr0+v1or29HVu2bOE2IxuBeNlnqFl2sr0waZqGVCpFUVERdu3aBQC4/fbbsX//fhQVFeG2224DRVHYuXMnBAIBzGbzso0FIVnq9XrceuutOHPmDBcwn3nmGbz11lt4//33uXVJKpVyZd5t27ahsrISg4OD2L59e9K+Vyxc0gETuCCGvhJWM75BEGmTZTAYkjbXGa33OD4+DrPZzNujk2VZSCQSWK1W9Pf3Q6fTJaXPRfrDdrsdbW1tSZdek4gEy+YM+UKjkODv9pZxfyaLJSndBoNBZGRkoKGhASKRCB8MGnBq1AaGZXF5VTgxYcEdwLFOA1gAlTkKyCJizCejVvx52AqlTLRM/xUADp+Zxpzdj3u+UIKa3EzcvTc6WUQiEuDmplwwzNI9u+gJwhdk4QnQEAsFqMqRoaurC9nZ2ZCqVXA4/fD4aSAO218kEsWULVyNzmuiIG2D0tLSlJUz+WAlr89AIMD1wkl1yO12o7OzE3V1dWklpBAv+/R6vUlVHfJ4PFCr1cjOzsbAwABqamrw/vvvY+vWraisrMSf/vQnXHnllZxAQiSxx+VygWEYKJVKuFwuvPvuu3j44YcBAMePH8djjz2GDz/8MEx6z2QycWvU6OgohoaGUFFRsabvwReXfMDkg3hzitFAnCt8Pl+YTRZfZisfhAZymqbR09MDkUjEW/OWkHskEgl2794Nu90Ok8mEkZERyGQy6HS6ZabIfEDTNDeHxtf1ZCNBFku1Wo2Ojg5oNBpIJBJ0dXUBAPJEKjTlZWCLfnm2rsoQ4fJqDVh2OVEHAK6o0kIlE6G1OPqi2lyshtLkRm5E7zJAM6AoQPT57xhkGBz8ZBJuP427v1CCrzTnYcETQK5SCr/fjw9OnUe7Q44vZqlwW6tqmZMJH0TTeQ0lUK2mZLkSiP9mqGhFuiCa1ycxD5BKpXA6nevCql8rQrNP4lfa0NAQln2uZWyF6Mj+67/+K+666y74/X5UVFTg0KFDUCgUuPvuu9HQ0ACJRIJnnnkGFEVhdnYW99xzD9555x0YDAbceuut3LnceeeduP766wEA9913H3w+H/bt2wfgwvjIRx99hIcffhgikQhCoRBPPfXUumX4VJzsKrkqvWkIv98fN8Ps6upCaWkp7xp5IBBAR0cHsrOzUVFRERYwyE1bV1e3pvMGgImJCc6HsL29Hfn5+bzp7PHIPaTfZzabwbJsmMrOSgGQjF/k5+eva3lttfDTDE6P2ZApYuE3jCwjJRGWpclk4sTBSek2VWxTT4DG7z6ZhIAC7v1CKSQiAfw0g19/OA5vkMG9XyhBpkyEnlkH1GIGxvEBMOpCvDfmRl1uJv56W3INxgGElSxtNhtkMhmXfa6GOOR0OtHV1ZXUvvZ6gLRXcnJy4HA4Uub1mWzYbDYMDAygpaWF+71Cs8/QNTAR1aHR0VH88z//M954442UnfsGIeoPuZlh8kA8wfRQECWdqqqqqDTqRI4VDwKBAC6XCxMTE6itreW9S+dD7iGD4cQUOTLTIEEj9P3RZO7SHXOLXhzvmoXHbsP/uaURmghR70iWpcFsDRvi1ul0UGdrcGrSCaVUhF3ly0XB/UEGp8asKFDLUBWRpfppBm91GiAVCXBDgx4CikLgc+asABSCDAsJAIlQgLv3FiPIsMiWSzBgcOKF0xMQ+xbw4C2tkCsykadzh2WVLn8QgwYXynPkyOLpjRkLoSXLSKUdmqah0Wig0+l4BQ1CAmtqarqoFGJI0CHtFQBRvT5JW2O9esDxYLVaMTQ0hNbW1rBqUWTvM/QfvmbZl5IXJrAZMHn3MPkEOUK2aWxsDFPSCUUyS7IOhwNGoxE7duzgtfCsdr5SIpGEOUqE9vuIyg5FURgfH0djY+NFtQiKA04UCRex7Qs1y4JlJEbNbrx41opdZRpcs6sGbrcbJpMJfz7bhTeGfMjMkKEiawtyssKDxpjFjbe6DMjJlOIH11aFHXPRHcC5qQUIBRS+uEWLTKkIKpkY37qsFBQojihkc/vx3OlpFGVl4LbWfEiCLmhhxxU7a7h7rSIn/Lq3Ty3ieI8Reyo0uLEhef3BSKWdQCAAq9XKBQ2VSsURhyJ712Sz0dramlYjLfFgsVi48w4NOrG8PicmJtalB5zIea/EaYgMngC47HMls+zNgLmJZYgXMEMF2+ORbZKRYbIsi5GRESwsLKCwsDClwTISkZmG0+nE8PAwbDYblEolzGYzKIq6KB6i6elpzM3N4e+v38mLIOX6XKzd7g2GBY3iklJIdUYEPU7MT09iuN8Z5utYnJ2BvZVL/46ETinFHdsKIBEKOCswT4DGn4etKMiSYVvJEsvV4aVh9wYxu+jF7OwszLMz+O4ty887QDM4N7WIHIUEVbpMNBZ6sTU/tQo/YrE4TGmH9MJJy4AEDYfDwY3prLdk3VpgNBoxPj4eN+jE8voklZl4Xp/Jhslk4uYVE7neJBhGyz4jzbIvJS9MYDNg8sJKQS4YDKK7uxtSqZQX2WatGSbRM5XJZKisrITdbo/7nlQp97Asi5mZGYjFYlx55ZVc6ZaQnYgkWToIYgPAqMkFb5BBba4CIyMj8Hg8aGtr4714NRWokKOQIiczvNQmFFC4qjYXwFIWR3wdQ4UCtut0yMlZnlGZnX70z7uwveRCH29mwYsTY1ZkZ0i4gFmiycDf7irCgnGW0+CNdt4zC14c7zFCKRPhu1+qwB0p6GeuhNCgAYCbdezs7ITH40F+fj4cDseycn66Ym5ujjNXT7TEGs3rk9wTEokkTDQh2QgN8mstDUcbWyHryUcffYTR0dFknPJFgc2AyQOxAqbH40F7ezuKi4t5E1zWkmGSuS8yO2Y2mzfMlisQCHBjDETmLnSujabpMEFs0tuJVqZbD7j9NF47P4cATeMyrQf52Qo0NjYuux4My2LK5oFWIVlm/kxRVEzbryGjE290zOOa2hy0FmeF+Tq6XC70Tczj6RPnsFVDoa1cD51OB6VSiX6DA5+MWBCgGU7ooESTgZvqczmZPeBzhw7jJEQMg4YVJOPyVFLsKM1CrkqaFpsUqVQKn88HuVyO7du3Y3FxkSvny+VyLmisRSIyVZiamoLJZOKlZxsPK3l9+ny+VbnvxILBYMDk5GRSgmUkQsdWjh49io8//hgvvfRSUj8jnXHJB0w+i4pQKEQgEAj7f1arFX19faivr0/IAWK1AZOw87Zu3cq5zcfLVlOl3MNH5k4oFC6b7zOZTBgfH4dYLIZOp1tXaTaZWIDGAjkGRyZQnFuI8tLobOJBgwuHP51GtV6Bb+xa7hnpC9J45tQUBBSFv91dDIlw6brOLXphcfkxs+BFa8TbFAoFPOIsWOGBT6mCQrEkiO5wOCCVK3FVuRw7qi7Q4iVCQZjNGFHByczMRHl5OUbMbsglQhRmLS/xysTCpPYr1wLiCckwDJqamsKE8YnyktlsRldXF8c2JRuJjQ724+PjWFxcREtLS0oyYeL1SRxBbDYbjEYjBgYG1rSRCM2IU7kxPXr0KJ544gn84Q9/2FDRhvXGJR8w+UAoFMLr9XJ/npycxOzsLLZt25bwgs+HZBSJ2dlZTExMoLW1Nax8E0tQIZW2XKuRuQst01VVVcHj8cBkMnHSbGRkJZULpdfjQZZzEn+zt2pFCTNVhgiqDBHyVdF/V2+AgdkZAEUtsV9JwNxdrkG+OgNF2dHft600C1KxAJU6BbQKCfLy8sAwDBYXF5FlNmNqoBPGKBqvZEQpLy8PRUVFmF304j/PTCNDLMQ/XlPJfX66gWEY9PT0ICMjA5WVlct+11DlpbKyMo5tOjExAafTCZVKxbFN17MiQfgBXq8XjY2N61I2jnTfIRuJ7u5ujoHMR7pwdnYWc3NzaGlpSek1e/PNN3HgwAG8/fbbl1SwBDbnMBEMBuNmfCaTCTabDVVVVejr6wNN06ivr191mebEiRPYu3dv3NcR5wan04mmpqZlD4HD4eCYqaHvSVWwJDJ3TU1NScsMyUJpMpngdDpTQowgQb6+vp7XLC3DshCscN3mFr0QUFRYyTQZIKMaJpMJwWAQarUaVqs1bETJ7afx6vlZZGWIcXNj7oZnYtFA0zQ6Ojqg1WpRWlqa8PtZluVKt1arFWKxOKX9vtDPHRwcBMMwqK2tTYtrS7w+zWYz7HY75/WZk5MTVm6dmZmBwWBAc3NzSglFb7zxBp588kkcO3bsLz1YRv3xL/mASTRUV4LVasXs7Czcbjd0Ot2axaH5BMxgMIjOzk5kZmaiuro66ue5XC4MDQ2hpaUFQGrJPSMjI5y6Sap2r4QYQTYoGRkZXMa1WlalwWDA+Pg4mpqakqplGw8mhw9quXhZBvjhkBlWVwA3NeRCIoqdvSwsLKCrqwsKhQI+n4/LuLRabVrbcxHnDuIbmgyQfp/ZbIbP5+MyrmT0+whYluWcUmI9bxuNUK9Pi8UCAMjJyQFN01z5OJX3xpEjR/CrX/0Kb731FtcW+gvGpnDBauH1emEwGNDU1LQujgSETFRaWrriorMetlxEck8mk6G5uTmlC0koMYIMxxNjaAC81YaApesxOTkJi8WCtra2VZEf/DSDM2M25KulqNSFj2a4/TSOds5Dr5TiSzXh+pi9s3a8eHYWjYUq/HVbQdjxPhyywBtgsKM0C0VRxkyApQ3a4OAg2traoFAowjKusbExrge8kjD4RoA48lRUVCT1OYns9xG1rIGBAc6yjZhlrwYMw4T1iNMxWALRvT4HBgZgs9kgEokwMDCQEq9PAHj99dfx1FNPXSrBMiY2A2YczM/PY3R0lJNESzVsNht6e3t5kYlIDzNV5J6NlLkLnXOMpjZElGWiZRmEbELT9JpIG+NmN451zUOnlOJ/XRMuOGB0+NA5Y4dCIsQXq7UQCi4sshKxEEIBBWlEBikRCnBHWwHs3iAKYrBtDQYD168mhA+KojhbKgBcD5gIg6eDPZfL5UJXVxdqampSuqASKUidTsfNAZvNZm5TRYhDmZmZvK4FGdPKzs5eVfl4IzE3NweGYXDZZZcBAEesi/T6XKuQyGuvvYbf/OY3eOuttxIiOP4lYrMkG6Mky7IshoeHYbfbUV1djZGREbS2tiblM0+cOIE9e/Yse6Cnp6cxPT0dpve4EgKBAM6dO4e2trak9ysdDgd6eno23LooGgir0GQyYWFhIayvIxAI0NXVBZVKteZsweUP4r0+E4qzM9BWEr5QMCyL9qlFZMslKM9Z3ldz+YLIkAhX7IVGYnJyEmazOWq/OhYie1wrqeykCna7HT09PWhoaIipcLUeiNT9VavVK7rvbJStWDIwNjYGh8OBhoaGqBtCMv9qNpvh9Xo5bkCiGsivvvoqfvvb3+LYsWOXWrDc7GFGA8Mwy0ak7VFQAAAgAElEQVRGIvuHhKkYzS18NTh58iR27drF3egkIyLMPD43NCnBnjt3DgzDcLvuZMjSEceSi0HmjmQZRqMRJpOJ6zNXVlamtQLJoMEJigKq9Znc5szr9aK+vn7VGXHo+I7FYuHIMjqdLmWlW1I+bmpqSqvrTRjIhDgkkUjCytik11pcXIy8vLyNPt2EMDo6CpfLxfteiRTOl0ql3LVYaWP+yiuv4He/+92lGCyBzYAZHZEB0+12o6OjA2VlZcjPzwewFEDPnj3LGaSuFWfOnOGGigOBADo7Ozk5LT4ZUSS5x+/3w2QywWQyrUlhh2VZbli7sbHxopIvI8LvZWVlCAaDa74WfOH202ifXkSVTsHbTmvBHcC/vz8CAHjgqnLMjg9BIpEknWxCyDImk4nzdEzmtTAajRgbG0NLS0taCg+EgpSxCXEoEAiguLgYpaWlF4XiEBA+8lJfX7/q39DlcnFViWhenwDw8ssv4+DBgzh27NhF5SaTRGwGzGgIDZhEeSPS545lWZw8eZLXKAgfnD17FvX19Rz9vqKigvcuNx65hyjsmEwm2O32uGUpAuLfyTAM6urqLppFBLiQ5TQ0NCAz8wI5J/JapIJpenrMhsOfTqOlWI3/voeftVqQYXCkfQ4Mw6ICBuh1OSnvnwWDQVitVu5arFV5aWZmBnNzc2hubk4bVw4+8Hq9OH/+PPR6PXw+HxYXF2OOaqQTSBUiEAigrq4uaRsrcl+YzWYcPnwYXV1dqKysREdHB44fP36pBktgM2BGB8uy8Pl8mJiYgNFoRHNzc9TdMt/ZST44f/48cnNzMTY2hsbGRt4+m4mSe1iW5cY0rFYrMjIyuFJMaPZIZO40Gg1KS0vTliUYDbOzs5ieno75uxFEzvaFlujWMlNqcvjwhx4jtpWoUV/A73cElghVHR0dKCkpWfeSYGjp1mq1ckQavnOO4+PjsNlsaGpqSusRl0i43W50dnaitraWKzGGjmoQ44BE2NjrgfWaD2UYBo8//jheffVVKJVKMAyD6667DnfccQdqa2tT8plpjM2AGQ3BYBDt7e2gKApbt26NGYiSGTBPnjwJlmWxbds2XqWsZM1XulwuGI3GsIVBqVRieHgY5eXlUf070xXEIcbhcPDu+4YiVCSApul1lWUjjNJ0IVQRgkhoGTuyRAdcyHJ8Pt+Kz0o6ghhWxxOvIGxs0g9fD8PwlUD4DRRFYcuWLSm9N1944QU8++yzeOutt6BUKmGz2fDuu+9Co9Fg3759KfvcNMVmwIwGn8+H6elpFBYWrngzJiNgMgyD/v5+mM1mNDY28qLfp0qMgGTV09PTnBFyOjmLrASGYdDX1wehUMgJv68F0dSGSBk72UFhcXERvb29G84ojQUy52gymbhyJQkYw8PDEIlEKV+4kw3C4m1sbAwr2cdDpJAGX7JMsrCeYgqHDx/Gc889xwXLTWwGzKhgWRZ+vz/u69YaMP1+PycX5vV6kZeXFze7SKXMHSllNjU1QSwWr6rvuREgJKmcnNT0/UIXSavVCrlcHrWMvRqYTCaMjo6uu+rQakHKlUajEVNTUxCLxSgqKkoaG3s9sLCwgP7+/qSweInGq9lsRjAYTOn8K8uy6O3thVQq5U0GXC2ef/55PP/88zh27NhmsLyAzYAZCz6fL+5rYs1O8oHT6URnZyenCTowMMAxFmMhVco9hGnncrnQ0NCwLCCSXp/RaITVaoVMJoNer09KwFgr+LikJBNECJuwKwFwwTPR/tbFSpIJnVXMyckJm+1LhURdMmGxWDA8PIzm5uakZ4SR869KpZKbf13r78swDHp7e5GRkYGKioqUB8vDhw/j2LFjCWXflwA2A2Ys8AmYp0+fxrZt2xJmFJpMJgwNDaGxsZHbvQ0PD0OpVMZc9FOl3ENsouRyOaqqqng9iCRgmEwmAEjqvGciIGW1RFxSko3Q/pbH4+HlYUh6rUSHN90y9pXg9/s5icbIe5WUbs1m8zLxiHTYEBAD5ZaWlpRv9AiJimi8CgQCjjgkl8sTCnihMn0VFRUpPGvgueeew0svvYSjR49uBsvl2AyYseD3++Nabn322WdobGzkPW/GsizHvI18aMfGxjg39sj3pKoES2TuCgoKUFhYuOpjkIDh9XpTPuNIQIQU0mk4PpraEBnTIAGD9Kwpikob9wu+8Hg86OjoQHV1NbRa7YqvJeIRJBNfS8BIBubm5jjFrI0I3uQ5MZvNcLvd3MYqOzt7xQ0wwzDo6uqCWq1GWVlZSs/x2Wefxcsvv4xjx45dNOX1dcZmwIwFPgHz/PnzqKmp4bVgk5IKgKhswomJCQiFwjA5rlQGy1TI3EWbcdTr9Unve05NTXG2RemQuUQD6fURhR2hUAitVguLxYLs7Oy0FvSOBsIoXW02H7mx4pOJJwvT09MwGo0JyQumEkRlhxCHYplDMwyDzs5OaDQalJTwm+ddDViWxXPPPYdXXnkFR48e3QyWsbEZMGOBT8Ds7OxEeXl53KY4KWPp9fqYM43T09OgaZojraQyWK6HzF20vicp3a62HEa8QH0+35rk4jYCdrsdnZ2dEAqF3PjORouj8wXxDk2UURoLJBMnsmwKhYLLxJNdKp2YmIDNZlvVmNF6INSBx2w2g2EYaLVaaDQajI2NQafTobi4OKWf/+yzz+LVV1/dDJbxsRkwYyEQCHA2WbHQ09ODwsLCFTUVHQ4HOjs7UVNTg5ycnJivm52dhc/nQ3l5eUrJPaFi3uuZna2170l6rQqFIuUMwWSDlDKrqqqQk5OzTGEnnX0tzWYzR5JJBYs31F1krSSqyOMSfdVYYuTpiEAgAJPJhOHhYQDgWLepEM5nWRbPPPMMXn/9dbz55pubwTI+NgNmLPAJmH19fdDr9TH7OUajEcPDw2hqaoq7MzcYDHA4HKisrEwJuYf0zliW3XCZu1CdWz59TzJ+sxGWYmuFw+FAd3d3zFJmqNqQxWJZ97m+lTA/P4/Jycl1IckQRIoE8O31hYJUIoLBYFIl49YDNE2jvb0d+fn5yM/Px+LiIkccSqY9FwmWR44cwZtvvpk2PIA0x2bAjAU+AXNoaCiqJybLshgbG4PFYuFNMiA7bJI9JfMhJ3OKWq027WTu4vU9iQIOyc4uJqzGtSO0PEfTNEeU4evlmCxMTU1xspAb1feL1uuLN/9KBvuFQuFFJ6ZAFMYKCws5k4dQRKovrbYPzLIsDh06hKNHj+KNN97YDJb8sRkwY4FPwBwZGYFCoQjT/SSlQ7FYjNraWt76rna7ndtZ6vX6pC2QRCuzoqIi7WXuIrMtoVAIj8eDhoaGtJCLSwRzc3OYmpqKq2e7EgKBALdAulyuVWVbiSJ05KWxsTFtSpnR5l8j9V0ZhkFPTw/kcnnKZxWTjUStxSL7wHzFNFiW5RxHNoNlwtgMmLEQDAZB0/SKrxkfH4dYLOZGMrxeL1c65MtqCyX3kMFnskBqNBro9XpkZWWt6uG32Wzo7++Pq5WZjpifn+dIDzabDcDGzXsmivHxcVit1qSyMlOpNkRANEqJO006BxxSujWbzXC5XMjKyoLD4UBOTk7KZxWTDRIsS0tLV7WpDd1MWCwWMAzDlW5DdZBZlsXTTz+Nt99+G2+88cZFoSyVZtgMmLHAJ2BOTU2BZVmUlJRgcXER3d3dqK2tjTujRrASuSdSvzNRYkiozN1G98ISAZlVjQw4ifY9NwLEQSIQCKRUiDwy24p001gNSHaWkZFx0ZGqAoEAzp07B5FIhEAggIyMjKhjGukIwqAvLy9fUeUrEZDKhNlshtPpxNGjR1FfXw+73Y733nsPR44cWXWwPH78OO6//37QNI177rkHDz74YNjfT05O4u/+7u+wsLAAmqaxf/9+3HjjjQCARx99FE8//TSEQiF+8Ytf4Lrrrlvzd11nbAbMWOATMAmzNSMjA6Ojo2hpaeFd4khEuSeyVBnLkou8dnh4GG63+6JTkSH+myzLrljOXg9Py0RB1FgyMjJ4KyYlC5EzjqGm0HyCNk3T3Lxfqj04k41AIICOjg6u7xc5psGyLLe5Wu8+cDyQYFlRUZGy/jzDMHj//ffx29/+FqdOnUJbWxtuueUW3HTTTSgvL0/oWDRNY8uWLfjjH/+IoqIi7NixAy+88AK2bt3KveZb3/oWWltb8e1vfxu9vb248cYbMT4+jt7eXvzN3/wNzpw5g9nZWVxzzTUYHBy8qNYnxAiYGz/Zmwbg82AJBAKYTCYIhULs3LmTd/mNpumE5ispikJWVhaysrJQVVXFZRft7e2cbyGZbyQyd01NTWm1OMRDMBhEV1cXsrKyUFZWtuK5C4VC6PV66PX6sM3E6OhoUuY9EwUhVel0upQOmMeCVCpFYWEhCgsLucrE3Nwc+vv745pCk3JgYWHhMpWpdAcJOGVlZVwpk6IoKBQKKBQKlJWVcdnW2NgYV7rdSGsuAp/Ph/b2dlRVVfGuSK0GFEVhbGwMDMNgenoaBoMBb7/9Nu677z489thjaGho4H2sM2fOoKqqiit5f+1rX8Obb74ZFjApioLdbgew5MJD7qk333wTX/va1yCVSlFeXo6qqiqcOXMGe/bsSeK33RhsBkweoGkaExMTYBgGO3bs4BWckiFGQFEUMjMzkZmZifLycni9XphMJvT09MBut0Oj0ay7+fBa4fV60dnZieLi4qjswJUQupmorq7mNhMdHR0AUt/3JOceTVt1IxC6gQrVMx0fH4dIJOL+LiMjg+u5V1RUJK0cuF4g5x4v4IjFYm5EI7QPPDw8zG2u1rt06/V60d7ennLvU5Zl8Zvf/AbvvfceXn/9dchkMpSWluI73/kOvvOd7yR8vJmZmTARhaKiIpw+fTrsNT/60Y9w7bXX4sCBA3C5XHjvvfe49+7evTvsvTMzM6v8ZumFzYAZB+SG12g0CAaD6xYso0Emk0GtVmN6ehqNjY2gaZrbTZPSXLo6RwAXJPpqamp4eYHGQ2h2QfqeQ0NDKel7Op1OdHd3J+3ckw2KoqBWq6FWq1FZWQmPxwOz2Yy+vj74fD74/f6UlgNTBSIEkeh1FwgE0Gg0XJAi1lzd3d3rZhhO1o5U3zMkWL7//vtcsEzGMSMReZ1eeOEFfPOb38T3vvc9nDx5Et/4xjfQ3d3N670XKzYD5gpYWFjgHDKEQiEmJibividVyj3AkjjC6Ogompubuf5pXl4eGIaB1WqFwWDAwMAAlEolJ7KQLn0Di8XCubakIgOUSCTLSpUzMzPo6+tbc98z2XJx64GMjAwUFxdDrVaju7sbpaWlWFxcxPT0dFr7nYbC5XKhs7MzKcxvsrkqLS3lDMMnJibgdDpTcj1IoK+trV1RHWytYFkWTz31FD744AO89tprSSP9FRUVYWpqivvz9PT0sjL+008/jePHjwMA9uzZw82O8nnvxYpN0g+WmuWBQCDs/83OzmJiYgItLS3IyMiAy+XC0NAQWlpaYh4nVbZchE1qsVjiytyR0pzRaITFYkkLP8uZmRnMzs6iubl53c8hkkSVaN/TaDRibGwsJZ6KqUY0MQWWZbGwsMApyhC1IZ1Ol1YsU6KalOpNCsMw3P1htVqTor5E5qHr6upSakXHsix+/etf48MPP8Qrr7yS1PszGAxiy5YteP/991FYWIgdO3bg8OHDqK+v515zww034Ktf/Sq++c1voq+vD1dffTVmZmbQ29uLO++8kyP9XH311RgaGkrrzVkUbLJkYyE0YJJxAbfbjcbGRo484fV60dPTg23btkU9RqLknkTOra+vj7OISjQQu1wuGI1GbiRBp9NBr9evy1xWPLPqjUDoiAbLsiv2PQlxYr21eJMBEuhbWlpWDISkVGkymbiZvo1mmS4sLKC/v39D7Nwi1ZeIvqtKpeLtH5usrHglsCyLX/3qV/j444/xyiuvpGSz88477+CBBx4ATdO4++678dBDD+Hhhx/G9u3bccstt6C3txf33nsvnE4nKIrCz372M1x77bUAgJ/85Cc4ePAgRCIRHn/8cdxwww1JP78UYzNgxgIJmMFgEJ2dnVAqlcvGBQKBAM6fP4+dO3eGvTeVTiOEkZmTk4OSkpI1H9vn88FkMsFoNCIQCHCLYyr6OMTiTCwWp61sWeS8JxGPUKlUYWLe6RDoE8Hs7CyX0ScS6DdCbSgSJCtuaWnZ8IyelG7NZjMcDgfUajUnjh7tniDBsqGhIa6r0VrAsix++ctf4pNPPklZsNzEZsCMCVKm6ujoQFlZWVT2JsMwOH36dBg1OpXBkuiqVlZWpoTVGAwGucXR6XQiOzubUxpa6+K40aMXqwHpexqNRhiNRkilUlRWViInJ+eiCpjj4+Ow2Wxoampa03lHarsSW66cnJyUZdsmk4krf6dbEAgt7VutVkgkEm7DKZPJOA/RVJeQWZbFgQMHcPLkSbz88stpd53+grAZMGPB5XLh9OnTaGhoiNlzYFkWJ0+exN69e7k/p4rcQ3bZ9fX1Kd2pEpDF0Wg0YmFhgZvnW02w8Hg8nHdouuvZRoIM9ZPRFdLn24h5z0RBRCx8Pl/SlYeILRfpAydDbSgS8/PzmJqa4m1gsNFwu91h4uh+vx+1tbXIzc1NWTWFZVn84he/wOnTp/Hyyy+n7b34F4LNgBkLDMPA5XLFvQFPnDiBvXv3pozcA1wgyDQ1NW3I7pGQhkgfRyqVQq/X8woWi4uL6O3tjWlvlc4gtmLRhvoj+545OTnQ6/Vpo3NL+twikWhdyt+x1IZWq4M8MzOD+fn5DXVLWS3sdju6u7tRVFQEu90Oh8PBsbI1Gk3Svg/LsnjiiSfw6aef4qWXXtoMlqnHZsCMBZZl4ff7477uxIkT2LVrV0pKsMTXz+v1or6+Pm3KgNHMoPV6/TIyBiGZNDU1XXRCz4TVWF1dHVeJhQiBG43GpASLtYI45iiVSpSXl6/7OUTqIMdTG4rE5OQkx/5Ol3ueLxYXF9HX1xdmuE1Kt6Q6IRaLuWrNap8LlmXx+OOP4+zZs3jxxRc3g+X6YDNgrgSfz7fi37Msi08++YQjIyRzYaJpGl1dXcjMzExrMWxCGjKZTPD7/dBqtdDr9Zz10MXIJrXb7ejp6VkVq3GtovlrRTAYREdHB/R6fZgqy0YhtDoRL1gQH1mn04mGhoa0FduIBcLkDQ2W0UAEJEwmEwKBAMe65SuowbIs/uM//gPnz5/HCy+8sBks1w+bAXMlrBQwCbmHlEuB2JlWoiBya0VFRRfVcC8hDY2MjMDv9yMvLw96vX5dGZVrBRFTSMb4QuS8Jyllp0qKjWirlpSUpK08osfj4UrZJFgQVja5b7Zu3Zq2G8RYsNlsGBgYSJjJGwwGuQ2W3W6Pm42zLIt/+7d/Q2dnJw4fPrwZLNcXmwFzJfj9/qiSTtHIPX6/H0ajkcu0SE8r0dk1kt3U1tampdzaSiClQIVCgfLyck6302azITMzk1MaStee1OzsLGZmZlImppDKvidRkeFTQk4XEP9XIqghkUhQXV2d9mpDkbBarZyAyVo2QpHZONH+zc7OhlKpBMuy+PnPf46uri688MILq6rcxLPn+u53v4sPPvgAwFJbgpD+AOAHP/gB3n77bTAMg3379uGJJ5646DY2a8RmwFwJ0QImH3IPybSMRmNCRtCk59fY2HjROaETgkxBQQFnqE3AsiwcDkfYwphOSjIsy2J8fBwLCwvr1jcjfU+TyQSPx7OmvicZX7gYiVVkNlcqlUKr1cJsNsNqtW6YMHqisFgsGB4eRmtra9I3WcRY4dChQ3j99ddRWlqKYDCIP/zhD6uaR+VjzxWKAwcO4Pz58zh48CBOnDiB73//+/joo48AAJdddhkeffRRXHnllWv5ihcbNu29EgFf5R6RSIS8vDxO09VisXB2S0SjUqvVcgGXLNg2mw1tbW0XXc+PzIfGym4oioJKpYJKpUJVVRWnnNLV1RVXWSfVYFkWAwMDoGkazc3N61Y6lkgkKCgoQEFBwTJLrkT6noSFfDFp2hIwDIOuri6oVCrOmzFUGD30HgkdWUmXrIbMiKYiWAJLxgrFxcX44Q9/CKFQiDNnzqCiogI7duxAS0sL7r77blx11VW8j8fHnisUL7zwAn784x8DWHqGvV4vl0QEAoG0cOdJB2wGzM9BURRYll2TGIFAIAizW1pYWIDRaMTw8DA3+G02myESidDS0nLR9PoIbDYb+vv7E1IykcvlKC0tRWlpKaesMzg4CJ/Pxy2MfGXH1oLQEnJNTc2GLcSRllyh/p4r6bqSfivRNr6YQNM0Ojo6oNPpopKTIl1nzGYzp7S0EWpDkTAajRgfH0dra2tKN7gsy+Kxxx7DyMgI3nnnHYjFYjAMg/Pnz8Pr9SZ0LD72XAQTExMYGxvDl770JQBLQupXXXUVZ9J93333oa6ubvVf7C8ImwEzBCRY0jQNgUCwpkWVoihkZ2cjOzsbLMvCZrOhp6cHwNICMTs7C71ef9E08ufm5jA1NYXW1tZVS5aFOoqQntbU1BQcDgeysrJSRhoKBALo6OhAXl4eioqKknrstSCWv2dkpuVwODA1NYW2traL5n4hCAaDaG9v5zLseAjNxkPVhgYHB9dFbSgSBoMBk5OT6xIs9+/fj+HhYTz33HPcZwkEgpj61fGOF4lY69mLL76I22+/natwDA8Po6+vD9PT0wCAffv24aOPPsIVV1yR8Hn8pWEzYH6OUHLPWoNlJNxuNwYHB1FbWwudTsc12Ds6OtZdED1RhPb82trakkbiEYlEyM3NRW5uLmf2azQaMTg4iMzMTG5hXOvnEfPhi0F5KFqm1dXVBY/Hg4KCArhcLojF4rQpU8YD6XWXlJSsqqQnEAig1Wqh1WrD1IbOnz8fVs1JFQdgfn4e09PTaG1tTSl5jWVZ/PSnP8Xo6Ciee+65pHxWIhZbL774Ip588knuz0eOHMHu3bu5sv8NN9yAU6dObQZMbJJ+ACyNlHz961/HtddeixtvvDGpjNV4Mnc+n49j3BJB9NUwblMBhmHQ398PiqJQU1OzLiUxQhoiDFOxWMwpDSVKCCEEmbq6upR6EqYCZE7R4XBg69atHAt5I+Y9VwOfz4f29nZOjzcVxyczwT6fL+mG4bOzs5ibm0u5+hDLsvjJT36C8fFxPPvss0n7LD72XAAwMDCA6667DmNjY9x1e+mll/Db3/4Wx48fB8uyuP766/HAAw/g5ptvTsq5XSTYZMnGAsuy6OrqwpEjR/D2229DrVbjy1/+Mm6++eY1aUNOT09jbm6Ot8wdcYswGo1wu92cMECyFoFEQJxbNBoNSktLNyx4ezwebkORyHgGmZVLlWF1KkHISQzDoK6uLuzaR/qdpqOfJRl72bJlC0fsSSVomuZcRVajNhQJItXX0tKS0g0Jy7J45JFHMDk5iWeeeSbpgTmePRcA/OhHP4LX68X+/fu599E0je985zv46KOPQFEUrr/+evz7v/97Us/tIsBmwOQDlmUxOjqKI0eO4OjRo2AYBjfddBNuvvlm3tJjyZC5I4sAGXJOZY8vEqSMWVpamlZD8ZGydGRDEUkaMhgMGB8fvyhNnxmGQU9PD2Qy2TKLuWggLOTQDcVGMkwJizrV5smxEEttiLiKxMP09DSMRiOam5vXJVhOTU3h97//fdrOK1/C2AyYiYJlWczPz+ONN97AG2+8AavViuuvvx633HIL6urqogauYDDIaXtWVFQkZdEK7fGFCgOkwnqKON2nexkzckOhVquh1+vhdDo5bdKLbWSHuKVkZ2ejrKws4fcnc95zNSD3Tqr9IBMBURsymUycIXQsD9j10rVlGAaPPPIIZmZmcOjQoc1gmZ7YDJhrhc1mw7Fjx3DkyBGOhn3zzTdj+/btEAqFGBkZQXt7O/bu3RvVUzMZiHQTkclkXPBcK4PSbDZjeHj4oitjkg3F0NAQ3G43NBoNcnNzodVqL5qgGQgE0N7eHtUtZTWIJopO1JdSEQyIEHk63zvEEJp4wGZlZSEnJwcajQbT09NYWFhAY2NjSis4DMPgX/7lXzA/P4+DBw9uBsv0xWbATCZcLheOHz+OI0eOoL29HfX19Thz5gwef/xx7Nu3b13Pg/T4yIyfXq9PuBRJ+q2pkopLJYiCjFgs5sYzSI+PSI7xLcltBEgJvKKiIiVm4anuexJiWzwh8nQC2WSZTCbMz8+DoihUVFSkdNSLYRj8+Mc/hsFgwKFDh9KWsLUJAJsBM3V48cUX///2zjysqTN9/3cAEWQzIAGURZFFBAUUC6UVsQIqmKC2Ba1r1eqgF1W/atXSRWfGjlNnRp1xOlYda6eLVkuidlRccG9FRcUF2ZRVEBJ2wpblnN8f/s4ZQEDIRoD3c129rgIhPCdC7vO+7/PcNz7//HMEBQXhzp078PPzg0AgwJQpU3T6BsLYa4nFYiiVSvZNsTNXGCZ4uLGxUa9ixboK05xkY2MDFxeXl77edkuupdNQT3chA/+LFvP09NSZn7Amzz0ZA351vVV7itzcXEilUowYMYJdfQLQ+FkwI5ZisRgHDx7sdX9n/RAimNpg9+7dOHv2LA4fPgwrKysolUr89ttvEIlESE5OxsiRI8Hn8zFt2jSdNkHI5XJWPDtqkGEaTAYOHAh3d3e9EJDu0NzcjPv378PJyalLW+Btz/g0PYrQXfThzE+dc09mqL837krQNI2nT5+yjXktr7Xla8Js8TOviSrbtRRFYcuWLaioqMCBAweIWPYOiGBqg8zMTLi5ubV7FkFRFO7fvw+hUIikpCRYW1tDIBAgKioKtra2OnuTViqV7BtAXV0d60BUWFgIe3t7vchS7C5MN6aqowvMGZ9YLGabhmxtbXWWnsGMvWgiWkxTdOfcs6SkBCUlJfDz8+t153DMropcLn9pbKctFEWxr0l1dTXMzc0xZMiQLrsNURSFzz//HFVVVdi/fz8Ry94DEcyehBk1EQqFOHnyJIyMjM8JHLgAACAASURBVDBjxgwIBAI4OTnpTDwpikJpaSmys7NhaGgILper1WYQbcCYkGtqZdbS97eyslLrFmxMUo0+j720Hc9omTrDnJlre/RCG9A0jezsbFAUhVGjRnXr745xG2LOgg0NDdmt2/ZueiiKwmeffYaamhrs27ev171W/RwimPoCTdMoLi6GSCTC8ePHIZVK2XGV7v4Rd5fq6mpkZGSwzkMtQ49NTU3Zjlt97S5lOnm11WBC0zTbNFReXg5DQ0PWaUgT4saszHx9ffX2NW4P5tyzqKgICoUCzs7OrIFEb9nKZwwhAGjEgL+pqYnduWHchkxMTODg4AADAwN88sknkEql+Prrr1UWS3UyLQsLC7Fs2TIUFRWBw+Hg9OnTKo0r9VOIYOor5eXlOHnyJI4fP46ioiKEhYVBIBDA399foy3uzMpm7NixL4lNW6HQx+7S4uJiVmx0dWbWtmmopdNQd99wCwoKUFlZqbMcTk3S8szP3d2dbZBpbGxkdymsrKz0NoGHpmlkZGTAyMhIK+f1zFzwd999h2+++QZDhgyBlZUVEhMTYWlpqfJzqpppCQChoaFISEhAeHg4pFIpDAwM9Gb7vxdABLM3UFdXh9OnT0MkEiE9PR0TJ06EQCBAcHCwymdFNE2zQ9ljxozp0sqGEQqxWKwXOZZ5eXmora3FmDFjekxsWloXdqdBhjkzYxpM9FVUOoJZmdE0/dIOSHvnnurY0mkDmqbx+PFjGBsbd8k9SR0oisLmzZtRWlqKkSNHIjk5GTweD8uXL0d0dHS3nuvGjRvYsmULzp49CwD405/+BADYvHlzu48PDg7G1q1bER4ejsePH2P58uW4fv26ehfUfyGC2dtobm5GcnIyhEIhbty4gQkTJkAgECA0NLTLq76WockduRO9CibHUiwWszmWPB6vXbcUTUPTNDIzM9k3a30Rm7ZCYWlpCR6P91LTELOyMTAw6NEcTlXpjth0du7ZUyMnNE0jPT0dpqamGnPe6ghGLOVyOb766iv2dzU3NxcVFRWYMGFCt57v559/RlJSEg4cOAAA+O6773Dz5k3s2bPnpccWFBQgKCgIz549g6GhIY4fP44DBw7A2NgYeXl5CAsLw/bt23vdzkYP0u4vin7cAhLaZeDAgYiMjERkZCQUCgWuX78OoVCILVu2wNPTEwKBABERER02viiVylYp96q+WbSXY1lQUACpVMqusrhcrla2uRibQXXq1wbtBUGLxWI8ffqUPQvmcrnIysrSy/q7AkVRePToEczNzbtUP4fDgZWVFaysrODm5saee7bN99TVuSczNmVmZgZXV1et/6xNmzZBoVC0EksAcHV1Vennq5NpqVAocO3aNdy7dw/Ozs6IjY3FoUOHsHTp0m7XQfgfRDB7CUZGRggNDUVoaCgoisLdu3eRmJiInTt3wsHBAXw+H5GRkWyUUmFhIZ48eYJRo0ZpxGqtZR0tcywrKytRWlqKrKysDldZqsCEPjs4OGDYsGEaql47tAyCZs6CS0tLkZmZCWNjY3C5XDQ1NfUaFxzgf762NjY2cHZ2Vuk5Bg0aBBcXF7i4uLCzjU+fPmXPPdWZbXwVFEXh4cOHsLKy0nqjC0VR2LhxIyiKekks1UGdTEtHR0f4+/uzQj1z5kykpKQQwVQTsiXby2G2/IRCIU6dOgVTU1MEBgbi2LFj2LVrF9566y2d1cGssioqKtjRDFtb226fZTHxUCNHjtSKVZy2kclkSEtLg7OzMwYPHtzKfYlZZelD3mlHKBQK3L9/H/b29lq5WVEqlaiqqoJYLNbKuSdFUayJfXvuT5qEoih89NFHAIA9e/ZoVPzVybRUKpUYN24cLly4AFtbW7z//vsICAjAqlWrNFZfH4ecYfZ1aJrGkSNHsGHDBnh4eKCxsRGRkZHg8/k6dfJpOa/W3RBoxv1m9OjRPRIPpS6M2Lu7u8PGxqbV15imIYlEgvr6elhbW4PH4+ksTaQrMCbwTk5OOol20/S5J7MyHjJkiNYNOSiKwoYNG2BgYIB//OMfWlkpq5ppCQDnz5/HunXrQNM0xo8fj3379vU6R6YehAhmX+eHH37AV199hcTERNjZ2UEikeDEiRMQiUQoKytDeHg4oqOjtZ7I0BZmPozx6eTxeODxeC9tUTIm3vqceNEZUqkUDx8+7JLYM9vZzCrL0tKSXWX1VGMGYzU4YsSIHlvZM+ee5eXl3fb+VSqVuH//Png8HhwdHbVaJ0VRWL9+PYyMjPD3v/9db5rRCBqDCGZf58yZMwgJCWlXbGpqanDq1CmIRCJkZWUhNDQUAoEAgYGBOn2Dbm5uZrco5XI523ErlUpRVFQEX1/fXmnizbgPjRkzplOz+/ZgtrNbGkgwTkO6WhE0NTUhLS1NZatBbcD4Ibec9+zo3FOpVCItLQ0ODg4aPbNvD4qisG7dOgwcOBC7du0iYtk3IYJJeEFjYyPOnz+PxMRE3LlzB0FBQeDz+Zg0aZJOt2yYLcr8/Hw0NjZi6NChsLe37zEzdFWpqKhATk6OxtyHWhpIcDgcdjtbW01DTGLKqFGj9DY0vLNzTwBslqi2cmgZKIrC2rVrMWjQIOzcuZOIZd+FCCbhZeRyOa5evQqhUIirV6/Cx8cHfD4f4eHhWt8WZfx1ZTIZPD09WT9Xxgyd6bjV5zelsrIyFBQUwM/PTys3G0xkm0QiabUi11TTELON3JOJKd2lbYh6Y2MjeDwe3NzctLo7oVQqsXbtWpibm+Nvf/ubXv9eEtSGCGZXOXbsGLZs2YKMjAzcunULAQEB7T7uVT6P8fHx+OabbyCVSnVRttpQFIVbt25BKBTi3LlzcHFxwYwZMxAZGanxrEZmRs7ExOSlgfiW4b6VlZUwNzdnDeL1xT0GAIqKiiAWi+Hr66uTuuRyOSoqKiAWi9mmIXVGM2pra5Geno6xY8f2yjNjpkHJzs4ONE2rdO7ZVRixtLCwwF//+lciln0fIphdhXFmWbFiBf7yl7+0K5iv8nlMTU3F7t27IRKJeo1gtoRxSElMTMTp06dhaWmJGTNmgM/nw87OTq03ImZswdbW9pUzfjRNo66ujh1XMTY2Zrcoe6rjj7Hqq6urg4+PT4806bSNneosiqs9mHgxbZnYaxtmdKdtg1Jb+0JNzHsqlUqsWbMGVlZW+Mtf/kLEsn9ABLO7hIaGdiiYnfk8KpVKhIWF4ccff4S7u3uvFMyW0DSN3NxciEQinDhxAjRNIyoqCnw+v9sONkwnpouLC+zs7LpdS319PbtFqYvzvbYw8VCM1aA+nLUyW5TMTYWJiQm7ymrvpqKiooJNfNEXY/3uwIilq6sra9TRHu3dVHR33lOpVGL16tXgcrnYsWMHEcv+A7HG0yTFxcWt5rwcHR1x8+ZNAC8GmAUCgdYbEHQFh8PByJEjsX79eqxbtw6lpaU4fvw41q1bh8rKSkybNg18Ph+jR4/u9A1F3dBnADAzM4OZmRmGDx/Onu9lZGRAoVColSTSFSiKwuPHjzFw4EB4eHjohVgCrS3p3N3d2ZuK+/fvg8PhsOI5aNAgiMVi5Ofnw9/fv1fO5DU3NyMtLQ1ubm4vzbm2xcDAgA17bnnumZ+fz84GDxkypMObBqVSiQ8//BBDhgzBn//8ZyKWhP4rmGFhYSgtLX3p89u2betSqkBHPo8lJSU4duwYLl++rIky9Q4OhwMHBwfExcUhLi4OVVVV+OWXX7B9+3bk5uZi8uTJEAgECAgIaLU1WFFRgezsbI02l5iYmMDJyQlOTk7sVhxjvWZjYwMejwdLS0uNCBszEM/lcvU+U7DlTQUzxpOVlYX6+nrQNA1vb+9elcXJoM7oS3s+t+Xl5UhPT283tk2pVCI+Ph48Hg/bt29XWSzVybMEXpwze3l5YdasWe2arhN0S78VzAsXLqj1/R35PN67dw9PnjyBm5sbgBd/BG5ubnjy5IlaP09f4XK5WLhwIRYuXIj6+nqcPXsWBw8eRHx8PIKDgxEdHY3i4mIcOXIEP//8s9a2AAcMGAAHBwc4ODiw2YRFRUWoq6vD4MGDWTN0Vd74GF/boUOHan3GT9MMHDgQjo6OoGkaFEVh6NChKC4uRlZWFptjqS0/V03CiKWnp6dGGtAGDRoEZ2dnODs7szdbubm52LBhA5ydnSGVSuHh4aGWWCqVSqxatapVn4NAIGiVZ7lz5072/5k8y5Z8+umnmDRpkmoXSdA4/VYw1WXChAnIyclBXl4ehg0bhiNHjrA+jy1Xrubm5n1WLNtiZmaG2bNnY/bs2ZDJZLh06RK++OIL5OTkIDQ0FBcuXMCUKVO0ft5oaGjIuglRFIWqqipIJBJkZ2d3uzmG2QIcMWIEeDyeVuvWFvn5+aiuroafnx8MDQ3h4ODAvi5lZWVsogpjlqBvEVCM3aC25kRb3mwlJiZi5cqVKCkpQX5+PsrKyiAQCDBt2rRuG1LcunULbm5urAH6nDlzcOLEiQ4DoA8fPoytW7eyH9+5cwdlZWWYNm0aUlNTVb9AgsbQ79vKHkIkEsHR0RE3btxAVFQUpk6dCgAoKSlBZGQkgBepHXv27MHUqVPh5eWFmJiYl0yR+zMDBgzAjRs3MGTIEOTk5GDlypVISUnB5MmTMX/+fPz000+oqanReh0GBgawsbHBqFGjEBQUBCcnJ9TU1OD27dtIS0tDSUkJ5HJ5u9/b0NCAe/fuwd3dvVeKJRNcXVdXh7Fjx7YSQuZ18fLyYl+Xuro6pKam4t69eyguLoZMJuvB6l/Q0NCA+/fvw8vLS+umCkqlEuvWrYO7uztu3ryJtLQ0fPjhh3jw4AEyMzO7/Xzt9TkUFxe3+9iCggLk5eWxYQmMm9COHTtUuxiCViArzHaYNWsWZs2a9dLnhw4ditOnT7MfM1mVndHbO2RVJTs7G9XV1Th69CgMDQ3x5ptv4s0332STJBITE8Hn82FjYwM+n48ZM2bA1tZWq400bZtjpFIpJBIJ7t27x65KbW1tYWJiwprAe3t7w9LSUms1aYuW3bw+Pj6dvq5tz/daNg0BgK2tLXg8HgYNGqSr8gH8r0ls9OjRWv83UCgUWLlyJVxcXPCHP/yB3YYNCAjocA77VaiTZ/nVV18hMjJS6wbyhO5BxkoIPQbj9CMUCvHLL7/A0NAQUVFRiI6OhpOTk067UJuamliD+ObmZsjlcnh7e3c6tqCvMJFvRkZGaqfUNDc3s3ONMpmMbaaysLDQ6r9PfX09Hjx4oBMHIoVCgbi4OIwYMQJ/+MMfNHZdnY2etcXf3x///Oc/ERwcDACYN28erl27BgMDA0ilUshkMqxcufKlRBKC1iBzmAT9haZpFBcXQyQS4fjx45BKpZg2bRoEAgFGjRqlM/GUSCR48uQJhg4diqqqKjQ1NbEZlprquNUmFEXh0aNHMDMzg6urq0brVSgUqKiogEQiQV1dHWsKoGozVUcwdn2qGNl3F4VCgRUrVmDkyJEaFUvmuVXNs2zJoUOHkJqaSrpkdQuZwyToLxwOB46OjoiPj0d8fDwqKipw8uRJbN26FUVFRQgLC4NAIIC/v7/WOjpLSkpQXFyMgIAADBgwAC4uLqxIFBYWQiqV6nVnqVKpxMOHD7UWnGxkZAQ7OzvY2dm91EylKftCZitcF3Z9jFi6ubnh97//vcZvhlr2OTB5lt7e3i/lWR4+fBhz5szR+5sxAllh6j3q+trOmzcPqampGDBgAF577TV8/fXXvW4Gr66uDmfOnIFQKER6ejomTpwIgUCA4OBgjXm4FhQUoLKy8qXmmJa0l2HJGMT3dGcpYzdoZ2en9SzItrRnX6hKCHRtbS0eP36MsWPHav28VKFQYPny5fDw8MDWrVt7lVjRNI2JEyciISEB06dPBwAcPXoUBw8eRFJSUg9X12cgW7K9EXV9bU+fPs3+Ub333nsICQlBXFycri9DYzQ3NyM5ORlCoRApKSkICAgAn8/H5MmTVZrxpGmaNTvw9vbu8qqRybAUi8WorKyEqakp6xyj6xsSxoTc0dFRL9ylmBBoiUQCmqZbmaF3RE1NDTIyMnTibatQKPDBBx9g1KhR2LJlS68SS4ZHjx7h3Xffxb1796BUKuHn54ekpCSMHDmyp0vrKxDB7M2o6mvbkp07d6K8vBzbtm3TfsE6QKFQ4Pr16xAKhbh8+TI8PT0hEAgQERHRpUYRpjnGwMAAnp6eKr9x0jQNqVTKZlgytmvdXWGpAuOrOnz4cL0cfZHJZKx4dnQeXF1djczMTJ2IpVwuxwcffMBujfZGsWT46KOPYGZmhvr6elhYWODTTz/t6ZL6EuQMs6/Sma8tg1wux3fffYfdu3frujytYWRkhNDQUISGhoKiKNy9exdCoRA7d+6Evb09BAIBIiMj2+10pSgKDx8+hIWFRbcN5NvC4XBgYWEBCwsLjBw5Eo2NjRCLxXj48CG7wtLGWAbjfuPu7v5KX9WewtjYGMOGDcOwYcPadWAyNTXF8+fP4e/vr3UjeLlcjmXLlmHMmDH49NNPe7VYAsDnn3+OcePGwdjYmBgb6AgimHqAtnxtW7Jy5UqEhIRg4sSJqheqxxgYGLAzc9u2bUNmZiaEQiFiY2NhYmKCGTNmQCAQYOjQoaiursbevXsxf/58rcy5mZqawsXFBS4uLpDJZBCLxcjKyoJMJtNYAHRDQwMePHigNfcbbdDWgamoqIg1Qs/JyWGdhrSRLSqXy7F06VL4+vrik08+6fViCbxw1oqNjYW5ubnWdzIILyCCqQdoy9eWYevWrZBIJPj666/V+jm9BQ6HAy8vLyQkJODjjz9GYWEhRCIRPvjgA0ilUtTU1GDBggU6aY4xNjaGo6MjHB0doVAoUF5ejry8PDYAmum47c4bODN20VtNFYAXeZylpaUICgqCsbExu6VdUFCg8S1tuVyOJUuWwN/fHwkJCX1CLBkMDAz0rlu7L0MEsw/Qka8tABw4cABnz55FcnJyv/zD4nA4cHFxwZo1azBr1ixER0cjKioKKSkpCAkJQXh4OAQCAcaOHav118fIyAj29vawt7cHRVGoqKjA8+fPkZmZCSsrK7bjtrM6amtrkZ6erpMZRW0hkUiQl5fXKmKssy3tlkki3YURy3HjxuHjjz/uU2JJ0D2k6UfPEYlEiI+Ph0QiweDBg+Hn54ezZ8+ipKQEy5YtY636Tp8+jTVr1rDzXgkJCQBevEm7uLiwTTCzZ8/GZ5991mPX01MUFBRg5syZ+Ne//oWgoCAALzozT506BZFIhKysLISGhoLP5yMoKEinYyI0TaO6uprtuDUzM2M7bltuTzLNMboYu9AWLfM4u9JNLJPJWKehpqYm2NjYwNbWFlZWVq8UP5lMhiVLliAgIACbN2/uk2K5ZcsWmJubY/369T1dSl+DdMkS+i8ymQyFhYVs7FpbGhsbcf78eQiFQqSmpiIoKAh8Ph8hISE6PR9q23FrbGwMHo8HQ0ND5Ofnw8/PT+vNMdqirKwMhYWF8PPzU2n0hmkakkgkqK2t7XRVLpPJ8P777yMwMBAbN25UWSxVzbNMS0tDXFwcamtrYWhoiISEBMTGxqpUA6FHIIJJIHQFuVyOa9euITExEVevXoW3tzcEAgHCw8O17j7TloaGBuTm5kIsFsPc3Bx2dnbg8XhaH7/QNKWlpXj27Bn8/Pw00tRDUVSrOVgzMzM0NTXB1dUVlpaWWLx4MV5//XV89NFHKotlZ/PN7cHkWR48eBDZ2dngcDhwd3dHSUkJxo8fj4yMjF7ToEUgYyUEQpcYMGAA3nrrLbz11lugKAq3b99GYmIiduzYAScnJ/D5fERGRmokyPhV1NTUoLGxERMnTgRFURCLxcjIyIBCoWh1tqfP243Pnz9HSUmJxsQSeNHswuVyweVy2VX5d999h5UrV0Imk2Hs2LGYP3++Wq+LOnmWHh4e7OeHDh0KHo/HHqsQei/9rwuEoBbHjh1jHXE6m/1KSkqCp6cn3NzcWiUs5OXlITAwEO7u7oiNjdWLzMXOMDAwQGBgIL788kvcvXsXX3zxBUpKSjB79mwIBALs378fpaWl7Y72qMuzZ89QUlLCnvcNHDgQTk5OGDduHPz9/WFqaoqnT5/i5s2bbJyaNupQh+LiYo2LZVuYOdilS5fCzc0NsbGxmDRpEhYtWoQ33ngD33//vUrPq06eZUtu3boFmUxGXHj6AEQwCd3Cx8cHQqEQISEhHT5GqVRi1apVOHPmDB4/fozDhw/j8ePHAICNGzdi7dq1yMnJAZfLxb///W9dla42HA4HPj4++Pzzz5GSkoJ9+/ZBLpdj8eLFiIiIwK5du/D06VONiFZBQQEkEkmHQjNgwAA4ODjA19cXEyZMwODBg1FcXIyUlBRkZGSgoqICFEWpXYc6PHv2DGVlZfDz89N6E1VzczMWLVqEkJAQ/PGPf8Tq1atx4cIF/PLLL/D391fpOdXJs2R4/vw5FixYgG+++aZfdqn3Nci/IKFbeHl5wdPTs9PHtNzKMjY2ZreyaJrGxYsX8c477wAAFi1ahOPHj+uibI3D4XDg6uqK9evX48qVK/j555/B5XKxfv16hIaG4osvvsCjR4+6LVqMt21NTQ18fX27JDSMIYC3tzcCAwNhZ2cHiUSCmzdv4uHDhygrK4NSqVT1UlWisLAQEomky9egDs3NzVi4cCEmTZqEdevWtRI1a2vrl+K0usqr5ptbcuTIEcydO7fV52praxEVFYU//vGPbGc2oXdDzjAJGqcjq76KigoMHjyYXTF1tsXVm+BwOHBwcEBcXBzi4uJQVVWFX375Bdu3b0dubi4mT54MgUCAgICATsWDCdSWy+UYM2aMSudvBgYGsLa2hrW1NWiaRm1tLTv3aGJiwo6rMPOP2qCgoADV1dXw9fXV+qqqubkZCxYswFtvvYW1a9dq9Cy3s/nmlmRlZaGqqgqvv/46+zmZTIZZs2Zh4cKFePfddzVWE6FnIYJJeAltWfV1Z4urN8PlcrFw4UIsXLgQ9fX1OHfuHA4ePIj4+HgEBwcjOjoab775ZqvRCoqikJmZCUNDQ4wePVojrwuHw4GVlRWsrKzg5uaG+vp6iMVipKWlwdDQkPW41eSYSl5eHurq6jBmzBiti2VTUxMWLFiAsLAwrFmzRq/yLI8ePYqrV6+ioqIChw4dAvAiCNrPz0+jNRJ0CxkrIaiEKukpmzZtgq2tLUpLS2FkZPTS4/o6MpkMly9fhlAoxPXr1+Hn58fOei5duhSLFy9GVFSUTm4impqaIJFIIBaLoVQqWfFUZ2wmNzcX9fX13YpJU5WmpibMnz8fERERWL16dZ+88SL0KO3+QpEzTILGabmVJZPJcOTIEQgEAnA4HEyePBk///wzAODbb7/t0oq1r2BsbIyIiAjs3bsX9+/fx+9+9zv8+uuv8PPzQ3NzM6RSKWpra3VSi4mJCZycnDB+/Hj4+/tj4MCByMnJQUpKCnJyclBTU9Pl5iWapvHkyRM0NDTAx8dHJ2I5b948TJ06lYglQaeQFSahW6hr1Zebm4s5c+agsrIS/v7++P777/tt0kJ9fT3efvtt8Pl8vPHGG0hMTERSUhKsra3B5/MRFRUFHo+nU0Fg3HTEYjHq6urA5XJZg/j2hJARS7lcDi8vL63Xyojl9OnTER8fT8SSoC2I0w+BoE+sX78evr6+WLBgAfs5RoCEQiFOnjwJQ0NDREVFITo6Gk5OTjoVCIqiUFVVxdq9WVhYgMfjwcbGBoaGhqBpGtnZ2aAoCqNGjdJ6bY2NjZg3bx5mzJiBVatWEbEkaBMimASCPkFRVKfblzRNo6SkBCKRCMePH0dtbS2mT58OPp+vk9Vc21pqamogkUhQUVEBExMTKJVKmJqa6qSWxsZGvPfeexAIBFi5ciURS4K2IYJJIPRmKioqcPLkSYhEIhQVFSEsLAx8Ph/jxo3T6VA8RVF4+PAhmpubAbzoJmWahrSxvd7Y2Ii5c+di5syZiIuLI2JJ0AVEMAmEvkJdXR3OnDkDoVCI9PR0TJw4EQKBAMHBwVqzoANerDQfP34MY2NjuLm5gcPhoLGxke24pWkatra2sLW11YhRPRFLQg9BBJPQ96isrERsbCzy8/MxfPhwHD16tF1T9I0bN+LUqVMAgE8//ZSNWkpOTsaGDRtAURTMzc1x6NChDiPA9JXm5mYkJydDJBLhxo0bGD9+PAQCASZPnqzRGUuappGeng5TU1O4urq2K14ymYwVT5lMBhsbG/B4PFhYWHRb7BoaGjB37ly8/fbbWLFiBRFLgi4hgknoe3z00UewtrbGpk2bsH37dlRVVeHPf/5zq8ecOnUKu3btwpkzZ9Dc3IxJkybh4sWLsLS0hIeHB06cOAEvLy989dVXuHXrFjto3htRKBS4fv06RCIRLl26BA8PD0RHRyMiIoINEVcFiqLw6NEjmJubs+kdXamF6biVSqWwtrZmO25fJX4NDQ2YM2cO3n33XSxfvpyIJUHXkDlMQt/jxIkTWLRoEYCOvWkfP36MSZMmwcjICGZmZvD19UVSUhKAF244zOxjTU1Nh16hvQUjIyOEhoZi9+7dSEtLw+bNm5GRkYHIyEi88847+M9//oPy8vJuPSdzZmlpadllsWRqsbOzw5gxYxAYGAgbGxs8f/4cKSkpSE9Ph0Qiadfjtr6+HrGxsYiJiVFLLDtKzGFYu3Yt/Pz84OfnBw8Pj1bRW99++y3c3d3h7u6Ob7/9VqWfT+h7kBUmoVczePBgVFdXsx9zuVxUVVW1esy5c+ewdetWnD9/Hg0NDXjttdewatUqrFu3DteuXcPMmTNhamoKS0tLpKSkwNLSUteXoXVomkZmZiaEQiH++9//wsTEBDNmzIBAIMDQoUM7FCWKovDgwQNwuVy4uLhorBYm/LmiogJmZmaoq6uDl5cXBg4ciNjYWMydOxfLli3rkfDnyspKBAQEIDU1FRwOB+PHj8ed4tdX8gAACd1JREFUO3d0kn9K0BvICpPQOwkLC4OPj89L/504caJL3x8REYHIyEgEBwdj7ty5eP3119nGmJ07d+L06dN49uwZ3n//ffzf//2fNi+lx+BwOPDy8kJCQgJ+++03HDp0CIaGhli+fDnCwsLw17/+FdnZ2a3cfRoaGnD37l3Y2NhoTCyZWgYPHgwPDw8EBQVhxIgRuHnzJsLDwxEYGAgnJyfWGUpVOkrM6YjDhw+zaSNnz55FeHg4rK2tweVyER4ezu5IEPo3xHydoPdcuHChw6/Z2dnh+fPncHBwwPPnz8Hj8dp9XEJCAus29N5778Hd3R0SiQT3799HYGAgACA2NhbTpk3T/AXoGRwOBy4uLlizZg1Wr14NiUSCEydOICEhAaWlpQgPD0dERAQ+/vhjrFy5sl2/YE3WYmFhgRUrVuDixYuIiIjAgAEDEBMTAw6Hgw8//BCzZ8/u9vN2lJjTHm3Dn7sTHE3oX5AVJqFXIxAI2DOmjrxpGbs3AHjw4AEePHiAiIgIcLlc1NTUIDs7GwBw/vx5eHl56a54PYDD4YDH4+GDDz7AqVOncPHiRXh6emL+/PkAgNu3b+PXX3/Vap6mVCpFbGws5s+fj02bNmHdunW4cuUKjhw58srs1Y5QJ/y5v6TqELoPWWESejWbNm1CTEwM/v3vf8PZ2RnHjh0DAKSmpmLv3r04cOAA5HI5Jk6cCACwtLTE999/z27J7t+/H2+//TYMDAzA5XJx8ODBHrsWfcDAwADff/89vvzyS7zzzjs4f/48fvjhB6xduxaBgYEQCAQICQnRmEEBI5YLFy7E+++/3+pr9vb2sLe3V+l5uxv+/M9//rPV916+fLnV94aGhqpUB6FvQZp+CAQCy9dffw0rKyvMmTOn1eflcjmuXbuGxMREXL16Fd7e3hAIBAgPD1fZoEAqlSImJgaLFy/G4sWLNVD9/1AoFPDw8EBycjKGDRuGCRMm4Mcff4S3t3erx2VlZWHq1KnIy8tjV5GVlZUYP3487t69CwAYN24c7ty5A2tra43WSNBryBwmoecpKipCSEgI+wZUVVWFcePG4fLlyxptLCFoD4qicPv2bSQmJuL8+fNwcnLCjBkzEBkZ2WVRqaurQ0xMDJYsWcKOBWma9hJz2oY/b9myBU1NTS+NnRw8eBBffPEFgBfn321Xv4Q+DxFMgn7w5Zdf4smTJ9i3bx9WrFiB4cOHY/PmzT1dFkEFGPefxMREnD59GhYWFuy4ip2dXbtnf4xYLl26FAsXLuyBqgmEV0IEk6AfyOVyjB8/HkuWLMH+/ftx7949GBsb93RZGkFdqz6apvHJJ5/g2LFjMDQ0RFxcHD788EOdXoOq0DSNvLw8NppMqVQiKioKfD6ftdJjxHLZsmWtYs0IBD2DCCZBfzh79iymTZuGc+fOITw8vKfL0RjqWvV98803uHTpEg4dOgQDAwOIxeIOR2X0GZqmUVZWhuPHj0MkEqGyshKTJk3ClStXsHr1arYLl0DQU4hxAUF/OHPmDBwcHPDo0aOeLkWjqGvV969//QufffYZG9fVG8USeDGGYW9vj9/97nc4e/Yszp07Bzs7O0yZMoWIJaHXQgSToHPS0tJw/vx5pKSkYOfOnXj+/HlPl6QxysrK4ODgAABwcHCAWCx+6TG+vr44c+YMGhoaUF5ejkuXLrEjEE+fPsVPP/2EgIAATJ8+HTk5OTqtX1twuVxs2LChXU9XAqG3QOYwCTqFpmnExcVh165dcHZ2xoYNG7B+/Xr88MMPPV1alwkLC0NpaelLn9+2bVuXvj8iIgK3b99GcHAwbG1tW1n1NTc3w8TEBKmpqRAKhViyZAmuXbum0foJBIJqkDNMgk7Zt28fkpOT8dNPPwF44cLz2muv4W9/+xsmTZrUw9Wpj6enJy5fvsxa9YWGhiIrK6vT73nvvfcwf/58REZGYtSoUUhKSsLw4cNB0zQGDx6MmpoaHVVPIBD+P+QMk9DzLF++nBVLADA0NMSdO3f6hFgC6ln1AcDMmTNx8eJFAMCVK1fg4eGho8oJBMKrICtMAkGDVFRUICYmBoWFhaxVn7W1dSurvqamJowbNw7AC6u+vXv3ws/PDwBQXV2NefPmobCwEObm5ti7dy98fX178pIIhP4IGSshEAi9g6SkJKxevRpKpRLLli3Dpk2bXnrM0aNHsWXLFnA4HPj6+uLHH38E8GK059SpU6AoCuHh4di9ezcxTyd0l3Z/YUjTD4FA0CuUSiVWrVrVKvxZIBC0Cn/OycnBn/70J/z666/gcrlsN/Jvv/2GX3/9FQ8ePAAAvPnmm7hy5QoxTydoBHKGSSAQ9IquhD/v378fq1atYl2UmHlVDoeDpqYmyGQyNDc3Qy6Xw87OTufXQOibEMEkEAh6RVcCnLOzs5GdnY033ngDQUFBrPHD66+/jsmTJ8PBwQEODg6YOnVqv8s4JWgPIpgEQh+nsrIS4eHhcHd3R3h4OKqqqtp93MaNG+Hj4wMfH59WncwM8fHxMDc313a5XQpwVigUyMnJweXLl3H48GEsW7YM1dXVePLkCTIyMvDs2TMUFxfj4sWLuHr1qtZrJvQPiGASCH2c7du3Y8qUKcjJycGUKVPadds5deoU7t69i7S0NNy8eRM7duxAbW0t+/XU1FRUV1frpN6uhD87OjoiOjoaAwYMwIgRI+Dp6YmcnByIRCIEBQXB3Nwc5ubmmD59OlJSUnRSN6HvQwSTQOjjqOtvq1QqsWHDBnz55Zc6qXfChAnIyclBXl4eZDIZjhw5wuZXMsycOROXLl0CAJSXlyM7Oxuurq5wdnbGlStXoFAoIJfLceXKFbIlS9AYRDAJhD6Ouv62e/bsgUAgYJ9D2xgZGWHPnj3s+WNMTAy8vb3x2Wef4eTJkwCAqVOnwsbGBqNHj8bkyZOxY8cO2NjY4J133sHIkSMxZswY+Pr6wtfXF3w+Xyd1E/o+ZA6TQOgDdOZvu2jRolbbqVwut91zzG3btuHYsWOwtbUFj8fDa6+9hnfffRcxMTG4fPkyjIyMYG5uDqlUqtVrIRD0AGJcQCD0R9Txt6VpGkuXLoWJiQkAoLCwEK6urnjy5IkuSicQegriJUsg9EfU8beNiopCaWkp8vPzkZ+fj0GDBhGxJPRbyAqTQOjjqOtv2xKyJUvoJ5AtWQKBQCAQugDZkiUQCAQCQVWIYBIIBAKB0AWIYBIIBAKB0AWIYBIIBAKB0AWIYBIIBAKB0AWIYBIIBAKB0AWIYBIIBAKB0AWIYBIIBAKB0AWMXvH1doc3CQQCgUDob5AVJoFAIBAIXYAIJoFAIBAIXYAIJoFAIBAIXYAIJoFAIBAIXYAIJoFAIBAIXYAIJoFAIBAIXeD/ARItTCsDjXT6AAAAAElFTkSuQmCC\n", "text/plain": [ "
" ] }, "metadata": { "needs_background": "light" }, "output_type": "display_data" } ], "source": [ "%matplotlib inline\n", "import numpy as np\n", "import sm\n", "from mpl_toolkits.mplot3d import Axes3D\n", "import matplotlib.pyplot as plt\n", "\n", "## test data (1581 points)\n", "X1=np.loadtxt(\"./dat/sphere.dat\")\n", "\n", "## plot\n", "fig = plt.figure(figsize=(8,8))\n", "ax = fig.add_subplot(111, projection='3d')\n", "ax.scatter(X1[:,0], X1[:,1], X1[:,2], s=2)\n", "ax.set_xlabel('X')\n", "ax.set_ylabel('Y')\n", "ax.set_zlabel('Z')\n", "plt.show()\n" ] }, { "cell_type": "markdown", "metadata": {}, "source": [ "Now estimate best fitting sphere with RANSAC. In our case `min_samples = 4`. Value of `residual_threshold` should be neither small nor large. After trial and error the value `0.07` might seem good." ] }, { "cell_type": "code", "execution_count": 10, "metadata": {}, "outputs": [ { "name": "stdout", "output_type": "stream", "text": [ "parameters : [-1.0067282 0.75263481 66.65209555 0.06952197]\n", "number of points : 1581\n", "number of outliers: 156\n" ] } ], "source": [ "ransac_model, inliers = sm.ransac(X1, SphereModel, 4, 0.003)\n", "\n", "np.set_printoptions(suppress=True)\n", "print(\"parameters : \", ransac_model.params)\n", "print(\"number of points : \", X1.shape[0])\n", "print(\"number of outliers: \", X1.shape[0]-np.sum(inliers))" ] }, { "cell_type": "markdown", "metadata": {}, "source": [ "If we repeat this calculation we get slightly different sets of parameters and number of outliers. This is due to the randomized nature of RANSAC - results depend on chance.\n", "\n", "Plot consensus set and outliers:" ] }, { "cell_type": "code", "execution_count": 11, "metadata": {}, "outputs": [ { "data": { "image/png": "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\n", "text/plain": [ "
" ] }, "metadata": { "needs_background": "light" }, "output_type": "display_data" } ], "source": [ "fig = plt.figure(figsize=(8,8))\n", "ax = fig.add_subplot(111, projection='3d')\n", "ax.scatter(X1[inliers,0], X1[inliers,1], X1[inliers,2],s=2)\n", "outliers = np.invert(inliers)\n", "ax.scatter(X1[outliers,0], X1[outliers,1], X1[outliers,2],s=2)\n", "ax.set_xlabel('X')\n", "ax.set_ylabel('Y')\n", "ax.set_zlabel('Z')\n", "plt.show()" ] }, { "cell_type": "markdown", "metadata": {}, "source": [ "### Fit residuals\n", "Plot histogram of residuals of conform points." ] }, { "cell_type": "code", "execution_count": 12, "metadata": {}, "outputs": [ { "name": "stdout", "output_type": "stream", "text": [ "mean: -0.00 %\n", "std : 0.72 %\n", "max: 5.49 %\n" ] }, { "data": { "image/png": "iVBORw0KGgoAAAANSUhEUgAAAmAAAAFlCAYAAABMTlT+AAAABHNCSVQICAgIfAhkiAAAAAlwSFlzAAALEgAACxIB0t1+/AAAADh0RVh0U29mdHdhcmUAbWF0cGxvdGxpYiB2ZXJzaW9uMy4xLjMsIGh0dHA6Ly9tYXRwbG90bGliLm9yZy+AADFEAAAU8klEQVR4nO3dcYxl1X0f8O+vjDFJXJsFxi7dRV2soDRO1NZ0RUlTRWw2tYFYXiIFyVUbTx2q/aM0dUqlmNRSrab/xElVu1YrR8jQjlXXsUscgSwal+JBaaUae7EdjE1c1liFKRQ2ApO0VuJQn/4xZ82wzLKz8+adNzPv85Ge3r3nnnvvuYeZ5TvnnndftdYCAMA4f2bWDQAAmDcCGADAYAIYAMBgAhgAwGACGADAYAIYAMBgC7NuwCu55JJL2sGDB2fdDACAs3rwwQf/oLW2uJm6OzqAHTx4MMePH591MwAAzqqq/udm67oFCQAwmAAGADCYAAYAMJgABgAwmAAGADCYAAYAMJgABgAwmAAGADCYAAYAMJgABgAwmAAGADCYAAYAMJgABgAw2MKsGwBMx+Hlw5uqt7K0MuWWAHA6I2AAAIMJYAAAgwlgAACDCWAAAIMJYAAAgwlgAACDCWAAAIMJYAAAgwlgAACDCWAAAIMJYAAAgwlgAACDCWAAAIMJYAAAgwlgAACDCWAAAIMJYAAAgwlgAACDCWAAAIMJYAAAgwlgAACDCWAAAIMJYAAAgwlgAACDCWAAAIMJYAAAgwlgAACDCWAAAIMJYAAAg501gFXVHVX1TFU9vK7soqq6t6oe7e/7enlV1Yeq6kRVPVRVV67bZ6nXf7SqlqZzOQAAO99mRsD+XZJrTyu7Ncl9rbUrktzX15PkuiRX9NexJB9O1gJbkvcl+WtJrkryvlOhDQBg3pw1gLXWfjfJs6cVH02y3JeXk9ywrvyjbc3nklxYVZcmeWuSe1trz7bWnktyb14e6gAA5sJW54C9obX2VJL099f38v1JnlhXb7WXnakcAGDubPck/NqgrL1C+csPUHWsqo5X1fGTJ09ua+MAAHaCrQawp/utxfT3Z3r5apLL1tU7kOTJVyh/mdbaba21Q621Q4uLi1tsHgDAzrXVAHZ3klOfZFxKcte68nf2T0NeneT5fovyM0neUlX7+uT7t/QyAIC5s3C2ClX18STXJLmkqlaz9mnGX03yyaq6KcnjSW7s1e9Jcn2SE0m+neRdSdJae7aq/nmSL/R6v9JaO31iPwDAXDhrAGut/a0zbDqyQd2W5OYzHOeOJHecU+sAAPYgT8IHABhMAAMAGEwAAwAYTAADABhMAAMAGEwAAwAYTAADABhMAAMAGOysD2IFxji8fHhT9VaWVqbcEgCmzQgYAMBgAhgAwGACGADAYAIYAMBgAhgAwGACGADAYAIYAMBgAhgAwGACGADAYAIYAMBgvooI5txmvwIp8TVIANvFCBgAwGACGADAYAIYAMBgAhgAwGACGADAYAIYAMBgAhgAwGACGADAYAIYAMBgAhgAwGACGADAYAIYAMBgAhgAwGACGADAYAIYAMBgAhgAwGACGADAYAIYAMBgAhgAwGACGADAYAIYAMBgAhgAwGACGADAYAIYAMBgC7NuAHBuDi8fnnUTAJiQETAAgMEEMACAwSYKYFX1j6rqq1X1cFV9vKouqKrLq+qBqnq0qj5RVef3uq/u6yf69oPbcQEAALvNlgNYVe1P8g+THGqt/WiS85K8I8n7k3ygtXZFkueS3NR3uSnJc621H0zygV4PAGDuTHoLciHJ91XVQpLvT/JUkp9Mcmffvpzkhr58tK+nbz9SVTXh+QEAdp0tB7DW2v9K8i+SPJ614PV8kgeTfKu19kKvtppkf1/en+SJvu8Lvf7Fpx+3qo5V1fGqOn7y5MmtNg8AYMea5BbkvqyNal2e5M8n+YEk121QtZ3a5RW2vVjQ2m2ttUOttUOLi4tbbR4AwI41yS3In0ryzdbaydbanyb5VJK/nuTCfksySQ4kebIvrya5LEn69tcleXaC8wMA7EqTBLDHk1xdVd/f53IdSfK1JCtJfrbXWUpyV1++u6+nb/9sa+1lI2AAAHvdJHPAHsjaZPovJvlKP9ZtSd6T5JaqOpG1OV63911uT3JxL78lya0TtBsAYNea6KuIWmvvS/K+04ofS3LVBnX/OMmNk5wPAGAv8CR8AIDBBDAAgMEEMACAwQQwAIDBBDAAgMEEMACAwQQwAIDBBDAAgMEEMACAwQQwAIDBBDAAgMEEMACAwQQwAIDBBDAAgMEEMACAwQQwAIDBBDAAgMEEMACAwQQwAIDBFmbdAGD3OLx8eFP1VpZWptwSgN3NCBgAwGACGADAYAIYAMBgAhgAwGACGADAYAIYAMBgAhgAwGACGADAYAIYAMBgAhgAwGACGADAYAIYAMBgAhgAwGACGADAYAIYAMBgAhgAwGACGADAYAIYAMBgAhgAwGACGADAYAIYAMBgAhgAwGACGADAYAuzbgCw9xxePrypeitLK1NuCcDOZAQMAGAwAQwAYLCJAlhVXVhVd1bV71fVI1X1Y1V1UVXdW1WP9vd9vW5V1Yeq6kRVPVRVV27PJQAA7C6TjoD9qyS/01r7i0n+cpJHktya5L7W2hVJ7uvrSXJdkiv661iSD094bgCAXWnLk/Cr6rVJfiLJ302S1tp3knynqo4muaZXW05yf5L3JDma5KOttZbkc3307NLW2lNbbj3sApudkA7A/JhkBOyNSU4m+bdV9aWq+khV/UCSN5wKVf399b3+/iRPrNt/tZcBAMyVSQLYQpIrk3y4tfbmJP83L95u3EhtUNZeVqnqWFUdr6rjJ0+enKB5AAA70yQBbDXJamvtgb5+Z9YC2dNVdWmS9Pdn1tW/bN3+B5I8efpBW2u3tdYOtdYOLS4uTtA8AICdacsBrLX2v5M8UVU/1IuOJPlakruTLPWypSR39eW7k7yzfxry6iTPm/8FAMyjSZ+E/wtJPlZV5yd5LMm7shbqPllVNyV5PMmNve49Sa5PciLJt3tdAIC5M1EAa619OcmhDTYd2aBuS3LzJOcDANgLPAkfAGAwAQwAYDABDABgMAEMAGAwAQwAYDABDABgMAEMAGAwAQwAYDABDABgMAEMAGAwAQwAYDABDABgMAEMAGAwAQwAYDABDABgMAEMAGAwAQwAYDABDABgMAEMAGAwAQwAYLCFWTcAmF+Hlw9vqt7K0sqUWwIwlhEwAIDBBDAAgMEEMACAwQQwAIDBBDAAgMEEMACAwQQwAIDBPAcMtmizz7ACgNMZAQMAGEwAAwAYTAADABhMAAMAGEwAAwAYTAADABhMAAMAGEwAAwAYTAADABhMAAMAGEwAAwAYTAADABhMAAMAGEwAAwAYTAADABhMAAMAGEwAAwAYTAADABhs4gBWVedV1Zeq6tN9/fKqeqCqHq2qT1TV+b381X39RN9+cNJzAwDsRtsxAvbuJI+sW39/kg+01q5I8lySm3r5TUmea639YJIP9HoAAHNnogBWVQeS/HSSj/T1SvKTSe7sVZaT3NCXj/b19O1Hen0AgLky6QjYB5P8UpLv9vWLk3yrtfZCX19Nsr8v70/yRJL07c/3+i9RVceq6nhVHT958uSEzQMA2Hm2HMCq6m1JnmmtPbi+eIOqbRPbXixo7bbW2qHW2qHFxcWtNg8AYMdamGDfH0/y9qq6PskFSV6btRGxC6tqoY9yHUjyZK+/muSyJKtVtZDkdUmeneD8AAC70pZHwFprv9xaO9BaO5jkHUk+21r720lWkvxsr7aU5K6+fHdfT9/+2dbay0bAAAD2umk8B+w9SW6pqhNZm+N1ey+/PcnFvfyWJLdO4dwAADveJLcgv6e1dn+S+/vyY0mu2qDOHye5cTvOBwCwm3kSPgDAYAIYAMBgAhgAwGACGADAYAIYAMBgAhgAwGACGADAYAIYAMBgAhgAwGACGADAYAIYAMBgAhgAwGACGADAYAIYAMBgAhgAwGACGADAYAIYAMBgAhgAwGACGADAYAIYAMBgAhgAwGACGADAYAIYAMBgC7NuAOw0h5cPz7oJAOxxRsAAAAYTwAAABhPAAAAGE8AAAAYTwAAABhPAAAAGE8AAAAYTwAAABhPAAAAG8yR8YMfb7LcTrCytTLklANvDCBgAwGACGADAYAIYAMBgAhgAwGACGADAYAIYAMBgAhgAwGACGADAYAIYAMBgAhgAwGACGADAYAIYAMBgWw5gVXVZVa1U1SNV9dWqencvv6iq7q2qR/v7vl5eVfWhqjpRVQ9V1ZXbdREAALvJJCNgLyT5x621H05ydZKbq+pNSW5Ncl9r7Yok9/X1JLkuyRX9dSzJhyc4NwDArrXlANZae6q19sW+/EdJHkmyP8nRJMu92nKSG/ry0SQfbWs+l+TCqrp0yy0HANiltmUOWFUdTPLmJA8keUNr7alkLaQleX2vtj/JE+t2W+1lpx/rWFUdr6rjJ0+e3I7mAQDsKBMHsKp6TZLfSvKLrbU/fKWqG5S1lxW0dltr7VBr7dDi4uKkzQMA2HEmCmBV9aqsha+PtdY+1YufPnVrsb8/08tXk1y2bvcDSZ6c5PwAALvRJJ+CrCS3J3mktfYv1226O8lSX15Kcte68nf2T0NeneT5U7cqAQDmycIE+/54kp9L8pWq+nIv+ydJfjXJJ6vqpiSPJ7mxb7snyfVJTiT5dpJ3TXBuAIBda8sBrLX237LxvK4kObJB/Zbk5q2eDwBgr/AkfACAwQQwAIDBBDAAgMEEMACAwQQwAIDBJnkMBewah5cPz7oJAPA9RsAAAAYTwAAABhPAAAAGE8AAAAYzCR+YO5v9UMbK0sqUWwLMKwGMXc2nGwHYjdyCBAAYTAADABhMAAMAGEwAAwAYzCR8YM/woQxgtzACBgAwmAAGADCYAAYAMJgABgAwmAAGADCYAAYAMJgABgAwmAAGADCYAAYAMJgABgAwmAAGADCYAAYAMJgABgAwmAAGADDYwqwbABs5vHx41k0AgKkxAgYAMJgABgAwmAAGADCYAAYAMJgABgAwmE9BMpRPNwKAETAAgOGMgAGcwbmM2K4srUyxJcBeI4ABbIPNhjVBDUjcggQAGM4IGNvC5HoA2DwjYAAAgwlgAACDuQUJMJDJ+juP/ybMwvARsKq6tqq+XlUnqurW0ecHAJi1oSNgVXVekn+T5G8mWU3yhaq6u7X2tZHtON08/vVj0jwwynb/e7OX/i1mfo2+BXlVkhOttceSpKp+M8nRJDMNYJs1q6A2jwER5p3QcmY7/d/ic7FX/n8xjYcW7/XfgdG3IPcneWLd+movAwCYG9VaG3eyqhuTvLW19vf6+s8luaq19gvr6hxLcqyv/lCSrw9r4Pa6JMkfzLoRe4S+3F76c/voy+2lP7ePvtxem+3Pv9BaW9zMAUffglxNctm69QNJnlxfobV2W5LbRjZqGqrqeGvt0KzbsRfoy+2lP7ePvtxe+nP76MvtNY3+HH0L8gtJrqiqy6vq/CTvSHL34DYAAMzU0BGw1toLVfUPknwmyXlJ7mitfXVkGwAAZm34g1hba/ckuWf0eWdg199G3UH05fbSn9tHX24v/bl99OX22vb+HDoJHwAA3wUJADCcAHYOquqiqrq3qh7t7/vOUG+p13m0qpbWlf/VqvpK/xqmD1VV9fJfr6rfr6qHquq3q+rCUdc0S1Pszxur6qtV9d2q2tOfAjrbV3tV1aur6hN9+wNVdXDdtl/u5V+vqrdu9ph72ZT6846qeqaqHh5zFTvDdvdlVV1WVStV9Uj//X73uKuZvSn05wVV9fmq+r3en/9s3NXM3jR+1/u286rqS1X16bM2orXmtclXkl9LcmtfvjXJ+zeoc1GSx/r7vr68r2/7fJIfS1JJ/lOS63r5W5Is9OX3b3TcvfiaYn/+cNaeIXd/kkOzvs4p9t95Sb6R5I1Jzk/ye0nedFqdv5/kN/ryO5J8oi+/qdd/dZLL+3HO28wx9+prGv3Zt/1EkiuTPDzra9zNfZnk0iRX9jp/Nsn/8LM5UX9Wktf0Oq9K8kCSq2d9rbu1P9ftd0uS/5Dk02drhxGwc3M0yXJfXk5ywwZ13prk3tbas62155Lcm+Taqro0yWtba/+9rf1X+uip/Vtr/7m19kLf/3NZez7aPJhWfz7SWtutD/A9F9/7aq/W2neSnPpqr/XW9/GdSY70kcKjSX6ztfYnrbVvJjnRj7eZY+5V0+jPtNZ+N8mzIy5gB9n2vmytPdVa+2KStNb+KMkjmZ9vUplGf7bW2v/p9V/VX/MyKXwqv+tVdSDJTyf5yGYaIYCdmze01p5Kkv7++g3qnOnrlvb35dPLT/fzWRvNmQcj+nMv28xXe32vTg/5zye5+BX2neevC5tGf86rqfZlvx305qyN2syDqfRnv1325STPZO0PXf25QZ1z+Pn8YJJfSvLdzTRi+GModrqq+i9J/twGm9672UNsUNZeoXz9ud+b5IUkH9vkuXa8WfbnHNhMH5xr/230R9m89Os0+nNeTa0vq+o1SX4ryS+21v5wyy3cXabSn621/5fkr/R5x79dVT/aWpuHuYrb3p9V9bYkz7TWHqyqazbTCAHsNK21nzrTtqp6uqouba091W+BPbNBtdUk16xbP5C1uUireemtxZd8DVOfXP62JEf6LbU9YVb9OSfO+tVe6+qsVtVCktdl7XbYK+17tmPuVdPqz3k0lb6sqldlLXx9rLX2qek0fUea6s9ma+1bVXV/kmuTzEMAm0Z/vj3J26vq+iQXJHltVf371trfOWMrZj0Zbje9kvx6Xjpp/Nc2qHNRkm9mbcL4vr58Ud/2hSRX58VJ49f38muTfC3J4qyvcS/057p978/enoS/kLUPJVyeFyeS/shpdW7OSyeSfrIv/0heOpH0saxNTD3rMffqaxr9uW6/g5mvSfjT+NmsrM31/OCsr2+P9Odikgt7ne9L8l+TvG3W17pb+/O0fa/JJibhz7wjdtMra/d/70vyaH8/FQQOJfnIuno/n7WJeSeSvGtd+aGs/XXxjST/Oi8+CPdE1u4pf7m/fmPW17rL+/NnsvZXyp8keTrJZ2Z9rVPsw+uz9mmwbyR5by/7lSRv78sXJPmPve8+n+SN6/Z9b9/v6+mfID3TMeflNaX+/HiSp5L8af+5vGnW17kb+zLJ38jaLaCH1v1bef3Ia9pj/fmXknyp9+fDSf7prK9xN/fnace+JpsIYJ6EDwAwmE9BAgAMJoABAAwmgAEADCaAAQAMJoABAAwmgAEADCaAAQAMJoABAAz2/wE6JhnSPFMOUQAAAABJRU5ErkJggg==\n", "text/plain": [ "
" ] }, "metadata": { "needs_background": "light" }, "output_type": "display_data" } ], "source": [ "def sphdist(sph,X,pos=1):\n", " \"\"\"\n", " sphdist(sph,X) calculate distances of points X from sphere sph\n", " sph: 4-element vector: [x0, y0, z0, r]\n", " x0, y0, z0: centre of sphere\n", " r: radius of sphere\n", " X: (n,3) matrix, coordinates of n points\n", " pos: if pos=0 calculate signed, otherwise unsigned deviations\n", " \"\"\"\n", " r = sph[3]\n", " c = sph[0:3]\n", " # distance from centre squared\n", " dc = np.sum((X-c)**2,axis=1)\n", " if pos==1:\n", " dist = np.abs(np.sqrt(dc)-r)\n", " else:\n", " dist = np.sqrt(dc)-r\n", " return dist\n", "\n", "# calculate signed distances (residuals)\n", "v = sphdist(ransac_model.params,X1[inliers,:],0)\n", "\n", "# residuals in percent relative to the radius of sphere\n", "print(\"mean: %7.2f\" % (100*np.mean(v)/ransac_model.params[3]) + \" %\")\n", "print(\"std : %7.2f\" % (100*np.std(v)/ransac_model.params[3]) + \" %\")\n", "\n", "# max. percent residual\n", "print(\"max: %7.2f\" % (100*np.max(np.abs((v)))/ransac_model.params[3]) + \" %\")\n", "\n", "# histogram of relative residuals\n", "fig = plt.figure(figsize=(10, 6))\n", "n, bins, patches = plt.hist(v, 50, density=True, facecolor='green', alpha=0.75)\n", "plt.show()" ] }, { "cell_type": "markdown", "metadata": {}, "source": [ "### Quantile-quantile (Q-Q) plot\n", "\n", "Check whether distribution is Gaussian by using a normal quantile-quantile plot." ] }, { "cell_type": "code", "execution_count": 13, "metadata": {}, "outputs": [ { "data": { "image/png": "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\n", "text/plain": [ "
" ] }, "metadata": { "needs_background": "light" }, "output_type": "display_data" } ], "source": [ "import scipy.stats as stats\n", "\n", "stats.probplot(v, dist=\"norm\", plot=plt)\n", "plt.show()" ] }, { "cell_type": "markdown", "metadata": {}, "source": [ "We see that tails follow a non-Gaussian distribution.\n", "\n", "### Additional RANSAC variants\n", "\n", "Many flavors of RANSAC have been developed during the years. We mention M-estimation MSAC, Maximum Likelihood MLESAC, or KALMANSAC. One flavor of RANSAC searches for multiple models in the data ([Toldo és Fusiello, 2008](https://pdfs.semanticscholar.org/0455/e5596d734e3dcf60c0179efb6404e62ceabb.pdf)). In BaySAC, instead of random model selection, probability of the model is maximised upon information from prior unplausible models. " ] } ], "metadata": { "kernelspec": { "display_name": "Python 3", "language": "python", "name": "python3" }, "language_info": { "codemirror_mode": { "name": "ipython", "version": 3 }, "file_extension": ".py", "mimetype": "text/x-python", "name": "python", "nbconvert_exporter": "python", "pygments_lexer": "ipython3", "version": "3.7.4" } }, "nbformat": 4, "nbformat_minor": 1 }